2015年高考英语试题含答案15套word版
加入VIP免费下载

2015年高考英语试题含答案15套word版

ID:588129

大小:5.03 MB

页数:208页

时间:2021-03-23

加入VIP免费下载
温馨提示:
1. 部分包含数学公式或PPT动画的文件,查看预览时可能会显示错乱或异常,文件下载后无此问题,请放心下载。
2. 本文档由用户上传,版权归属用户,天天资源网负责整理代发布。如果您对本文档版权有争议请及时联系客服。
3. 下载前请仔细阅读文档内容,确认文档内容符合您的需求后进行下载,若出现内容与标题不符可向本站投诉处理。
4. 下载文档时可能由于网络波动等原因无法下载或下载错误,付费完成后未能成功下载的用户请联系客服处理。
网站客服:403074932
资料简介
目录 2015年普通高等学校招生全国统一考试各地英语科试题及答案 2015年普通高等学校招生全国统一考试(新课标 I)英语试题................................................................5 (河北、河南、山西、江西、山东) 2015年普通高等学校招生全国统一考试(新课标 I)英语试题参考答案.............................................187 2015年普通高等学校招生全国统一考试(新课标 II)英语试题..............................................................15 (甘肃、广西、贵州、黑龙江、吉林、辽宁、内蒙古、宁夏、青海、西藏、新疆、云南、海南) 2015年普通高等学校招生全国统一考试(新课标 II)英语试题参考答案............................................188 2015年普通高等学校招生全国统一考试(北京卷)英语试题.................................................................25 2015年普通高等学校招生全国统一考试(北京卷)英语试题参考答案...............................................188 2015年普通高等学校招生全国统一考试(天津卷)英语试题.................................................................39 2015年普通高等学校招生全国统一考试(天津卷)英语试题参考答案...............................................189 2015年普通高等学校招生全国统一考试(上海卷)英语试题.................................................................49 2015年普通高等学校招生全国统一考试(上海卷)英语试题参考答案...............................................190 2015年普通高等学校招生全国统一考试(广东卷)英语试题.................................................................63 2015年普通高等学校招生全国统一考试(广东卷)英语试题参考答案...............................................191 2015年普通高等学校招生全国统一考试(江苏卷)英语试题.................................................................75 2015年普通高等学校招生全国统一考试(江苏卷)英语试题参考答案...............................................192 2015年普通高等学校招生全国统一考试(安徽卷)英语试题.................................................................89 2015年普通高等学校招生全国统一考试(安徽卷)英语试题参考答案...............................................193 2015年普通高等学校招生全国统一考试(浙江卷)英语试题.................................................................99 2015年普通高等学校招生全国统一考试(浙江卷)英语试题参考答案...............................................194 2015年普通高等学校招生全国统一考试(浙江卷)自选模块英语部分试题.......................................111 2015年普通高等学校招生全国统一考试(浙江卷)自选模块英语部分试题参考答案.......................195 2015年普通高等学校招生全国统一考试(福建卷)英语试题...............................................................113 2015年普通高等学校招生全国统一考试(福建卷)英语试题参考答案...............................................195 2015年普通高等学校招生全国统一考试(湖南卷)英语试题...............................................................127 2015年普通高等学校招生全国统一考试(湖南卷)英语试题参考答案...............................................203 2015年普通高等学校招生全国统一考试(湖北卷)英语试题...............................................................139 2015年普通高等学校招生全国统一考试(湖北卷)英语试题参考答案...............................................204 2015年普通高等学校招生全国统一考试(四川卷)英语试题...............................................................153 2015年普通高等学校招生全国统一考试(四川卷)英语试题参考答案...............................................205 2015年普通高等学校招生全国统一考试(重庆卷)英语试题...............................................................165 2015年普通高等学校招生全国统一考试(重庆卷)英语试题参考答案...............................................206 2015年普通高等学校招生全国统一考试(陕西卷)英语试题...............................................................175 2015年普通高等学校招生全国统一考试(陕西卷)英语试题参考答案...............................................207 2015 年普通高等学校招生全国统一考试 各地英语科试题 2015年普通高等学校招生全国统一考试(新课标 I) 英 语 注意事项: 1. 本试卷分第Ⅰ卷(选择题)和第Ⅱ卷(非选择题)两部分。 2. 答题前,考生务必将自己的姓名,准考证号填写在本试卷相应的位置。 3. 全部答案在答题卡上完成,答在本试卷上无效。 4. 第Ⅰ卷听力部分满分 30分,不计入总分,考试成绩录取时提供给高校作参考。 5. 考试结束后,将本试卷和答题卡一并交回。 第Ⅰ卷 第一部分 听力(共两节,满分 30 分) 做题时,现将答案标在试卷上,录音内容结束后,你将有两分钟的时间将试卷上的答案转涂 到答题卡上。 第一节(共 5小题;每小题 1.5分,满分 7.5分) 听下面 5段对话,每段对话后有一个小题。从题中所给的 A,B,C三个选项中选出最佳选项, 并标在试卷的相应位置。听完每段对话后,你都有 10秒钟的时间来回答有关小题和阅读下一小题。 每段对话仅读一遍。 例: How much is the shirt? A. £ 19.15 B. £ 9.18 C. £ 9.15 答案是 C。 1. 1. What time is it now? A. 9:10 B. 9:50 C. 10:00 2. What does the woman think of the weather? It’s nice. It’s warm It’s cold. 3. What will the man do? A. Attend a meeting. B. Give a lecture C. Leave his office. 4. What is the woman’s opinion about the course? A. Too hard B. Worth taking. C. Very easy. 5. What does the woman want the man to do? A. Speak louder B. Apologize to her. C. Turn off the radio. 第二节(共 15小题;每小题 1.5分,满分 22.5分) 听下面 5短话或独白,没段话或独白后有几个小题,从题中所给的 A、B、C三个选项中选出 最佳选项,并标在试卷的相应位置。听每段对话或独白,你将有时间阅读各个小题,每小题 5秒 钟;听完后,各小题将给出 5秒钟的做大时间,每段对话或独白读两遍。 听第 6段材料,回答第 6、7题。 6. How long did Michael stay in China? A. Five days. B. One week. C. Two weeks. 7. Where did Michael go last year? A. Russia B. Norway C. India 听第 7段材料,回答第 8、9题。 8. What food does Sally like? A. Chicken. B. Fish. C. Eggs. 9. What are the speakers going to do? A. Cook dinner. C. Go shopping. C. Order dishes. 听第 8段材料,回答第 10至 12题。 10. Where are the speakers? A. In a hospital. B. In the office. C. At home. 11. When is the report due? A. Thursday. B. Friday. C. Next Monday. 12. What does George suggest Stephanie do with the report? A. Improve it. B. Hand it in later. C. Leave it with him. 听第 9段材料,回答第 13至 16题。 13. What is the probable relationship between the speakers? A. Salesperson and customer. B. Homeowner and cleaner. C. Husband and wife. 14. What kind of apartment do the speakers prefer? A. One with two bedroom. B. One without furniture. C. One near a market. 15. How much rent should one pay for the one-bedroom apartment? A. $350. B. $400. C. $415. 16. Where is the apartment the speakers would like to see? A. On Lake Street B. On Market Street. C. On South Street. 听第 8段材料,回答第 10至 12题。 17. What percentage of the world’s tea exports go to Britain? A. About 15%. B. About 30%. C. Over 40%. 18. Why do tea tasters taste tea with milk? A. Most British people drink that way. B. Tea tastes much better with milk. C. Tea with milk is healthy. 19. Who suggests a price for each tea? A. Tea tasters. B. Tea exporters. C. Tea companies. 20. What is the speaker talking about? A. The life of tea tasters. B. Afternoon tea in Britain. C. The London Tea Trade Centre. 第二部分 阅读理解(共两节,满分 60分) 第一节 (共 15小题;每小题 3分,满分 45分) 阅读下列短文,从每题所给的四个选项(A、B、C、和 D)中,选出最佳选项,并在答题卡 上将该项涂黑。 A Monthly Talks at London Canal Museum Our monthly talks start at 19:30 on the first Thursday of each month except August. Admission is at normal charges and you don’t need to book. They end around 21:00. November 7th The Canal Pioneers, by Chris Lewis. James Brindley is recognized as one of the leading early canal engineers. He was also a major player in training others in the art of nanal planning and building. Chris Lewis will explain how Brindley made such a positive contribution to the education of that group of early “civil enginerrs”. December 5th Ice for the Metropolis, by Malcolm Tucker. Well before the arrival of freezers, there was a demand for ice for food preservation and catering, Malcolm will explain the history of importing natural ice and the technology of building ice wells, and how London’s ice trade grew. February 6th An Update on the Cotsword Canals, by Liz Payne. The Smoudwater Canal is moving towards reopenling. The Thames and Severn Canal will take a little longer. We will have a report on the present state of play. March 6th Eyots and Aits- Thames Islands, by Miranda Vickers. The Thames had many islands. Miranda has undertaken a review of all of them. She will tell us about those of greatest interest. Online bookings:www.canalmuseum.org.uk/book More into:www.canalmuseum.org.uk/whatson London Canal Museum 12-13 NewWharf Road, London NI 9RT www.canalmuseum.org.uk www.canalmuseum.mobi Tel:020 77130836 21.When is the talk on James Brindley? A. November 7th. B. March 6th. C. February 6th. D. December 5th. 22. What is the topic of the talk in February? A. The Canal Pioneers. B. Ice for the Metropolis C. Eyots and Aits- Thames Islands D. An Update on the Cotsword Canals 23. Who will give the talk on the islands in the Thames? A. Chris Lewis B. Malcolm Tucker C. Miranda Vickers D. Liz Payne B The freezing Northeast hasn’t been a terribly fun place to spend time this winter, so when the chance came for a weekend to Sarasota, Florida, my bags were packed before you could say “sunshine”. I left for the land of warmth and vitamin C(维生素 C), thinking of beaches and orange trees. When we touched down to blue skies and warm air, I sent up a small prayer of gratefulness. Swimming pools, wine tasting, and pink sunsets(at normal evening hours, not 4 in the afternoon) filled the weekend, but the best part- particularly to my taste, dulled by months of cold- weather root vegetables- was a 7 a.m. adventure to the Sarasota farmers’ market that proved to be more than worth the early wake-up call. The market, which was founded in 1979, sets up its tents every Saturday from 7:00 am to 1 p.m, rain or shine, along North Lemon and State streets. Baskets of perfect red strawberries, the red-painted sides of the Java Dawg coffee truck; and most of all, the tomatoes: amazing, large, soft and round red tomatoes. Disappointed by many a broken, vine-ripened(蔓上成熟的) promise, I’ve refused to buy winter tomatoes for years. No matter how attractive they look in the store, once I get them home they’re unfailingly dry, hard, and tasteless. But I homed in, with uncertainty, on one particular table at the Brown’s Grove Farm’s stand, full of fresh and soft tomatoes the size of my fist. These were the real deal- and at that moment, I realized that the best part of Sarasota in winter was going to be eating things that back home in New York I wouldn’t be experiencing again for months. Delighted as I was by the tomatoes in sight, my happiness deepened when I learned that Brown’s Grove Farm is one of the suppliers for Jack Dusty, a newly opened restaurant at the Sarasota Ritz Carlton, where- luckily for me- I was planning to have dinner that very night. Without even seeing the menu, I knew I’d be ordering every tomato on it. 24. What did the author think of her winter life in New York? A. Exciting. B. Boring. C. Relaxing. D. Annoying. 25. What made the author’s getting up late early worthwhile? A. Having a swim. B. Breathing in fresh air. C. Walking in the morning sun. D. Visiting a local farmer’s market. 26. What can we learn about tomatoes sold in New York in winter? A. They are soft. B. They look nice. C. They taste great. D. They are juicy. 27. What was the author going to that evening? A. Eat in a restaurant. B. Check into a hotel. C. Go to a farm. D. Buy fresh vegetables. C Salvador Dali (1904-1989) was one of the most popular of modern artists. The Pompidou Centre in Paris is showing its respect and admiration for the artist and his powerful personality with an exhibition bringing together over 200 paintings, sculptures, drawings and more. Among the works and masterworks on exhibition the visitor will find the best pieces, most importantly The Persistence of Memory. There is also L’Enigme sans Fin from 1938, works on paper, objects, and projects for stage and screen and selected parts from television programmes reflecting the artist’s showman qualities. The visitor will enter the World of Dali through an egg and is met with the beginning, the world of birth. The exhibition follows a path of time and subject with the visitor exiting through the brain. The exhibition shows how Dali draws the viewer between two infinities (无限 ). “From the infinity small to the infinity large, contraction and expansion coming in and out of focus: amazing Flemish accuracy and the showy Baroque of old painting that he used in his museum-theatre in Figueras,” explains the Pompidou Centre. The fine selection of the major works was done in close collaboration (合作 )with the Museo Nacional Reina Sofia in Madrid, Spain, and with contributions from other institutions like the Salvador Dali Museum in St. Petersburg. 28. Which of the following best describe Dali according to Paragraph 1? A. Optimistic. B. Productive C. Generous. D. Traditional. 29. What is Dali’s The Persistence of Memory considered to be? A. One of his masterworks. B. A successful screen adaptation. C. An artistic creation for the stage. D. One of the beat TV programmes. 30. How are the exhibits arranged at the World of Dali? A. By popularity. B. By importance. C. By size and shape. D. By time and subject. 31. What does the word “contributions” in the last paragraph refer to? A. Donations. B. Projects. C. Artworks. D. Documents. D Conflict is on the menu tonight at the café La Chope. This evening, as on every Thursday night, psychologist Maud Lehanne is leading two of France’s favorite pastimes, coffee drinking and the “talking cure”. Here they are learning to get in touch with their true feelings. It isn’t always easy. They customers-some thirty Parisians who pay just under $2 (plus drinks) per session-care quick to intellectualize (高谈阔论 ), slow to open up and connect. “You are forbidden to say ‘one feels,’ or ‘people think’,”Lehane told them. “Say ‘I think,’ ‘Think me’.” A café society where no intellectualizing is allowed? It couldn’t seem more un-French. But Lehanne’s psychology café is about more than knowing oneself: It’s trying to help the city’s troubled neighborhood cafes. Over the years, Parisian cafes have fallen victim to changes in the French lifestyle-longer working hours, a fast food boom and a younger generation’s desire to spend more time at home. Dozens of new theme cafes appear to change the situation. Cafes focused around psychology, history, and engineering are catching on, filling tables well into the evening. 32.What are people encouraged to do at the cafe La Chope? A. Learn a new subject B. Keep in touch with friends. C. Show off their knowledge. D. Express their true feelings. 33. How are cafes affected by French lifestyle changes? A. They have bigger night crowds. B. They stay open for longer hours. C. They are less frequently visited. D. They start to serve fast food. 34. What are theme cafes expected to do? A. Save the cafe business. B. Supply better drinks. C. Create more jobs. D. Serve the neighborhood. 35. Why are psychology cafes becoming popular in Paris? A. They bring people true friendship. B. They give people spiritual support. C. They help people realize their dreams. D. They offer a platform for business links. 第二节 (共 5小题,每小题 2分,满分 10分) 根据短文内容,从短文后的选项中选出能填入空白处的最佳选项,选项中有两项为多余选项。 Building Trust in a Relationship Again Trust is a learned behavior that we gain from past experiences, 36 . That is a risk. But you can’t be successful when there’s a lack of trust in a relationship that results from an action where the wrongdoer takes no responsibility to fix the mistake. Unfortunately, we’ve all been victims of betrayal. Whether we’ve been suffer from, lied to , misled, or cheated on, there are different levels of losing trust. Sometimes people simply can’t trust anymore, 37 . It’s understandable, but if you’re willing to build trust in a relationship again, we have some steps you can take to get you there.  38 having confidence in yourself will help you make better choices because you can see what the best outcome would be for your well-being.  39 If you’ve been betrayed, you are the victim of your circumstance. But there’s a difference between being a victim and living with a “victim mentality”. At some point in all of our lives, we’ll have our trust tested or violated.  You didn’t lose “everything”. Once trust is lost, what is left? Instead of looking at the situation from this hopeless angle, look at everything you still have and be thankful for all of the good in your life. ____40____ instead, it’s a healthy way to work through the experience to allow room for positive growth and forgiveness. A. Learn to really trust yourself. B. It is putting confidence in someone. C. Stop regarding yourself as the victim. D. Remember that you can expect the best in return. E. Seeing the positive side of things doesn’t mean you’re ignoring what happened. F. This knowledge carries over in their attitude toward their future relationships. G. They’ve been too badly hurt and they can’t bear to let it happen again. 第三部分 英语知识运用 (共两节,满分 45) 第一节 完形填空(共 20小题;每小题 15分,满分 30分) 阅读下面短文,从短文后各题所给的四个选项(A、B、C和 D)中,选出可以填入空白处的 最佳选项,并在答题卡上将该项涂黑。 My kids and I were heading into the supermarket over the weekend. On the way ,we spotted a man holding a piece of paper that said, “ 41 my job. Family to Feed.” At this store, a 42 like this is not normal. My 10-year-old noticed him and make a ___43___ on how bad it must be to have to stand 44 in the cold wind. In the store, I asked each of my kids to 45 something they thought our “friend” there would 46 . They got apples, a sandwich and a bottle of juice. Then my 17-year-old suggested giving him a 47 . I thought about it. We were 48 on cash ourselves, but… well, sometimes 49 from our need instead of our abundance is 50 what we need to do! All the kids 51 something they could do away with for the week. When we handed him the bag of 52 , he lit up and thanked us with 53 eyes. When I handed him the gift card, saying he could use it for 54 his family might need, he burst into tears. This has been a wonderful 55 for our family. For days the kids have been looking for others we can 56 ! Things would have played out so 57 if I had simply said, “No, we really don’t have 58 to give more.” Stepping out not only helped a brother in 59 , it also gave my kids the 60 taste of helping others. It’ll go a long way with them. 41. A. Quit B. Changed C. Lost D. Finished 42. A condition B. place C. sight D. show 43. A. suggestion B. comment C. decision D. call 44. A. by B. proudly C. outside D. angrily 45. A. draw B. say C. arrange D. pick 46. A. appreciate B. supply C. order D. discover 47. A. dollar B. job C. hot meal D. gift card 48. A. easy B. low C. soft D. loose 49. A. giving B. saving C. spending D. begging 50. A. yet B. even C. still D. just 51. A. declared B. shared C. ignored D. expected 52. A . food B. medicine C. toys D. clothes 53. A. sleepy B. watery C. curious D. sharp 54. A. whoever B. whatever C. whichever D. whenever 55. A. message B. example C. experience D. adventure 56. A. rely on B. respect C. learn from D. help 57. A. suddenly B. vividly C. differently D. perfectly 58. A. time B. power C. patience D. money 59. A. need B. love C. fear D. memory 60. A. strong B. sweet C. strange D. simple 第 II卷 注意事项: 用 0.5毫米黑色笔迹的签字笔将答案写在答题卡上,写在本试卷无效。 第三部分 英语知识运用 (共两节,满分 45) 第二节 (共 10小题;每小题 1.5分,共 15分) 阅读下面材料,在空白处填入适当内容(1个单词)或括号内单词的正确形式。 Yangshuo, China It was raining lightly when I 61 (arrive) in Yangshuo just before dawn. But I didn’t care. A few hours 62 , I’d been at home in Hong Kong, with 63 (it) choking smog. Here, the air was clean and fresh, even with the rain. I’d skipped nearby Guilin, a dream place for tourists seeking the limestone mountain tops and dark waters of the Li River 64 are pictured by artists in so many Chinese 65 (painting). Instead, I ‘d head straight for Yangshuo. For those who fly to Guilin, it’s only an hour away ___66___ car and offers all the scenery of the better-known city. Yangshuo 67 (be) really beautiful. A study of travelers 68 (conduct) by the website TripAdvisor names Yangshuo as one of the top 10 destinations in the world. And the town is fast becoming a popular weekend destination for people in Asia. Abercrombie & Kent, a travel company in Hong Kong, says it 69 (regular) arranges quick getaways here for people 70 (live) in Shanghai and Hong Kong. 61. __________ 62. __________ 63. __________ 64. __________ 65. __________ 66. __________ 67. __________ 68. __________ 69. __________ 70. __________ 第四部分 写作 (共两节 满分 35) 第一节 短文改错(共 10小题;每小题 1分,共 10分) 假定英语课上老师要求同桌之间交换修改作文,请你修改你同桌写的以下作文。作文 中共有 10 处语言错误,每句中最多有两处。每处错误仅涉及一个单词的增加、删除 或修改。 增加:在缺词处加一个漏子符号(∧)并在其下面写出该加的词。 删除:在错的词下划一横线,并在该词下面写出修改后的词。 注意:1.每处错误及其修改均仅限一词; 2.只允许修改 10 处,多者(从第 11 处起)不计分。 When I was a child, I hoped to live in the city. I think I would be happy there. Now I am living in a city ,but I miss my home in countryside. There the air is clean or the mountains are green. Unfortunately, on the development of industrialization, the environment has been polluted. Lots of studies have been shown that global warming has already become a very seriously problem. The airs we breathe in is getting dirtier and dirtier. Much rare animals are dying out . We must found ways to protect your environment. If we fail to do so , we’ll live to regret it . 第二节 书面表达(满分 25分) 假定你是李华,你校英文报“外国文化”栏目拟刊登美国节日风俗和中学生生活的短文。请给 美国朋友彼得写信约稿,要点如下: 1. 栏目介绍。 2. 稿件内容; 3. 稿件长度:约 400词汇 4. 交稿日期:6月 28日前天 注意: 1. 词数 100左右; 2. 可以适当增加细节,以使行文连贯的; 3. 开头语已为你写好。 2015年普通高等学校招生全国统一考试(新课标 II) 英 语 本试卷分第Ⅰ卷(选择题)和第Ⅱ卷(非选择题)两部分。考试结束后,将本试卷和答题卡 一并交回。 第Ⅰ卷 注意事项: 1. 答第Ⅰ卷前,考生务必将自己的姓名、准考证号填写在答题卡上。 2. 选出每小题答案后,用铅笔把答题卡上对应题目的答案标号涂黑。如需改动,用橡皮擦 干净后,再选涂其他答案标号。不能答在本试卷上,否则无效。 第一部分 听力(共两节,满分 30 分) 做题时,现将答案标在试卷上,录音内容结束后,你将有两分钟的时间将试卷上的答案转涂 到答题卡上。 第一节(共 5小题;每小题 1.5分,满分 7.5分) 听下面 5段对话,每段对话后有一个小题。从题中所给的 A,B,C三个选项中选出最佳选项, 并标在试卷的相应位置。听完每段对话后,你都有 10秒钟的时间来回答有关小题和阅读下一小题。 每段对话仅读一遍。 例: How much is the shirt? A. £ 19.15 B. £ 9.18 C. £ 9.15 答案是 C。 1. 1. What time is it now? A. 9:10 B. 9:50 C. 10:00 2. What does the woman think of the weather? It’s nice. It’s warm It’s cold. 3. What will the man do? A. Attend a meeting. B. Give a lecture C. Leave his office. 4. What is the woman’s opinion about the course? A. Too hard B. Worth taking. C. Very easy. 5. What does the woman want the man to do? A. Speak louder B. Apologize to her. C. Turn off the radio. 第二节(共 15小题;每小题 1.5分,满分 22.5分) 听下面 5短话或独白,没段话或独白后有几个小题,从题中所给的 A、B、C三个选项中选出 最佳选项,并标在试卷的相应位置。听每段对话或独白,你将有时间阅读各个小题,每小题 5秒 钟;听完后,各小题将给出 5秒钟的做大时间,每段对话或独白读两遍。 听第 6段材料,回答第 6、7题。 6. How long did Michael stay in China? A. Five days. B. One week. C. Two weeks. 7. Where did Michael go last year? A. Russia B. Norway C. India 听第 7段材料,回答第 8、9题。 8. What food does Sally like? A. Chicken. B. Fish. C. Eggs. 9. What are the speakers going to do? A. Cook dinner. C. Go shopping. C. Order dishes. 听第 8段材料,回答第 10至 12题。 10. Where are the speakers? A. In a hospital. B. In the office. C. At home. 11. When is the report due? A. Thursday. B. Friday. C. Next Monday. 12. What does George suggest Stephanie do with the report? A. Improve it. B. Hand it in later. C. Leave it with him. 听第 9段材料,回答第 13至 16题。 13. What is the probable relationship between the speakers? A. Salesperson and customer. B. Homeowner and cleaner. C. Husband and wife. 14. What kind of apartment do the speakers prefer? A. One with two bedroom. B. One without furniture. C. One near a market. 15. How much rent should one pay for the one-bedroom apartment? A. $350. B. $400. C. $415. 16. Where is the apartment the speakers would like to see? A. On Lake Street B. On Market Street. C. On South Street. 听第 8段材料,回答第 10至 12题。 17. What percentage of the world’s tea exports go to Britain? A. About 15%. B. About 30%. C. Over 40%. 18. Why do tea tasters taste tea with milk? A. Most British people drink that way. B. Tea tastes much better with milk. C. Tea with milk is healthy. 19. Who suggests a price for each tea? A. Tea tasters. B. Tea exporters. C. Tea companies. 20. What is the speaker talking about? A. The life of tea tasters. B. Afternoon tea in Britain. C. The London Tea Trade Centre. 第二部分 阅读理解(共两节,满分 40 分) 第一节(共 15小题:每小题 2分,满分 30分) 阅读下列短文,从每题所给的四个选项(A、B、C和 D)中,选出最佳选项,并在答题卡 上将该项涂黑。 A My color television has given me nothing but a headache.I was able to buy it a little over a year ago because I had my relatives give me money for my birthday instead of a lot of clothes that wouldn’t fit.I let a salesclerk fool me into buying a discontinued model,I realized this a day late,when I saw newspaper advertisements for the set at seventy-five dollars less than I had paid,The set worked so beautifully when I first got it home that I would keep it on until stations signed off for the night,Fortunately, I didn’t got any channels showing all-night movies or I would never have gotten to bed. Then I started developing a problem with the set that involved static (静电 ) noise. For some reason,when certain shows switched into a commercial, a loud noise would sound for a few seconds. Gradually,this noise began to appear during a show, and to get rid of it,I had to change to another channel and then change it back.Sometimes this technique would not work,and I had to pick up the set and shake it to remove the sound. I actually began to build up my arm muscles(肌肉) shaking my set. When neither of these methods removed the static noise ,I would sit helplessly and wait for the noise to go away.At last I ended up hitting the set with my fist,and it stopped working altogether .My trip to the repair shop cost me $62, and the set is working well now,but I keep expecting more trouble. 21. Why did the author say he was fooled into buying the TV set? A. He got an older model than he had expected. B. He couldn’t return it when it was broken. C. He could have bought it at a lower price. D. He failed to find any movie shows on it. 22. Which of the following can best replace the phrase”signed off”in Paragraph 1? A. ended all their programs B. provided fewer channels C. changed to commercials D. showed all-night movies 23. How did the author finally get his TV set working again? A. By shaking and hitting it. B. By turning it on and off. C. By switching channels. D. By having it repaired. 24. How does the author sound when telling the story ? A. Curious B. Anxious C. Cautious D. Humorous B Your house may have an effect on your figure . Experts say the way you design your home could play a role in whether you pack on the pounds or keep them off . You can make your environment work for you instead of against you . Here are some ways to turn your home into part of diet plan. Open the curtains and turn up the lights . Dark environments are more likely to encourage overeating , for people are often less self-conscious(难为情)when they’re in poorly lit places-and so more likely to eat lots of food . If your home doesn’t have enough window light , get more lamps and flood the place with brightness. Mind the colors . Research suggests warm colors fuel our appetites . In one study , people who ate meals in a blue room consumed 33 percent less than those in a yellow or red room . Warm colors like yellow make food appear more appetizing , while cold colors make us feel less hungry . So when it’s time to repaint , go blue. Don’t forget the clock-or the radio. People who eat slowly tend to consume about 70 fewer calories(卡路里) per meal than those who rush through their meals. Begin keeping track of the time, and try to make dinner last at least 30 minutes, And while you’re at it, actually sit down to eat. If you need some help slowing down , turn on relaxing music. It makes you less likely to rush through a meal. Downsize the dishes, Big serving bowls and plates can easily make us fat. We eat about 22 percent more when using a 12-inch plate instead of a 10-inch plate. When we choose a large spoon over a smaller one, total intake(摄入)jumps by 14 percent. And we’ll pour about 30 percent more liquid into a short, wide glass than a tall, skinny glass. 25. The text is especially helpful for those who care about_______. A. their home comforts B. their body shape C. house buying D. healthy diets 26. A home environment in blue can help people_________. A. digest food better B. reduce food intake C. burn more calories D. regain their appetites 27. What are people advised to do at mealtimes? A. Eat quickly. B. Play fast music. C. Use smaller spoons. D. Turn down the lights. 28. What can be a suitable title for the test? A. Is Your House Making You Fat? B. Ways of Serving Dinner C. Effects of Self-Consciousness D. Is Your Home Environment Relaxing? C More student than ever before are taking a gap-year (间隔年)before going to university.It used to be called the “year off” between school and university.The gap-year phenomenon originated(起源 ) with the months left over to Oxbridge applicants between entrance exams in November and the start of the next academic year. This year, 25,310 students who have accepted places in higher education institutions have put off their entry until next year, according to statistics on university entrance provided by University and College Admissions Serbice(UCAS). That is a record 14.7% increase in the number of students taking a gap year. Tony Higgins from UCAS said that the statistics are good news for everyone in higher education. “Students who take a well-planned year out are more likely to be satisfied with, and complete, their chosen course. Students who take a gap year are often more mature and responsible,” he said. But not everyone is happy. Owain James, the president of the National Union of Students(NUS), argued that the increase is evidence of student had ship – young people are being forced into earning money before finishing their education. “New students are now aware that they are likely to leave university up to £15,000 in debt. It is not surprising that more and more students are taking a gap year to earn money to support their study for the degree.NUS statistics show that over 40% of students are forced to work during term time and the figure increases to 90% during vacation periods,” he said. 29. What do we learn about the gap year from the text? A. It is flexible in length. B. It is a time for relaxation. C. It is increasingly popular. D. It is required by universities. 30. According to Tony Higgins,students taking a gap year______. A. arc better prepared for college studies B. know a lot more about their future jobs C. are more likely to leave university in debt D. have a better chance to enter top universities 31. How does Owain James feel about the gap-year phenomenon? A. He's puzzled. B. He's worried. C. He's surprised. D. He's annoyed. 32. What would most students do on their vacation according to NUS statistics? A. Attend additional courses. B. Make plans for the new term. C. Earn money for their education. D. Prepare for their graduate studies. D Choose Your One-Day Tours Tour A-Bath & Stonchenge including entrance fees to the ancient Roman bathrooms and Stonehenge-£until 26 March and £39 thereafter. Visit the city with over 2,000 years of history and Bath Abbey,the Royal Crescent and the Costume Museum. Stonehenge is one of the world’s most famous prehistoric monuments dating back over 5,000 years. Tour B-Oxford & Stratford including entrance fees to the University St Mary’s Church Tower and Anne Hathaway's house一 32 until 12 March and 36 thereafter. Oxford: Includes a guided of England’s oldest university city and colleges. Look over the “city of dreaming spires(尖顶 )”form St Mary’s Church Tower. Stratford: Includes a guided tour exploring much of the Shakespeare wonder. Tour C—Windsor Castle & Hampton Court including entrance fees to Hampton Court Palace--£34 until March and £37 thereafter. Includes a guided tour of Windsor and Hampton Court, Henry Mill’s favourite palace. Free time to visit Windsor Castle(entrance fees not included). With 500 years of history, Hampton Court was once the home of four Kings and one Queen. Now this former royal palace is open to the public as a major tourist attraction. Visit the palace and its various historic gardens, which include the famous maze(迷 宫)where it is easy to get lost! Tour D-Cambridge including entrance fees to the Tower of Saint Mary the Great-£33 .until 18 March and £37 thereafter. Includes a guided tour of Cambridge, the famous university town, and the gardens of the 18th century. 33.Which tour will you choose if you want to see England’s oldest university city? A. Tour A B. Tour B C. Tour C D. Tour D 34. Which of the following tours charges the lowest fee on 17 March? A. Windsor Castle & Hampton Court. B. Oxford & Stratford C. Bath &Stonehenge. D. Cambridge. 35. Why is Hampton Court a major tourist attraction? A. It used to be the home of royal families. B. It used to be a well-known maze C. It is the oldest palace in Britain D. It is a world-famous castle. 第二节(共 5小题:每小题 2分,满分 10分) 根据短文内容,从短文后的选项中选出能填入空白处的最佳选项。选项中有两项为多余选项。 Training for a marathon requires careful preparation and steady, gradual increases in the length of the runs. 36 , buy the best-fitting, best-built running shoes you can find. No one can say which brand will work best for you or feel best on your feet,so you have to rely on your experience and on the feel of each pair as you shop. When you have found shoes that seem right, walk in them for a few days to double-check the fit. 37 . As always, you should stretch(伸展) at least ten minutes before each run to prevent injuries. During the first week,do not think about distance,but run five minutes longer each day. ___38___, it is wise to take a day off to rest.But during the next week,set a goal of at least a mile and a half per run. 39 . After two weeks,start timing yourself. 40 . Depending on the kind of race you plan to enter,you can set up a timetable for the remaining weeks before the race. A. After six days B. For a good marathon runner C. Before you begin your training D. With each day,increase the distance by a half mile E. If they still feel good,you can begin running in them F. Time spent for preparation raises the quality of training G. Now you are ready to figure out a goal of improving distance and time 第三部分 英语知识运用(共两节,满分 45 分) 第一节 完形填空(共 20小题;每小题 1.5分,满分 30分) 阅读下面短文,从短文后各题所给四个选项(A、B、C和 D)中,选出可以填入空白处的最佳选 项,并在答题卡上将该项涂黑。 Where do you go when you want to learn something?A friend?A tutor? These are all ___41___ aces of learning.But it may well be that the learning you really 42 want somewhere else instead. I had the 43 of seeing this first hand on a 44 . My daughter plays on a recreational soccer team. They did very well this season and so ___45___ a tournament, which normally was only for more skilled club teams. This led to some ___46___experiences on Saturday as they played against teams 47 trained. Through the first two games, her 48 did not get on serious shot on goal. As apparent, I 49 seeing my daughter playing her best, 50 still defeated. IT seemed that something clicked with the 51 between Saturday and Sunday. When they 52 for their Sunday game, they were 53 different. They had begun integrate (融合) the kinds of play and teamwork they had 54 the day before into their 55 . They played aggressively and 56 scored a goal. It 57 me that playing against the other team was a great 58 moment for all the girls on the team. I think it is a general principle. 59 is the best teacher. The lessons they may not be 60 what they would have gotten in school. But are certainly more personal and meaningful, because they had to work them out on their own. 41. A. public B. traditional C. official D. special 42. A. passes B. works C. lies D. ends 43. A. dream B. idea C. habit D. chance 44. A. trip B. holiday C. weekend D. square 45. A. won B. entered C. organized D. watched 46. A. painful B. strange C. common D. practical 47. A. less B. poorly C. newly D. better 48. A. fans B. tutors C. class D. team 49. A. imagined B. hated C. avoided D. missed 50. A.if B. or C. but D. as 51. A. girls B. parents C. coaches D. viewers 52. A. dressed B. showed up C. made up D. planned 53. A. slightly B. hardly C. basically D. completely 54. A. seen B. known C. heard D. read 55. A. styles B. training C. game D. rules 56. A. even B. still C. seldom D. again 57. A. confused B. struck C. reminded D. warned 58. A. touching B. thinking C. encouraging D. learning 59. A. Experience B. Independence C. Curiosity D. Interest 60. A. harmful to B. mixed with C. different from D. applied to 第 II卷 注意:将答案写在答题卡上,写在本试卷上无效。 第三部分 英语知识运用(共两节,满分 45 分) 第二节 (共 10小题:每小题 1.5分,满分 15分) 阅读下面材料,在空白处填入适当的内容(1个单词)或括号内单词的正确形式。 The adobe dwellings(土坯房) 61 (build) by the Pueblo Indians of the American Southwest are admired by even 62 most modern of architects and engineers. In addition to their simple beauty, what makes the adobe dwellings admirable is their 63 (able) to “air condition”a house without 64 (use) electric equipment.Walls made of adobe take in the heat from the sun on hot days and give out that heat 65 (slow) during cool nights,thus warning the house. When a new day breaks, the walls have given up their heat and are now cold enough 66 (cool) the house during the hot day: 67 the same time, they warm up again for the night This cycle 68 (go) day after day: The walls warm up during the day and cool off during the night and thus always a timely offset (抵消) for the outside temperatures. As 69 (nature) architects, the Pueblo Indians figured out exactly 70 thick the adobe walls needed to be to make the cycle work on most days. 61. __________ 62. __________ 63. __________ 64. __________ 65. __________ 66. __________ 67. __________ 68. __________ 69. __________ 70. __________ 第四部分 写作(共两节,满分 35 分) 第一节 短文改错(共 10小题,每小题 1分,满分 10分) 假定英语课上老师要求同桌之间交换修改作文,请你修改你同桌写的以下作文。作文中共有 10处语言错误,每句中最多有两处。每处错误仅涉及一个单词的增加、删除或修改。 增加:在缺词处加一个漏子符号(∧)并在其下面写出该加的词。 删除:在错的词下划一横线,并在该词下面写出修改后的词。 注意:1.每处错误及其修改均仅限一词; 2.只允许修改 10处,多者(从第 11处起)不计分。 One day, little Tony went to a shopping center with his parent.It was very crowded. Tony saw a toy on a shop window. He liked it so very much that he quickly walked into the shop. After looks at the toy for some time, he turned around and found where his parents were missing. Tony was scared and begun to cry. A woman saw him crying and telling him to wait outside a shop. Five minutes later. Tony saw parents. Mom said,” How nice to see you again! Dad and I were terrible worried.” Tony promised her that this would never happen again. 第二节 书面表达(满分 25分) 假如你是李华,计划和同学去敬老院(nursing home)陪老人们过重阳节(the Double Ninth Festival)。 请给外教露西写封邮件,邀她一同前往,内容包括: 1.出发及返回时间; 2.活动:包饺子、表演节目等。 注意: 1.词数 100左右; 2.可以适当增加细节,以使行文连贯; 3.结语已为你写好。 2015 年普通高等学校招生全国统一考试 英 语(北京卷) 本试卷共 150分。考试时间为 120分钟。考生务必将答案答在答题卡上,在试卷上作答无效。 考试结束后,将本试卷和答题卡一并交回。 第一部分:听力理解(共三节,30 分) 第一节(共 5 小题;每小题 1.5 分,共 7.5 分) 听下面 5段对话。每段对话后有一道小题,从每题所给的 A、B、C三个选项中选出最佳选项。听 完每段对话后,你将有 10秒钟的时间来回答有关小题和阅读下一小题,每段对话你将听一遍。 例:What is the man going to read? A.A newspaper B.A magazine C.A book 答案是 A 1. What kind of music does the woman like? A.Classical music B. Rock music C. Country music 2. Which sweater will the man take? A. The red one B. The blue one C. The yellow one 3. How will the man pay? A. In cash B. By credit card C. By traveler’s cheque 4. Where are the woman’s keys? A. In her pocket B. On the floor C. On the table 5. What will the man probably do tonight? A. Read Chapter 4 B. Study in the library C. Watch the football match 第二节(共 10小题;每小题 1.5分,共 15分) 听下面 4段对话或独白。每段对话或独白后有几道小题,从每题所给的 A、B、C三个选项 中选出最佳选项。听每段对话或独白前,你将有 5秒钟的时间阅读每小题。听完后,每小题将给 出 5秒钟的作答时间。每段对话或独白你将听两遍。 听第 6段材料,回答第 6至 7题。 6. What is the relationship between the two speakers? A. Travel agent and customer B. Husband and wife C. Boss and secretary 7. When will the meetings end? A. At3:00pm. B. At 5:00pm. C. At 6:30pm. 听第 7段材料,回答第 8至 9题。 8. Where is the foreign language section? A. In Row 3. B. In Row 5. C. In Row 7 9. What does the man decide to borrow? A. Novels. B. Biographies. C. Magazines 听第 8段材料,回答第 10至 12题。 10. Why dose the man make the call? A. To ask for information B. To make an appointment C. To send out an invitation 11. What sport is the man interested in? A. Football B. Basketball C. Swimming 12. On what days is the sports center closed? A. Mondays B. Fridays C. Sundays 听第 9段材料,回答第 13至 15题。 13. What is the speaker mainly talking about? A. Rules of the school B. Course requirements C. Notices of the new term. 14. Which club will meet on Wednesdays? A. Baseball B. Dance C. Chess 15. When will the parents’ evening for Year 8 be held? A. In September B. In October C. In November 第三节(共 5小题;每小题 1.5分,共 7.5分) 听下面一段对话,完成第 16 至第 20 五道小题,每小题仅填写一个..词。听对话前,你将有 20秒钟的时间阅读试题,听完后你将有 60秒钟的作答时间。这段对话你将听两遍。 Cell Phone Repair Form Customer’s Name Thomas 16 Telephone No. 17 Time of Purchase 18 1st,2015 Problem Screen went 19 Solution(解决方案) 20 it up and check the inside 第二部分:知识运用(共两节,45 分) 第一节 单项填空(共 15 小题;每小题 1 分,共 15 分) 从每题所给的 A、B、C、D 四个选项中,选出可以填入空白处的最佳选项,并在答题卡上 将该项涂黑。 例:It’s so nice to hear from her again. __________, we last met more than thirty years ago. A. What’s more B. That’s to say C. In other words D. Believe it or not 答案是 D。 21. __________ the early flight,we ordered a taxi in advance and got up very early. A. Catching B. Caught C. To catch D. Catch 22. —Did you enjoy the party? —Yes.We __________ well by our hosts. A. were treated B. would be treated C. treated D. had treated 23. The park was full of people __________ themselves in the sunshine. A. having enjoyed B. enjoyed C. enjoying D. to enjoy 24. Opposite is St.Paul’s Church, __________ you can hear some lovely music. A. which B. that C. when D. where 25. He is a shy man, __________ he is not afraid of anything or anyone. A. so B. but C. or D. as 26. In the last few years,China __________ great achievements in environmental protection. A. has made B. had made C. was making D. is making 27. —Did you have difficulty finding Ann’s house? —Not really.She __________ us clear directions and we were able to find it easily. A. was to give B. had given C. was giving D. would give 28. You won’t find paper cutting difficult __________ you keep practicing it. A. even though B. as long as C .as if D. ever since 29. —Can’t you stay a little longer? —It’s getting late. I really __________ go now, My daughter is home alone. A. may B. can C. must D. dare 30. —Dr. Jackson is not in his office at the moment. —All right. I __________ him later. A. will call B. have called C. call D. will be calling 31. If __________ for the job, you’ll be informed soon. A. to accept B. accept C. accepting D. accepted 32. __________ the damage is done, it will take many years for the farmland to recover. A. Until B. Unless C. Once D. Although 33. I truly believe __________ beauty comes from within. A. that B. where C. what D. why 34. If I __________ it with my own eyes , I wouldn’t have believed it. A. didn’t see B. weren’t seeing C. Wouldn’t see D. hadn’t seen 35. __________ we understand things has a lot to do with what we feel. A. Where B. How C. Why D. When 第二节 完形填空(共 20 小题;每小题 1.5 分,共 30 分) 阅读下面短文, 掌握其大意,从每题所给的 A、B、C、D 四个选项中,选出最佳选项, 并在答题卡上讲该项 涂黑。 AWelcome Gift Dario and his mother loved their new apartment. The living room was large enough for their piano. That night, the two of them 36 side by side at the piano. They played jazz music to celebrate their new home. The loud 37 filled the room and made them feel very happy. The next morning, 38 , their happiness disappeared. Someone had left a 39 under their door during the night. One of their neighbor had written to complain(抱怨) about the sound of the piano. Dario’s mother asked the building superintendent(管理员 ) if he knew anything about it. But he said that they were all 40 people and he couldn’t imagine any of them had done that. Later that morning, Dario suggested that they write a letter to their 41 and apologize for their playing. “Maybe we could go and 42 everyone in person.” his mother said. “What if we invited them to come here for a 43 instead? Dario asked. They both loved the 44 . Over the next few days, they sent out invitations and prepared desserts 45 their guests. They decorated the apartment with streamers(彩带) and party lights. Finally, the day of the party 46 . Some guests brought presents. Others brought flowers. Some even brought desserts to 47 . One woman, Mrs. Gilbert, 48 Dario’s mother with a book of piano music by Chopin. “I heard you playing the other night,” she said. “The sounds woke me out of bed. I ___49___ that you might play like this every night. So I wrote a short note. I hope you don’t think I disliked the playing.” Dario’s mother smiled at Mrs. Gilbert. “I think maybe we 50 you an apology.” she said. “I didn’t 51 how late it was when we were playing. Maybe we should play some quieter music at night. “You play, you play!” Mrs. Gilbert said. “I like what you play! Just not so loud at night.” She pointed to the book she had given them. “These songs are not such 52 music.” “These songs are beautiful music.” Dario’s mother said. “We will be 53 to play them in the evening.” “And we won’t play so loud or late!” Dario said. He was already looking forward to ___54___ the new music. More than that, however, he was happy to see the big smile on his mother’s face. It gave him a feeling of 55 and made him feel that they were home at last. 36. A. sat B. stood C. lay D. walked 37. A. voice B. ring C. music D. cry 38. A. therefore B. however C. otherwise D. instead 39. A. note B. poster C. bill D. report 40. A. proud B. rich C. lucky D. nice 41. A. neighbors B. friends C. relatives D. audience 42. A. blame B. instruct C. question D. visit 43. A. party B. concert C. show D. play 44. A. experience B. idea C. performance D. action 45. A. to B. with C. for D. from 46. A. continued B. arrived C. passed D. finished 47. A. order B. sell C. share D. advertise 48. A. treated B. presented C. helped D. served 49. A. promised B. admitted C. agreed D. worried 50. A. give B. send C. offer D. owe 51. A. realize B. remember C. understand D. accept 52. A. sweet B. strange C. funny D. loud 53. A. brave B. sorry C. happy D. afraid 54. A. changing B. practicing C. recording D. writing 55. A. equality B. freedom C. warmth D. sympathy 第三部分:阅读理解 (共两节,共 20 分) 第一节(共 15 小题;每小题 2 分,共 30 分) 阅读下列短文:从每题所给的 A、B、C、D 四个选项中,选出最佳选项,将正确的选项涂 在答题卡上。 A The Boy Made It! One Sunday, Nicholas, a teenager, went skiing at Sugarloaf Mountain in Maine. In the early afternoon, when he was planning to go home, a fierce snowstorm swept into the area. Unable to see far, he accidentally turned off the path. Before he knew it, Nicholas was lost, all alone! He didn’t have food, water, a phone, or other supplies. He was getting colder by the minute. Nicholas had no idea where he was. He tried not to panic. He thought about all the survival shows he had watched on TV. It was time to put the tips he had learned to use. He decided to stop skiing. There was a better chance of someone finding him if he stayed put. The first thing he did was to find shelter form the freezing wind and snow. If he didn’t, his body temperature would get very low, which could quickly kill him.Using his skis, Nicholas built a snow cave. He gathered a huge mass of snow and dug out a hole in the middle. Then he piled branches on top of himself, like a blanket, to stay as warm as he could. By that evening, Nicholas was really hungry. He ate snow and drank water from a nearby stream so that his body wouldn’t lose too much water. Not knowing how much longer he could last, Nicholas did the only thing he could — he huddled (蜷缩) in his cave and slept. The next day, Nicholas went out to look for help, but he couldn’t find anyone. He followed his tracks and returned to the snow cave,because without shelter, he could die that night. On Tuesday,Nicholas went out again to find help. He had walked for about a mile when a volunteer searcher found him. After two days stuck in the snow, Nicholas was saved. Nicholas might not have survived this snowstorm had it not been for TV. He had often watched Grylls’ survival show Man vs. Wild.That’s where he learned the tips that saved his life. In each episode(一期节目 ) of Man vs. Wild, Grylls is abandoned in a wild area and has to find his way out.When Grylls heard about Nicholas’ amazing deeds, he was super impressed that Nicholas had made it since he knew better than anyone how hard Nicholas had to work to stay alive. 56. What happened to Nicholas one Sunday afternoon? A. He got lost. B. He broke his skis. C. He hurt his eyes. D. He caught a cold. 57. How did Nicholas keep himself warm? A. He found a shelter. B. He lighted some branches. C. He kept on skiing. D. He built a snow cave. 58. On Tuesday, Nicholas __________. . A. returned to his shelter safely. B. was saved by a searcher. C. got stuck in the snow D. stayed where he was 59. Nicholas left Grylls a very deep impression because he __________. A. did the right things in the dangerous situation. B. watched Grylls’ TV program regularly C. created some tips for survival D. was very hard-working B Revolutionary TV Ears TV Ears has helped thousands of people with various degrees of hearing loss hear the television clearly without turning up the volume(音量) and now it’s better and more affordable than ever! With TV Ears wireless technology, you set your own headset volume, while other TV listeners hear the television at a volume level that’s comfortable for them. You can even listen through the headset only and put the TV on mute(静音 ) if the situation calls for a quiet environment —maybe the baby is sleeping. Or perhaps you are the only one who is interested in listening to the ballgame. TV Ears patented technology includes a revolutionary noise reduction car tip, not used in any other commercially available headset. This tip reduces outside noise so that television dialogue is clear and understandable. Get the technology that has proven to help the most demanding customers. That’s why TV Ears has earned the trust and confidence of audiologists(听觉学家) nationwide as well as world-famous doctors. Doctor Recommended TV Ears! “My wife and I have used TV Ears almost daily for the past two years and find them a great help in our enjoyment of television . As a retired ear doctor, I heartily recommend TV Ears to people with normal hearing as well as those with hearing loss.” — Robert Forbes, M. D, CA Customer Recommended TV Ears! “ Now my husband can have the volume as loud as he needs and I can have the TV at my hearing level. TV Ears is so comfortable that Jack forgets he has them on ! He can once again hear and understand the dialogue.” — Darlene & Jack B, CA Risk Free Trial! TV Ears comes with a 30-day risk free trial. Special Offer — Now $59.95. If you’re not satisfied, return it. Money-back guarantee! Call now ! 800-123-7832 60. TV Ears helps you __________. A. improve your sleeping quality B. listen to TV without disturbing others C. change TV channels without difficulty D. become interested in ballgame programs 61. What makes TV Ears different from other headsets? A. It can easily set TV on mute B. Its headset volume is adjustable C. It has a new noise reduction ear tip D. It applies special wireless technology 62. This advertisement is made more believable by ______. A. using recommendations B. offering reasons for this invention C. providing statistics D. showing the results of experiments C Life in the Clear Transparent animals let light pass through their bodies the same way light passes through a window. These animals typically live between the surface of the ocean and a depth of about 3,300 feet—as far as most light can reach. Most of them are extremely delicate and can be damaged by a simple touch. Sonke Johnsen, a scientist in biology, says, “These animals live through their life alone. They never touch anything unless they’re eating it, or unless something is eating them.” And they are as clear as glass. How does an animal become see-through? It’s trickier than you might think. The objects around you are visible because they interact with light. Light typically travels in a straight line. But some materials slow and scatter(散射) light, bouncing it away from its original path. Others absorb light, stopping it dead in its tracks. Both scattering and absorption make an object look different from other objects around it, so you can see it easily. But a transparent object doesn’t absorb or scatter light, at least not very much, Light can pass through it without bending or stopping. That means a transparent object doesn’t look very different from the surrounding air or water. You don’t see it ----you see the things behind it. To become transparent, an animal needs to keep its body from absorbing or scattering light. Living materials can stop light because they contain pigments(色素) that absorb specific colors of light. But a transparent animal doesn’t have pigments, so its tissues won’t absorb light. According to Johnsen, avoiding absorption is actually easy. The real challenge is preventing light from scattering. Animals are built of many different materials----skin, fat, and more----and light moves through each at a different speed. Every time light moves into a material with a new speed, it bends and scatters. Transparent animals use different tricks to fight scattering. Some animals are simply very small or extremely flat. Without much tissue to scatter light, it is easier to be see—through. Others build a large, clear mass of non-living jelly-lie(果冻状的) material and spread themselves over it . Larger transparent animals have the biggest challenge, because they have to make all the different tissues in their bodies slow down light exactly as much as water does. They need to look uniform. But how they’re doing it is still unknown. One thing is clear for these larger animals, staying transparent is an active process. When they die, they turn a non-transparent milky white. 63. According to Paragraph 1, transparent animals __________. A. stay in groups B. can be easily damaged C. appear only in deep ocean D. are beautiful creatures 64. The underlined word “dead” in Paragraph 3 means __________. A. silently B. gradually C. regularly D. completely 65. One way for an animal to become transparent is to __________. A. change the direction of light travel B. gather materials to scatter light. C. avoid the absorption of light D. grow bigger to stop light. 66. The last paragraph tells us that larger transparent animals __________. A. move more slowly in deep water B. stay see-through even after death C. produce more tissues for their survival D. take effective action to reduce light spreading D Technological change is everywhere and affects every aspect of life, mostly for the better. However, social changes are brought about by new technology are often mistaken for a change in attitudes. An example at hand is the involvement of parents in the lives of their children who are attending college. Surveys (调查) on this topic suggests that parents today continue to be “very” or “somewhat” overly-protective even after their children move into college dormitories. The same surveys also indicate that the rate of parental involvement is greater today than it was a generation ago. This usually interpreted as a sign that today’s parents are trying to manage their children’s lives past the point where this behavior is appropriate. However, greater parental involvement does not necessarily indicate that parents are failing to let go of their “adult” children. In the context (背景) of this discussion, it seems valuable to first find out the cause of change in the case of parents’ involvement with their grown children. If parents of earlier generations had wanted to be in touch with their college-age children frequently, would this have been possible? Probably not. On the other hand, does the possibility of frequent communication today mean that the urge to do so wasn’t present a generation ago? Many studies show that older parents—today’s grandparents—would have called their children more often if the means and cost of doing so had not been a barrier. Furthermore, studies show that finances are the most frequent subject of communication between parents and their college children. The fact that college students are financially dependent on their parents is nothing new; nor are requests for more money to be sent from home. This phenomenon is neither good nor bad; it is a fact of college life, today and in the past. Thanks to the advanced technology, we live in an age of bettered communication. This has many implications well beyond the role that parents seem to play in the lives of their children who have left for college. But it is useful to bear in mind that all such changes come from the technology and not some imagined desire by parents to keep their children under their wings. 67. The surveys inform us of __________. A. the development of technology B. the changes of adult children’s behavior C. the parents’ over-protection of their college children D. the means and expenses of students’ communication 68. The writer believes that__________. A. parents today are more protective than those in the past B. the disadvantages of new technology outweigh its advantages C. technology explains greater involvement with their children D. parents’ changed attitudes lead to college children’s delayed independence 69. What is the best title for the passage? A. Technology or Attitude B. Dependence or Independence C. Family Influences or Social Changes D. College Management or Communication Advancement 70. Which of the following shows the development of ideas in this passage? 第二节(共 5 小题;每小题 2 分。共 10 分) 根据短文内容,从短文后的七个选项中选出正确 的填入空白处。选项中有两项为多余选项。 This Way to Dreamland Daydreaming means people think about something pleasant, especially when this makes them forget what they should be doing. Daydreamers have a bad reputation for being unaware of what’s happening around them. They can seem forgetful and clumsy. 71 They annoy us because they seem to be ignoring us and missing the important things. But daydreamers are also responsible for some of the greatest ideas and achievements in human history. 72 Can you imagine what kind of world we would have without such ideas and inventions? So how can you come up with brilliant daydreams and avoid falling over tree roots or otherwise looking like a fool? First, understand that some opportunities(机会) for daydreaming are better than others. Feeling safe and relaxed will help you to slip into daydreams. 73 And if you want to improve your chances of having a creative idea while you’re daydreaming, try to do it while you are involved in another task—preferably something simple, like taking a shower or walking, or even making meaningless drawings. It’s also important to know how to avoid daydreams for those times when you really need to concentrate. “Mindfulness”, being focused, is a tool that some people use to avoid falling asleep. ___74___ Finally, you never know what wonderful idea might strike while your mind has moved slowly away. 75 Always remember that your best ideas might come when your head is actually in the clouds. A. Having interesting things to think about also helps. B. They stare off into space and wander by themselves. C. Without wandering minds, we wouldn’t have relatively, Coke or Post-it notes. D. At one time, daydreaming was thought to be a cause of some mental illnesses. E. It involves slow, steady breathing for self-control that helps people stay calm and attentive. F. Daydreams are often very simple and direct, quite unlike sleep dreams, which may be hard to understand. G. Therefore, it’s a good idea to keep a notebook or voice recorder nearby when you’re in the daydream zone. 第四部分:书面表达(共两节,35 分) 第一节(15分) 假如你是红星中学高三李华,请给你的美国朋友 Jim写一封信,告诉他你打算参加龙舟训 练营,希望他一起参加,邮件的内容包括: 1. 介绍训练营的相关内容(例如;时间,地点,参加者等); 2. 说明你打算参加的原因; 3. 询问对方的意向。 注意: 1. 词数不少于 50; 2. 开头和结尾已给出, 不计入总词数。 提示词:龙舟训练营 Dragon Boat Training Camp Dear Jim, Yours, Li Hua 第二节 ( 20 分) 假设你是红星中学高三学生李华。请根据以下四幅图的先后顺序,介绍在“传统文化进校园”活动 中,向面人艺术家学习捏面人的过程,并以“A Day with a Craftsman”为题,给校刊“英语角”写一篇 英文稿件。 注意: 词数不少于 60。 提示词: 一个面团 a piece of dough, 面人 dough figurine ____________________________________________________________________________________ ____________________________________________________________________________________ ____________________________________________________________________________________ ____________________________________________________________________________________ ____________________________________________________________________________________ ____________________________________________________________________________________ ____________________________________________________________________________________ ____________________________________________________________________________________ ____________________________________________________________________________________ (请务必将作文写在答题卡指定区域内) 2015年普通高等学校招生全国统一考试(天津卷) 英语 笔试 本试卷分第Ⅰ卷(选择题)和第Ⅱ卷(非选择题)两部分,共 130分,考试时间 100分钟。 答卷前,考生务必将自己的姓名、准考号填写在答题卡上,并在规定位置粘贴考试用条形码。 答卷时,考生务必将答案涂写在答题卡上,答在试卷上的无效。考试结束后,将本试卷和答题卡 一并收回。 祝各位考生考试顺利! 第Ⅰ卷 注意事项: 1. 每小题选出答案后,用铅笔将答题卡上对应题目的答案标号涂黑,如需改动,用橡皮擦干 净后,再选涂其他答案标号。 2. 本卷共 55小题,共 95分 第一部分:英语知识运用(共两节,满分 45 分) 第一节:单项填空(共 15小题,每小题 1分,满分 15分) 从 A、B、C、D四个选项中,选出可以填入空白处的最佳选项。 例:Stand over there __________ you’ll be able to see it better. A. or B. and C. but D. while 答案是 B。 1. —Sorry, Liz. I think I was a bit rude to you. —__________, but don’t do that again! A. Go ahead B. Forget it C. It depends D. With pleasure 2. The quality of education in this small school is better than __________ in some larger schools. A. that B. one C. it D. this 3. Only when Lily walked into the office __________ that she had left the contract at home. A. she realized B. has she realized C. she has realized D. did she realize 4. —Jack, you seem excited. —__________? I won the first prize A. Guess what B. So what C. Pardon me D. Who cares 5. __________ in painting, John didn’t notice evening approaching. A. To absorb B. To be absorbed C. Absorbed D. Absorbing 6. Jane can’t attend the meeting at 3 o’clock this afternoon because she __________ a class at that time. A. will teach B. would teach C. has taught D. will be teaching 7. I __________ have worried before I came to the new school, for my classmates here are very friendly to me. A. mightn’t B. mustn’t C. needn’t D. couldn’t 8. __________ for two days, Steve managed to finish his report on schedule. A. To work B. Worked C. To be working D. Having worked 9. Despite the previous rounds of talks, no agreement __________ so far by the two sides. A. has been reached B. was reached C. will reach D. will have reached 10. Tom had to __________ the invitation to the party last weekend because he was too busy. A. turn in B. turn down C. turn over D. turn to 11. If you have any doubts about your health, you’d better __________ your doctor at once. A. convince B. consult C. avoid D. affect 12. We need to get to the root of the problem __________ we can solve it. A. while B. after C. before D. as 13. I wish I __________ at my sister’s wedding last Tuesday, but I was on a business trip in New York then. A. will be B. would be C. have been D. had been 14. See, your computer has broken down again! It doesn’t __________ sense to buy the cheapest brand of computer just to save a few dollars. A. have B. make C. display D. bring 15. The boss of the company is trying to create an easy atmosphere __________ his employees enjoy their work. A. where B. which C. when D. who 第二节:完形填空(共 20小题:每小题 1.5分,满分 30分) 阅读下面短文,掌握其大意,然后从 16~35各题所给出的 A、B、C、D四个选项中选出最 佳选项。 My fiance (未婚夫) and I were excited about shopping for our first home. But our funds were ___16___, and none of the houses in our price range seemed satisfactory. One agent 17 a house in particular. Although her description sounded wonderful, the price was 18 our range, so we declined. But she kept urging us to have a look 19 . We finally did and it was 20 at first sight. It was Our Home, small and charming, overlooking a quiet lake. Walking through the rooms and talking with the owners, a nice elderly couple, we felt the warmth and 21 of the marriage within that home. As perfect as it was, the price remained too high for us. But every day, we would sit by the lake, looking at the house and dreaming of ___22___ it would be like to live there. Days later, we made a(n) 23 —far below the asking price. Surprisingly, they didn’t ___24___ us. They renewed their offer 25 . It was also much more than we could afford, but far ___26___ than the original asking price. The next day, we got a 27 message that another buyer had offered a much higher price. Even so, we decided to talk with the 28 directly. We made our final offer, which 29 was thousands of dollars less than the other buyer’s bid. We knew it, 30 we had to try. “Sold!” said the owner. Then he 31 : He’d seen us sitting by the lake all those times; he knew how much we loved the place and that we’d 32 the years of work they had put into their home; he realized he would take a 33 by selling it to us, but it was worthwhile; we were the people they wanted to live there. He told us to consider the 34 in the price “an early wedding present.” That’s how we found our home and how I learned that when people are 35 they are not strangers, only friends we haven’t yet met. 16. A. needed B. limited C. enough D. large 17. A. recommended B. decorated C. sold D. rented 18. A. below B. within C. beyond D. between 19. A. at least B. at most C. at times D. at hand 20. A. relief B. concern C. love D. curiosity 21. A. pride B. happiness C. challenge D. desire 22. A. which B. why C. that D. what 23. A. effort B. offer C. promise D. profit 24. A. come across B. look after C. depend on D. laugh at 25. A. instead B. indeed C. aside D. apart 26. A. worse B. better C. less D. higher 27. A. relaxing B. disappointing C. pleasant D. regular 28. A. agents B. buyers C. managers D. owners 29. A. already B. still C. generally D. ever 30. A. so B. or C. for D. but 31. A. apologized B. complained C. criticized D. explained 32. A. check B. analyze C. appreciate D. ignore 33. A. loss B. risk C. chance D. lead 34. A. increase B. difference C. interest D. average 35. A. kind B. polite C. smart D. energetic 第二部分: 阅读理解(共 20 小题;每小题 2.5 分,满分 50 分) 阅读下列短文,从每题所给的 A、B、C、D四个选项中,选出最佳选项。 A University Room Regulations Approved and Prohibited Items The following items are approved for use in residential (住宿的) rooms: electric blankets, hair dryers, personal computers, radios, televisions and DVD players. Items that are not allowed in student rooms include: candles, ceiling fans, fireworks, waterbeds, sun lamps and wireless routers. Please note that any prohibited items will be taken away by the Office of Residence Life. Access to Residential Rooms Students are provided with a combination (组合密码) for their room door locks upon check-in. Do not share your room door lock combination with anyone. The Office of Residence Life may change the door lock combination at any time at the expense of the resident if it is found that the student has shared the combination with others. The fee is $25 to change a room combination. Cooking Policy Students living in buildings that have kitchens are only permitted to cook in the kitchen. Students must clean up after cooking. This is not the responsibility of housekeeping staff. Kitchens that are not kept clean may be closed for use. With the exception of using a small microwave oven (微波炉) to heat food, students are not permitted to cook in their rooms. Pet Policy No pets except fish are permitted in student rooms. Students who are found with pets, whether visiting or owned by the student, are subject to an initial fine of $100 and a continuing fine of $50 a day per pet. Students receive written notice when the fine goes into effect. If, one week from the date of written notice, the pet is not removed, the student is referred to the Student Court. Quiet Hours Residential buildings must maintain an atmosphere that supports the academic mission of the University. Minimum quiet hours in all campus residences are 11:00 pm to 8:00 am Sunday through Thursday. Quiet hours on Friday and Saturday nights are 1:00 am to 8:00 am. Students who violate quiet hours are subject to a fine of $25. 36. Which of the following items are allowed in student rooms? A. Ceiling fans and waterbeds. B. Wireless routers and radios. C. Hair dryers and candles. D. TVs and electric blankets. 37. What if a student is found to have told his combination to others? A. The combination should be changed. B. The Office should be charged. C. He should replace the door lock. D. He should check out of the room. 38. What do we know about the cooking policy? A. Amicrowave oven can be used. B. Cooking in student rooms is permitted. C. A housekeeper is to clean up the kitchen. D. Students are to close kitchen doors after cooking. 39. If a student has kept a cat in his room for a week since the warning, he will face __________. A. parent visits B. a fine of $100 C. the Student Court D. a written notice 40. When can students enjoy a party in residences? A. 7:00 am, Sunday. B. 7:30 am, Thursday. C. 11:30 pm, Monday. D. 00:30 am, Saturday. B Whether in the home or the workplace, social robots are going to become a lot more common in the next few years. Social robots are about to bring technology to the everyday world in a more humanized way, said Cynthia Breazeal, chief scientist at the robot company Jibo. While household robots today do the normal housework, social robots will be much more like companions than mere tools. For example, these robots will be able to distinguish when someone is happy or sad. This allows them to respond more appropriately to the user. The Jibo robot, arranged to ship later this year, is designed to be a personalized assistant. You can talk to the robot, ask it questions, and make requests for it to perform different tasks. The robot doesn’t just deliver general answers to questions; it responds based on what it learns about each individual in the household. It can do things such as reminding an elderly family member to take medicine or taking family photos. Social robots are not just finding their way into the home. They have potential applications in everything from education to health care and are already finding their way into some of these spaces. Fellow Robots is one company bringing social robots to the market. The company’s “Oshbot” robot is built to assist customers in a store, which can help the customers find items and help guide them to the product’s location in the store. It can also speak different languages and make recommendations for different items based on what the customer is shopping for. The more interaction the robot has with humans, the more it learns. But Oshbot, like other social robots, is not intended to replace workers, but to work alongside other employees. “We have technologies to train social robots to do things not for us, but with us,” said Breazeal. 41. How are social robots different from household robots? A. They can control their emotions. B. They are more like humans. C. They do the normal housework. D. They respond to users more slowly. 42. What can a Jibo robot do according to Paragraph 3? A. Communicate with you and perform operations. B. Answer your questions and make requests. C. Take your family pictures and deliver milk. D. Obey your orders and remind you to take pills. 43. What can Oshbot work as? A. A language teacher. B. A tour guide. C. A shop assistant. D. A private nurse. 44. We can learn from the last paragraph that social robots will __________. A. train employees B. be our workmates C. improve technologies D. take the place of workers 45. What does the passage mainly present? A. A new design idea of household robots. B. Marketing strategies for social robots. C. Information on household robots. D. An introduction to social robots. C One day when I was 12, my mother gave me an order: I was to walk to the public library, and borrow at least one book for the summer. This was one more weapon for her to defeat my strange problem — inability to read. In the library, I found my way into the “Children’s Room.” I sat down on the floor and pulled a few books off the shelf at random. The cover of a book caught my eye. It presented a picture of a beagle. I had recently had a beagle, the first and only animal companion I ever had as a child. He was my secret sharer, but one morning, he was gone, given away to someone who had the space and the money to care for him. I never forgot my beagle. There on the book’s cover was a beagle which looked identical to my dog. I ran my fingers over the picture of the dog on the cover. My eyes ran across the title, Amos, the Beagle with a Plan. Unknowingly, I had read the title. Without opening the book, I borrowed it from the library for the summer. Under the shade of a bush, I started to read about Amos. I read very, very slowly with difficulty. Though pages were turned slowly, I got the main idea of the story about a dog who, like mine, had been separated from his family and who finally found his way back home. That dog was my dog, and I was the little boy in the book. At the end of the story, my mind continued the final scene of reunion, on and on, until my own lost dog and I were, in my mind, running together. My mother’s call returned me to the real world. I suddenly realized something: I had read a book, and I had loved reading that book. Everyone knew I could not read. But I had read it. Books could be incredibly wonderful and I was going to read them. I never told my mother about my “miraculous” (奇迹般地) experience that summer, but she saw a slow but remarkable improvement in my classroom performance during the next year. And years later, she was proud that her son had read thousands of books, was awarded a PhD in literature, and authored his own books, articles, poetry and fiction. The power of the words has held. 46. The author’s mother told him to borrow a book in order to __________. A. encourage him to do more walking B. let him spend a meaningful summer C. help cure him of his reading problem D. make him learn more about weapons 47. The book caught the author’s eye because __________. A. it contained pretty pictures of animals B. it reminded him of his own dog C. he found its title easy to understand D. he liked children’s stories very much 48. Why could the author manage to read the book through? A. He was forced by his mother to read it. B. He identified with the story in the book. C. The book told the story of his pet dog. D. The happy ending of the story attracted him. 49. What can be inferred from the last paragraph? A. The author has become a successful writer. B. The author’s mother read the same book. C. The author’s mother rewarded him with books. D. The author has had happy summers ever since. 50. Which one could be the best title of the passage? A. The Charm of a Book B. Mum’s Strict Order C. Reunion with My Beagle D. My Passion for Reading D Once when I was facing a decision that involved high risk, I went to a friend. He looked at me for a moment, and then wrote a sentence containing the best advice I’ve ever had: Be bold and brave — and mighty (强大的) forces will come to your aid. Those words made me see clearly that when I had fallen short in the past, it was seldom because I had tried and failed. It was usually because I had let fear of failure stop me from trying at all. On the other hand, whenever I had plunged into deep water, forced by courage or circumstance, I had always been able to swim until I got my feet on the ground again. Boldness means a decision to bite off more than you can eat. And there is nothing mysterious about the mighty forces. They are potential powers we possess: energy, skill, sound judgment, creative ideas — even physical strength greater than most of us realize. Admittedly, those mighty forces are spiritual ones. But they are more important than physical ones. A college classmate of mine, Tim, was an excellent football player, even though he weighed much less than the average player. “In one game I suddenly found myself confronting a huge player, who had nothing but me between him and our goal line,” said Tim. “I was so frightened that I closed my eyes and desperately threw myself at that guy like a bullet — and stopped him cold.” Boldness — a willingness to extend yourself to the extreme—is not one that can be acquired overnight. But it can be taught to children and developed in adults. Confidence builds up. Surely, there will be setbacks (挫折) and disappointments in life; boldness in itself is no guarantee of success. But the person who tries to do something and fails is a lot better off than the person who tries to do nothing and succeeds. So, always try to live a little bit beyond your abilities—and you’ll find your abilities are greater than you ever dreamed. 51. Why was the author sometimes unable to reach his goal in the past? A. He faced huge risks. B. He lacked mighty forces. C. Fear prevented him from trying. D. Failure blocked his way to success. 52. What is the implied meaning of the underlined part? A. Swallow more than you can digest. B. Act slightly above your abilities. C. Develop more mysterious powers. D. Learn to make creative decisions. 53. What was especially important for Tim’s successful defense in the football game? A. His physical strength. B. His basic skill. C. His real fear. D. His spiritual force. 54. What can be learned from Paragraph 5? A. Confidence grows more rapidly in adults. B. Trying without success is meaningless. C. Repeated failure creates a better life. D. Boldness can be gained little by little. 55. What is the author’s purpose in writing this passage? A. To encourage people to be courageous. B. To advise people to build up physical power. C. To tell people the ways to guarantee success. D. To recommend people to develop more abilities. 第 II卷 注意事项: 1.用黑色墨水的钢笔或签字笔将答案写在答题卡上。 2.本卷共 6小题,共 35分。 第三部分: 写作 第一节:阅读表达(共 5小题;每小题 2分,满分 10分) 阅读短文,并按照题目要求用英语回答问题。 Six days a week, up and down the red hills of northeast Georgia, my grandfather brought the mail to the folks there. At age 68, he retired from the post office, but he never stopped serving the community. On his 80th birthday, I sent him a letter, noting the things we all should be thankful for — good health, good friends and good outcomes. By most measurements he was a happy man. Then I suggested it was time for him to slow down. At long last, in a comfortable home, with a generous pension, he should learn to take things easy. “Thank you for your nice words,” he wrote in his letter back, “and I know what you meant, but slowing down scares me. Life isn’t having it made; it’s getting it made.” “The finest and happiest years of our lives were not when all the debts were paid, and all difficult experiences had passed, and we had settled into a comfortable home. No. I go back years ago, when we lived in a three-room house, when we got up before daylight and worked till after dark to make ends meet. I rarely had more than four hours of sleep. But what I still can’t figure out is why I never got tired, never felt better in my life. I guess the answer is, we were fighting for survival, protecting and providing for those we loved. What matters are not the great moments, but the partial victories, the waiting, and even the defeats. It’s the journey, not the arrival, that counts.” The letter ended with a personal request: “Boy, on my next birthday, just tell me to wake up and get going, because I will have one less year to do things — and there are ten million things waiting to be done.” Christina Rossetti, an English poet, once said: “Does the road wind uphill all the way? Yes, to the very end.” Today, at 96, my grandfather is still on that long road, climbing. 56. What was the author’s grandfather before he retired? (no more than 5 words) 57. What did the author advise his grandfather to do in his letter? (no more than 10 words) 58. What is the grandfather’s view on life according to his letter back? (no more than 10 words) 59. How do you understand the underlined sentence in the last paragraph? (no more than 10 words) 60. Do you agree with the grandfather’s view on life? Give reasons in your own words, (no more than 20 words) 第二节:书面表达(满分 25分) 61. 假设你是晨光中学学生会主席李津,你校拟向美国友好交流学校的中文班捐赠一批图书。请 根据以下提示,写一封邮件与中文班班长 Chris 联系,并告知将于今年七月赴对方学校参加夏令 营时带去赠书。 (1)自我介绍; (2)拟捐赠图书的类型、册数、用途等; (3)询问对方的其他需求。 注意: (1)词数不少于 100; (2)请适当加入细节,使内容充实、行文连贯。 参考词汇: 学生会 the Student Union Dear Chris, 2015年普通高等学校招生全国统一考试 上海 英语试卷 考生注意: 1. 考试时间120分钟,试卷满分150分。 2. 本考试设试卷和答题纸两部分。试卷分为第I卷和第II卷。全卷共12页,所有答题必须涂(选择 题)或写(非选择题)的答题纸上,坐在试卷上一律不得分。 3. 答题前,务必在答题纸上填写准考证号和姓名,并将核对后的条形码贴在指定位置上,在答题 纸反面清楚地填写姓名。 第 I 卷 (共103分) I. Listening Comprehension Section A Directions: In Section A, you will hear ten short conversations between two speakers. At the end of each conversation, a question will be asked about what was said. The conversations and the questions will be spoken only once. After you hear a conversation and the question about it, read the four possible answers on your paper, and decide which one is the best answer to the questions you have heard. 1. A. impatient B. confused C. pleased D. regretful 2. A. at a bus stop B. at a laundry C. at the dentist’s D. at the chemist’s 3. A. An actor B. A salesman C. A translator D. A writer 4. A. He lost his classmate’s homework. B. He can’t help the woman with her math. C. He broke the woman’s calculator. D. He doesn’t know where the “on” button is. 5. A. The woman should go to another counter. B. The woman gives the man so many choices. C. The man dislike the sandwiches offered there. D. The man is having trouble deciding what to eat. 6. A. She has no idea where to find the man’s exam result. B. She isn’t allowed to tell students their grades. C. Dr. White hasn’t finish grading the papers. D. Dr. White doesn’t want to be contacted while he’s away. 7. A. Move to a neat dormitory B. Find a person to share their apartment C. Clean the room with the roommate D. Write an article about their roommate 8. A. Bob won’t take her advice B. Bob doesn’t want to go abroad C. She doesn’t think Bob should study overseas D. She hasn’t talked to Bob since he went aboard 9. A. The snack bar isn’t usually so empty. B. Dessert is served in the snack bar. C. The snack bar is near the library. D. Snacks aren’t allowed in the library. 10. A. Take her bicycle to the repair shop. B. Leave her bicycle outside. C. Clean the garage after the rain stops. D. Check if the garage is dry. Section B Directions: In Section B, you will hear two short passages, and you will be asked three questions on each of the passages. The passages will be read twice, but the questions will be spoken only once. When you hear a question, read the four possible answers on your paper and decide which one would be the best answer to the question you have heard. Questions 11 through 13 are based on the following passage. 11. A. It helps care for customers’ dogs. B. You have to buy food for dogs. C. None of the dogs are caged. D. There is a dog named Princess. 12. A. She likes the food there. B. She enjoys the fun with a pet. C. She can have free coffee. D. She doesn’t like to be alone. 13. A. A new kind of cafe. B. A new brand of cafe. C. A new home for pets. D. A new way to raise pets. Questions 14 through 16 are based on the following passage. 14. A. A trend that high achievers are given a lower salary. B. A view that life quality is more important than pay. C. A dream of the young for fast-paced jobs. D. A new term created by high achievers. 15. A. 10% B. 12% C. 6% D. 7% 16. A. People are less satisfied with their lives. B. The financial investment may increase. C. Well-paid jobs are not easy to find. D. Unexpected problems may arise. Section C Directions: In Section C, you will hear two longer conversations. The conversations will be read twice. After you hear each conversation, you are required to fill in the numbered blanks with the information you have heard. Write your answers on your answer sheet. Blanks 17 through 20 are based on the following conversation. Complete the form. Write NO MORE THAN ONE WORD for each answer. Blanks 21 through 24 are based on the following conversation. Complete the form. Write NO MORE THAN THREE WORDS for each answer. II. Grammar and Vocabulary Section A Directions: After reading the passages below, fill in the blanks to make the passages coherent and grammatically correct. For the blanks with a given word, fill in each blank with the proper form. of the given word; for the other blanks, use one word that best fits each blank. (A) Gift from a stranger SRT Service Notes Account No.: 17 Service Request: Check the 18 Solutions: Send another 19 2 pm on 20 In what way are these climbers special? They are all 21 . Why did they choose to conquer Mount Kilimanjaro? To prove 22 . What did they do in time of difficulty? They turned 23 . How did they record their adventure? By keeping 24 . My local supermarket is always busy. The first parking space I found was convenient, but I'd noticed a woman in a blue car circling for a while. (25) ____________ I was in a good mood, I let her have it. On the edge of the car park I backed into the next available spot—it was a tight fit. Pretty soon I'd made my way through the supermarket and was back in the fresh air. Feeling good, I (26) ____________ (empty) my purse change into the hands of a homeless man and helped a struggling woman reverse park. Just as I approached my car, 1 saw the woman I'd let have my car space earlier. She was giving me (27) ____________ odd look—half puzzled, half intent (热切的). I smiled and wished her a pleasant day. As I squeezed back into my car, I spotted the same lady (28) ____________ (look) in at me. "Hello," she said, hesitantly. "This (29) ____________ sound crazy but I was on my way to drop some of my mother's things off at the charity bins.” You are just so much (30) ____________ her.” You helped those people, I noticed, and you seemed so happy.” She looked at me meaningfully and passed a box in through the window. “I think she would like you to have it.” (31) ____________ (shock), I took it from her automatically. She smiled and walked away. After a pause, I opened the box. Inside was a beautiful gold necklace with a large grey pearl. It was (32) ____________ (nice) gift I'd ever received, and it was from a complete stranger. The necklace was around my neck, a warm reminder of human kindness. (B) Ask helpful Hannah Dear helpful Hannah, I’ve got a problem with my husband, Sam. He bought a smart phone a couple of months ago and he took it on our recent ski vacation to Colorado, it was a great trip except for one problem. He has a constant urge (33) ____________ for next messages; he checks his phone every five minutes! He’s so addicted to it that he just can’t stand the idea (34) ____________ there may be an important text. He can’t help checking even at inappropriate times like when we are eating in a restaurant and I am talking to him! He behaves (35) ____________ any small amount of boredom can make him feel the need to check his phone even when he know he shouldn’t. The temptation to see (36) ____________ is connecting him is just too great. When I ask him to put down the phone and stop (37)____________ (ignore) me, he say, “In a minute.” but still checks to see if (38) has posted something new on the Internet. Our life (39) ____________ (interrupt). If we go somewhere and I ask him to have the phone at home, he suffers from withdrawal symptom. May this dependency on his smart phone has become more than an everyday problem. I recently read an article about “nomophobia,” (40) ____________ is a real illness people can’t suffer from the fear of being without your phone! I am worried that Sam maybe suffering from this illness because he feels anxious if he doesn’t have his phone with him, even for a short time. Who would have thought that little devices like these could have brought so much trouble! Sick and Tired Sadie Section B Directions: Complete the following passage by using the words in the box. Each word can only be used once. Note that there is one word more than you need. A. access B. alternatives C. assigned D. confirmed E. conflicting F. elements G. function H. innovative I. prospective J. separate K. supporting Considering how much time people spend in offices, it is important that work be well designed. Well-designed office spaces help create a cooperation?s image. They motivate workers and they make an impression on people who visit and might be potential or, 41 , customer. They make businesses work better, and they are a part of the corporate culture we live in. As we move away from an industrial-based economy to a knowledge-based one, office designers have come up with 42 to the traditional work environments of the past. The design industry has moved away from a fixed office setup and created more flexible “strategic management environments.” These 43 solutions are to meant to support better organizational performances. As employee hierarchies (等级制度) have flattened or decreased, office designers‘ response to this change has been to move open-plain areas to more desirable locations within the office, and create fewer formal private offices. The need for increased flexibility has also been 44 by changes in work station design. Offices and work spaces often are not 45 to a given person on a permanent basis because of changes to method of working, new designs allow for expansion or movement of desks, storage, and equipment within the workstation. Another important design goal is communication, which designers have improved by lowering the walls that 46 workstations. Designers have also created informal gathering places,and upgraded employees’ 47 to heavily trafficked areas such as copy and coffee rooms. Corporate and institutional office designers often struggle to resolve a number of competing and often 48 demands, including budgetary limits, employee hierarchies, and technological innovation (especially in relation to computerization). These demands must also be balanced with the need to create interiors (内 饰 ) that in some way enhance,establish,or promote a company‘s image and will enable employees to 49 at their best. All these 50 of office design are related. The most successful office designs are like a good marriage --the well-designed office and the employees that occupy it are seemingly made for each other. III. Reading Comprehension Section A Directions: For each blank in the following passage there are four words or phrases marked A, B, C and D. Fill in each blank with the word or phrase that best fits the context. If you studied pictures that ancient people left on rock walls and you tried to determine their meaning, you would not detect interest in romance among the artists. 51 , you would see plenty of animals with people running after them. Life for ancient people’s earned to center on hunting and gathering wild foods for meals. In modern times, when food is available in grocery stores, finding love is more 52 in people’s lives. The 53 is all around us. It is easy to prepare a list of modern stories having to do with love. An endless number of books and movies qualify as love stories in popular culture. Researchers are studying whether love, a highly valued emotional state, can be 54 . They ask, what is love? Toothpaste companies want us to think attraction is all about clean teeth, but clean teeth go only so far. Scientists wonder how much the brain gets involved. You have probably heard that opposites attract but that 55 attract, too. One thing is certain: The truth about love is not yet set in stone. First Impression To help determine the 56 of attraction, researchers paired 164 college classmates and had them talk for 3, 6 or 10 minutes so they could get a sense of each other’s individuality. Then students were asked to 57 what kind of relationship they were likely to build with their partners. After nine weeks, they reported what happened. As it turned out, their 58 judgements often held true. Students seemed to 59 at an early stage who would best fit into their lives. The 60 Knows Scientists have also turned to nonhumans to increase understanding of attraction. Many animals give off pheromones — natural chemicals that can be detected by, and then can produce a response in, other animals of the same species. Pheromones can signal that an animal is either ready to fight or is feeling 61 to partnerships. In contrast, humans do not seem to be as 62 as other animals at detecting such chemicals. Smell, however, does seem to play a part in human attraction. Although we may not be aware of chemicals like pheromones consciously, we give and receive loads of information through smell in every interaction with other people. Face Value Being fond of someone seems to have a number of factors, including seeing something we find attractive. Researchers had people judge faces for 63 . The participants had 0.013 seconds to view each face, yet somehow they generally considered the images the same as people who had more time to study the same faces. The way we 64 attractiveness seem to be somewhat automatic. When shown an attractive face and then words with good or bad associations, people responded to 65 words faster after viewing an attractive face. Seeing something attractive seems to cause happy thinking. 51. A. Instead B. Therefore C. Moreover D. Otherwise 52. A. romantic B. stressful C. central D. beneficial 53. A. priority B. proof C. possibility D. principle 54. A. tested B. imposed C. changed D. created 55. A. appearances B. virtues C. similarities D. passions 56. A. illustrations B. imaginations C. ingredients D. instructions 57. A. predict B. investigate C. diagnose D. recall 58. A. critical B. initial C. random D.mature 59. A. memorize B. distinguish C. negotiate D. question 60. A. Nose B. Eye C. Heart D. Hand 61. A. open B. alert C. resistant D. superior 62. A. disappointed B. amazed C. confused D. gifted 63. A. emotion B. attractiveness C. individuality D. signals 64. A. enhance B. possess C. maintain D. assess 65. A. familiar B. plain C. positive D. insulting Section B Directions: Read the following three passages. Each passage is followed by several questions or unfinished statements. For each of them there are four choices marked A, B, C and D. Choose the one that fits best according to the information given in the passage you have just read. (A) Look to many of history’s cultural symbols, and there you’ll find an ancestor of Frosty, the snowman in the movie Frozen. It appeared on some of the first postcards, starred in some of the earliest silent movies, and was the subject of a couple of the earliest photos, dating all the way back to the 1800s. I discovered even more about one of humanity’s earliest forms of life art during several years of research around the world. For example, snowmen were a phenomenon in the Middle Ages, built with great skill and thought. At a time of limited means of expression, snow was like free art supplies dropped from the sky. It was a popular activity for couples to leisurely walk through town to view the temporary works of chilly art. Some were created by famous artists, including a 19-year-old Michelangelo, who in 1494 was appointed by the ruler of Florence, Italy, to build a snowman in his mansion’s courtyard. The Miracle of 1511 took place during six freezing works called the Winter of Death. The city of Brussels was covered in snowmen—an impressive scene that told stories on every street corner. Some were political in nature, criticizing the church and government. Some were a reflection of people’s imagination. For the people of Brussels, this was a defining moment of defining freedom. At least until spring arrived, by which time they were dealing with damaging floods. If you fear the heyday of the snowman has passed, don’t worry: I’ve learned that some explosive snowman history is still being made today. Every year since 1818, the people of Zurich, Switzerland, celebrate the beginning of spring by blowing up a snowman. On the third Monday of April, the holiday Sechselauten is kicked off when a cotton snowman called the Boogg is stuffed with explosive and paraded through town by bakers and other tradesmen who throw bread to the crowds. The parade ends with the Boogg being placed on a 40-foot pile of firewood. After the bells of the Church of St. Peter have rung six times, representing the passing of winter, the pile is lit. When the snowman explodes, winter is considered officially over—the quicker it is burnt down, the longer summer is said to be. 66. According to the passage, why did snowmen become a phenomenon in the Middle Ages? A. People thought of snow as holy art supplies. B. People longed to see masterpieces of snow. C. Building snowmen was a way for people to express themselves. D. Building snowmen helped people develop their skill and thought. 67. “The heyday of the snowman” (paragraph 4) means the time when __________. A. snowmen were made mainly by artists B. snowmen enjoyed great popularity C. snowmen were politically criticized D. snowmen caused damaging floods 68. In Zurich, the blowing up of the Boogg symbolizes __________. A. the start of the parade B. the coming of a longer summer C. the passing of the winter D. the success of tradesmen 69. What can be concluded about snowmen from the passage? A. They were appreciated in history B. They have lost their value C. They were related to movies D. They vary in shape and size (B) 70. In the film review, what is paragraph A mainly about? A. The introduction to the leading roles B. The writer’s opinion of acting C. The writer’s comments on the story D. The background information 71. According to the film review, “monster” (paragraph B) refers to __________. A. a gun-crazy hunter B. a brainy dog C. a scary rabbit D. a giant vegetable 72. Which of the following is a reason why the writer recommends the film? A. It’s full of wit and humour. B. Its characters show feelings without words. C. It is an adventure film directed by Peter Sallis. D. It is about the harmony between man and animals. (C) One of the executives gathered at the Aspen Institute for a day-long leadership workshop using the works of Shakespeare was discussing the role of Brutus in the death of Julius Caesar. “Brutus was not an honorable man,” he said. “He was a traitor(叛徒). And he murdered someone in cold blood.” The agreement was that Brutus had acted with cruelty when other options were available to him. He made a bad decision, they said—at least as it was presented by Shakespeare—to take the lead in murdering Julius Caesar. And though one of the executives acknowledged that Brutus had the good of the republic in mind, Caesar was nevertheless his superior. “You have to endeavor,” the executives said, “our policy is to obey the chain of command.” During the last few years, business executives and book writers looking for a new way to advise corporate America have been exploiting Shakespeare’s wisdom for profitable ends. None more so than husband and wife team Kenneth and Carol Adelman, well-known advisers to the White House, who started up a training company called “Movers and Shakespeares”. They are amateur Shakespeare scholars and Shakespeare lovers, and they have combined their passion and their high level contacts into a management training business. They conduct between 30 and 40 workshops annually, focusing on half a dozen different plays, mostly for corporations, but also for government agencies. The workshops all take the same form, focusing on a single play as a kind of case study, and using individual scenes as specific lessons. In Julius Caesar , sly provocation(狡诈的挑唆)of Brutus to take up arms against the what was a basis for a discussion of methods of team building and grass roots organism. Although neither of the Adelmans is academically trained in literature, the programmes, contain plenty of Shakespeare tradition and background. Their workshop on Henry V, for example, includes a helpful explanation of Henry’s winning strategy at the Battle of Agincourt. But they do come to the text with a few biases (偏向 ): their reading of Henry V minimizes his misuse of power. Instead, they emphasize the story of the youth who seizes opportunity and becomes a masterful leader. And at the workshop on Caesar, Mr. Adelmans had little good to say about Brutus, saying “the noblest Roman of them all” couldn’t make his mind up about things. Many of the participants pointed to very specific elements in the play that they felt related Caesar’s pride, which led to his murder, and Brutus’s mistakes in leading the after the murder, they said, raise vital questions for anyone serving as a business when and how do you resist the boss? 73. According to paragraph 1, what did all the executives think of Brutus? A. Cruel. B. Superior. C. Honourable. D. Rude 74. According to the passage, the Adelmans set up “Movers and Shakespeares” to __________. A. help executives to understand Shakespeare’s plays better B. give advice on leadership by analyzing Shakespeare’s plays C. provide case studies of Shakespeare’s plays in literature workshops D. guide government agencies to follow the characters in Shakespeare’s plays. 75. Why do the Adelmans conduct a workshop on Henry V? A. To highlight the importance of catching opportunities. B. To encourage masterful leaders to plan strategies to win. C. To illustrate the harm of prejudices in management. D. To warn executives against power misuse. 76. It can be inferred from the passage that __________. A. the Adelmans’ programme proves biased as the roles of characters are maximized. B. executives feel bored with too many specific elements of Shakespeare’s plays. C. the Adelmans will make more profits if they are professional scholars. D. Shakespeare has played an important role in the management field. 77. The best title for the passage is __________. A. Shakespeare’s plays: Executives reconsider corporate culture B. Shakespeare’s plays: An essential key to business success C. Shakespeare’s plays: a lesson for business motivation D. Shakespeare’s plays: Dramatic training brings dramatic results Section C Directions: Read the passage carefully. Then answer the questions or complete the statements in the fewest possible words. Youth sport has the potential to accomplish three important objectives in children’s development. First, sport programs can provide youth with opportunities to be physically active, which can lead to improved physical health. Second, youth sport programs have long been considered important to youth’s psychosocial development, providing opportunities to learn important life skills such as cooperation, discipline, leadership, and self-control. Third, youth sport programs are critical for the learning of motor skills; these motor skills serve as a foundation for future national sport stars and recreational adult sport participants. When coachers develop activities for youth practices and when sport organizations design youth-sport programs, they must consider the implication of deliberate play and deliberate practice. Research from Telama (2006) states that regular participation in deliberate play or deliberate practice activities during childhood and youth (ages nine to eighteen) increases the likelihood of participation in sports during adulthood by six times for both males and females. Côté (2002) defines deliberate play activities in sport as those designed to maximize enjoyment. These activities are regulated by flexible rules adapted from standardized sport rules and are set up by the children or by an involved adult. Children typically change rules to find a point where their game is similar to the actual sport but still allows for play at their level. For example, children may change soccer and basketball rules to suit their needs and environment (e.g. in the street. on a playing field or in someone’s backyard). When involved in deliberate play activities, children are less concerned with the outcome of their outcome of their behavior. (whether they win or lose) than with the behavior. (having fun). On the other hand, Ericsson (1993) suggests that the most effective learning occurs through involvement in highly structured activities defined as deliberate practice. Deliberate practice activities require effort, produce no immediate rewards, and are motivated by the goal of improving performance rather than the goal of enjoyment. When individuals are involved in deliberate play, they experiment with different combinations of behaviors, but not necessarily in the most effective way to improve performance. In contrast, when individuals are involved in deliberate practice, they exhibit behavior. focused on improving performance by the most effective means available. For example, the backhand skills in tennis could be learned and improved over time by playing matches or by creating fun practice situations. However, players could more effectively improve their backhand performance by practicing drills that might be considered less enjoyable. Although drills are used in most effective means available practice might not be the most enjoyable, they might be the most relevant to improving performance. (Note: Answer the questions or complete the statements in NO MORE THAN TEN WORDS) 78. Besides the learning of motor skills, what are the other two important objectives of youth sport? 79. If children participate in deliberate play or deliberate practice activities, they are more likely to ___________________. 80. In deliberate play activities, what do children do to maximize enjoyment? 81. In contrast to deliberate play, deliberate practice is aimed at ___________________. 第 II 卷 (共47分) I. Translation Directions: Translate the following sentences into English, using the words given in the brackets. 1. 美食是人们造访上海的乐趣之一。 (visit) 2. 街头艺术家运用创意将鲜艳明亮的色彩带进了老社区。 (bring) 3. 在你生命中,如果有一个人你需要对他说对不起,那么就去向他道歉吧。 (apology) 4. 这个游戏的独特之处在于它让孩子学会如何应对现实生活中的问题。 (what) 5. 申请材料需要精心准备,这样你心仪的学校才会对你的能力有全面,准确地了解。(in order that) II. Guided Writing Directions: Write an English composition in 120–150 words according to the instructions given below in Chinese. 学校即将举办“读书节”,目前正广泛征集“读书节”宣传册图片。假设你是该校学生潘阳,你 已找到以下三幅图片,决定给读书节组委会写一封信,推荐其中一幅,你的信须包括以下内容: 1.简单描述你想推荐的那幅图片; 2.阐述你用这幅图片宣传“读书节”的理由。 2015年普通高等学校招生全国统一考试(广东卷) 英 语 本试卷共三大题,满分 135分。考试用时 120分钟。 注意事项:1. 答卷前,考生务必用黑色字迹的钢笔或签字笔将自己的姓名、考生号、试室号和座 位号填写在答题卡上。用 2B铅笔将试卷类型(B)填涂在答题卡相应位置上。将条 形码横贴在答题卡右上角“条形码粘贴处”。 2. 选择题每小题选出答案后,用 2B铅笔把答题卡上对应题目的答案信息点涂黑,如 需改动,用橡皮擦干净后,再选涂其他答案,答案不能答在试卷上。 3. 非选择题必须用黑色字迹钢笔或签字笔作答,答案必须写在答题卡各题目指定区域 内相应位置上;如需改动,先划掉原来的答案,然后再写上新的答案,不准使用铅 笔和涂改液。不按以上要求作答的答案无效。 4. 考生必须保持答题卷和答题卡的整洁,考试结束后,将试卷、答题卷和答题卡一并 交回。 I语言知识及应用(共两节,满分 45 分) 第一节 完形填空(共 15小题;每小题 2分,满分 30分) 阅读下面短文,掌握其大意,然后从 1~15各题所给的 A、B、C和 D项中,选出最佳选项, 并在答题卡上将该项涂黑。 How long can human beings live? Most scientists who study old age think that the human body is 1 to live no longer than 120 years. However, 110 years is probably the longest that anyone could hope to live —— if he or she is 2 healthy and lucky. Some scientists even say we can live as long as 130 years! Yet, our cells simply cannot continue to reproduce 3 . They wear out, and as a result, we get old and 4 die. Even though we can’t live forever, we are living a 5 life than ever before. In 1900, the average American life span (寿命) was only 47 years, but today it is 75 years! When does old age begin then? Sixty-five may be out-of-date as the 6 line between middle age and old age. After all, many older people don’t begin to experience physical and mental 7 until after age 75. People are living longer because more people 8 childhood. Before modern medicine changed the laws of nature, many children died of common childhood 9 . Now that the chances of dying 10 are much lower, the chances of living long are much higher due to better diets and health care. On the whole, our population is getting older. The 11 in our population will have lasting effects on our social development and our way of life. Some people fear such changes will be for the worse, while some see 12 , not disaster, many men and women in their “golden years” are healthy, still active, and young in 13 if not in age. As the society grows old, we need the 14 of our older citizens. With long lives ahead of them, they need to 15 active and devoted. 1. A. designed B. selected C. improved D. discovered 2. A. completely B. generally C. apparently D. extremely 3. A. rapidly B. harmlessly C. endlessly D. separately 4. A. eventually B. hopelessly C. automatically D. desperately 5. A. busier B. longer C. richer D. happier 6. A. finishing B. guiding C. waiting D. dividing 7. A. stress B. damage C. decline D. failure 8. A. survive B. enjoy C. remember D. value 9. A. problems B. fears C. worries D. diseases 10. A. poor B. young C. sick D. quiet 11. A. changes B. recovery C. safety D. increases 12. A. dreams B. chances C. strengths D. choices 13. A. mind B. appearance C. voice D. movement 14. A. protection B. suggestions C. contributions D. permission 15. A. sound B. appear C. turn D. stay 第二节 语法填空(共 10小题; 每小题 1.5分,满分 15分) 阅读下面短文,按照句子结构的语法性和上下文连贯的要求,在空格处填入一个适当的词或使 用括号中词语的正确形式填空,并将答案填写在答题卡标号为 16-25的相应位置上。 Mr. Johnson lived in the woods with his wife and children. He owned 16 farm, which looked almost abandoned. 17 (lucky), he also had a cow which produced milk every day. He sold or exchanged some of the milk in the towns nearly 18 other food and made cheese and butter for the family with what 19 (leave). The cow was their only means of support, in fact. One day, the cow was eating grass 20 it began to rain heavily. While making great efforts to run away, she 21 (fall) over the hill and died. Then the Johnson tried to make a living 22 the cow. In order to support his family, Mr. Johnson began to plant herbs and vegetables. Since the plants took a while to grow, he started cutting down trees 23 (sell) the wood. Thinking about his children’s clothes, he started growing cotton too. When harvest came around, he was already selling herbs, vegetables and cotton in the market 24 people from the town met regularly. Now it occurred to 25 that his farm had much potential and that the death of the cow was a bit of luck. 16. __________ 17. __________ 18. __________ 19. __________ 20. __________ 21. __________ 22. __________ 23. __________ 24. __________ 25. __________ II 阅读(共两节,25题,满分 50分) 第一节 阅读理解(共 20小题;每小题 2分,满分 40分) 阅读下列短文,从每题所给的 A、B、C和 D项中,选出最佳选项,并在答题卡上将该项涂 黑。 A Peter loved to shop used articles. Almost a month ago, he bought popular word game that used little pieces of wood with different letters on them. As he was purchasing it, the salesgirl said, “Uh, look, the game box haven’t even been opened yet. That might be worth some money. ” Peter examined the box, and, sure enough, it was completely covered in factory-sealed plastic. And he saw a date of 1973 on the back of the box. “You should put that up for auction (拍卖) on the Internet, and see what happens.”the salesgirl said. “Yes, you’re right. People like something rare.” Peter agreed, “I can’t imagine there being very many unopened boxes of this game still around 40 years later.” “Don’t forget to tell me if you sell it.” the salesgirl smiled. “No problem.” Peter said. After he got home, Peter went online to several auction websites looking for his game. But he couldn’t find it. Then he typed in the name of the word game and hit Search. The search result was 543 websites containing information about the changes of the game. Over the years, the game had been produced using letters in different sizes and game boards in different colors. He also found some lists of game fans looking for various versions of the game. Peter emailed some of them, telling them what he had. Two weeks later, Peter went back to the shop. “Hello. Do you still remember the unopened word game?” The salesgirl looked at him for a second, then recognized him and said, “Oh, hi!” “I’ve got something for you,” Peter said. “I sold the game and made $1,000. Thank you for your suggestion.” He handed her three $ 100 bills. “Wow!” the salesgirl cried out. “Thank you, I never expected it.” 26. Which of the following best describes Peter’s word game? A. It was made around 40 years ago. B. It had game boards in different sizes. C. It was kept in a plastic bag with a seal. D. It had little pieces of wood in different colors. 27. What did the salesgirl probably think of Peter’s word game? A. Old and handy. B. Rare and valuable. C. Classic and attractive. D. Colorful and interesting 28. Peter got the names of the game fans from __________. A. an auction B. the Internet C. a game shop D. the second-hand shop 29. What happened at the end of the story? A. Peter gave the girl $300 as a reward. B. The salesgirl became Peter’s friend. C. Peter returned the word game for $ 1,000. D. The salesgirl felt confused to see Peter again. 30. What is the main theme of the story? A. It’s important to keep a promise. B. It’s great to share in other people’s happiness. C. We should be grateful for the help from others. D. Something rare is worth a large amount of money. B When I was nine years old, I loved to go fishing with my dad. But the only thing that wasn’t very fun about it was that he could catch many fish while I couldn’t catch anything. I usually got pretty upset and kept asking him why. He always answered, “Son, if you want to catch a fish, you have to think like a fish”, I remember being even more upset then because, “I’m not a fish!” I didn’t know how to think like a fish. Besides, I reasoned, how could what I think influence what a fish does? As I got a little older I began to understand what my dad really meant. So, I read some books on fish. And I even joined the local fishing club and started attending the monthly meetings. I learned that a fish is a cold-blooded animal and therefore is very sensitive to water temperature. That is why fish prefer shallow water to deep water because the former is warmer. Besides, water is usually warmer in direct sunlight than in the shade. Yet, fish don’t have any eyelids(眼皮) and the sun huts their eyes… The more I understood fish, the more I became effective at finding and catching them.. When I grew up and entered the business world, I remember hearing my first boss say, “We all need to think like sales people.” But it didn’t completely make sense. My dad never once said, “If you want to catch a fish you need to think like a fisherman.” What he said was, “You need to think like a fish.” Years later, with great efforts to promote long-term services to people much older and richer than me, I gradually learned what we all need is to think more like customers. It is not an easy job. I will show you how in the following chapters. 31. Why was the author upset in fishing trips when he was nine? A. He could not catch a fish. B. His father was not patient with him. C. His father did not teach him fishing. D. He could not influence a fish as his father did. 32. What did the author’s father really mean? A. To read about fish. B. To learn fishing by oneself. C. To understand what fish think. D. To study fishing in many ways. 33. According to the author, fish are most likely to be found __________. A. in deep water on sunny days B. in deep water on cloudy days C. in shallow water under sunlight D. in shallow water under waterside trees. 34. After entering the business world, the author found __________. A. it easy to think like a customer B. his father’s fishing advice inspiring C. his first boss’s sales ideas reasonable D. it difficult to sell services to poor people 35. This passage most likely comes from _________. A. a fishing guide B. a popular sales book C. a novel on childhood D. a millionaire’s biography C Daniel Anderson, a famous psychologist, believes it’s important to distinguish television’s influences on children from those of the family. We tend to blame TV, he says, for problems it doesn’t really cause, overlooking our own roles in shaping children’s minds. One traditional belief about television is that it reduces a child’s ability to think and to understand the world. While watching TV, children do not merely absorb words and images (影像 ). Instead, they learn both explicit and hidden meanings from what they see. Actually, children learn early the psychology of characters in TV shows. Furthermore, as many teachers agree, children understand far more when parents watch TV with them, explaining new words and ideas. Yet, most parents use an educational program as a chance to park their kids in front of the set and do something in another room. Another argument against television is that it replaces reading as a form of entertainment. But according to Anderson, the amount of time spent watching television is not related to reading ability. TV doesn’t take the place of reading for most children; it takes the place of similar sorts of recreation, such as listening to the radio and playing sports. Things like parents’ educational background have a stronger influence on a child’s reading. “A child’s reading ability is best predicted by how much a parent reads.” Anderson says. Traditional wisdom also has it that heavy television-watching lowers IQ (智商 ) scores and affects school performance. But here, too, Anderson notes that no studies have proved it. In fact, research suggests that it’s the other way around. “If you’re smart young, you’ll watch less TV when you’re older,” Anderson says. Yet, people of lower IQ tend to be lifelong television viewers. For years researchers have attempted to show that television is dangerous to children. However, by showing that television promotes none of the dangerous effects as conventionally believed, Anderson suggests that television cannot be condemned without considering other influences. 36. By watching TV, children learn __________. A. images through words B. more than explicit meanings C. more about images than words D. little about people’s psychology 37. An educational program is best watched by a child __________. A. on his own B. with other kids C. with his parents D. with his teachers 38. Which of the following is most related to children’s reading ability? A. Radio-listening B. Television-watching C. Parents’ reading list D. Parents’ educational background 39. Anderson believed that __________. A. the more a child watches TV, the smarter he is B. the younger a child is, the more he watches TV C. the smarter a child is, the less likely he gets addicted to TV D. the less a child watches TV, the better he performs at school 40. What is the main purpose of the passage? A. To advise on the educational use of TV. B. To describe TV’s harmful effects on children. C. To explain traditional views on TV influences. D. To present Anderson’s unconventional ideas. D It was once common to regard Britain as a society with class distinction. Each class had unique characteristics. In recent years, many writers have begun to speak the 'decline of class' and 'classless society' in Britain. And in modern day consumer society everyone is considered to be middle class. But pronouncing the death of class is too early. A recent wide-ranging society of public opinion found 90 percent of people still placing themselves in particular class; 73 percent agreed that class was still a vital part of British society; and 52 percent thought there were still sharp class differences. Thus, class may not be culturally and politically obvious, yet it remains an important part of British society. Britain seems to have a love of stratification. One unchanging aspect of a British person's class position is accent. The words a person speaks tell her or his class. A study of British accents during 1970s found that a voice sounding like a BBC newsreader was viewed as the most attractive voice, Most people said this accent sounded 'educated' and 'soft'. The accents placed at the bottom in this study, on the other hand, were regional(地区的)city accents. These accents were seen as 'common' and 'ugly'. However, a similar study of British accents in the US turned these results upside down and placed some regional accents as the most attractive and BBC English as the least. This suggests that British attitudes towards accent have deep roots and are based on class prejudice. In recent years, however, young upper middle-class people in London, have begun to adopt some regional accents, in order to hide their class origins. This is an indication of class becoming unnoticed. However, the 1995 pop song 'Common People' puts forward the view that though a middle-class person may 'want to live like common people' they can never appreciate the reality of a working-class life. 41. A recent study of public opinion shows that in modern Britain __________. A. it is time to end class distinction B. most people belong to middle class C. it is easy to recognize a person’s class D. people regard themselves socially different 42. The word stratification in Paragraph 3 is closest in meaning to __________. A. variety B. most people belong to middle class C. authority D. qualification 43. The study in the US showed that BBC English was regarded as __________. A. regional B. educated C. prejudiced D. unattractive 44. British attitudes towards accent __________. A. have a long tradition B. are based on regional status C. are shared by the Americans D. have changed in recent years 45. What is the main idea of the passage? A. The middle class is expanding B. A person’s accent reflects his class C. Class is a key part of British society D. Each class has unique characteristics. 第二节 信息匹配(共 5小题;每小题 2分,满分 10分) 请阅读下列应用文及相关信息,并按照要求匹配信息。请在答题卡上将对应题号的相应选项 字母涂黑。 首先阅读下列活动介绍: A. B. A Night of Glamor and Intrigue at Shanghai Bund in 1930 To celebrate Asia heritage month, Trendy New York is proud to present “Cheongsam Night out--A date with Cheongsam beauties in Shanghai Bund 1930”. May 16, 9:00 PM-May 17, 12:00 AM. EDT 330 West 40th Street, New York. NY 10018 Picking Partners---NEW YORK Featuring adaptations from Chinese and Western classic, including works from Chinese Academy Award---winning composer Tan Dun, the Beijing Guitar Duo teamed up with Cuban guitar virtuoso Manuel Barrueco (right) for a China West Concert at the New York Historical Society on April 23. C. D. Heroes of History: Legacy of My Chinese Family Join us as actress Tina Chen recounts the fascinating story of three generations of her mother’s family and their contributions to the history of China. Friday, May 8, 6:30PM--7:30PM China Institute 125 East 65th Street, New York, NY 10065 Great shorts---NEW YORK A photography exhibition held by HAN Media to celebrate its founding in New York City on April 24, featuring three emerging Chinese photographers; Yingxi Michael Shi, Haiyin Lin and Liming Guan, whose works have appeared in publications such as Vogue, ELLE, The New York Times and others. E. F. Forbidden delights---NEW YORK The first session of the China Institute in America short course Beijing: The City Through Its Architecture opens on Wednesday. Nancy S. Steinhardt discusses the Forbidden City and Beijing’s imperial architecture. Passing on the Kunqu Art: From Master to Disciples Kunqu Society, the classical Chinese theater which combines singing, dancing and acting to literary works by masters of Ming and Qing Dynasties, performing introduces four signature plays of Kunqu Master Jiqing Zhang to American audiences. Sunday, April 19, 2:00PM, EST Miller Theatre at Columbia University 2960 Broadway, New York, NY 10027 请根据以下人物介绍选择他们可能参加的活动: 46. Edward Leonardo Norton, connoisseur of Chinese and Japanese antiques. He has a strong interest in classical Chinese literary works. He even starts going to evening classes to learn classical Chinese at Columbia University. 47. Daphne Sui-yuan Tan, former director of National Association of Photographers. After reading some history books on how the first group of Chinese immigrants survived in America of the 19th century, she has become keen on her own family history and that of others. 48. Sharon Collins, pop singer and amateur photographer. Her marriage with a serious music critic has drawn her to his world, so she is now crazy about classical music and will not miss any chance to attend a concert with her husband. 49. Michelle Higgins, eminent photographer and columnist for quite a few internationally-known travel magazines. Recently, she has shown great interest in photo exhibits which feature young artists with Islamic or Chinese background. 50. Caroline Hugo, famous writer and influential movie critic. Last year her fantasy story which involved the mysterious Forbidden City received critical acclaim. Now she is conceiving a romance that has Shanghai of the 1930s as the setting. III写作(共两节, 满分 40分) 第一节 基础写作(共 1小题,满分 15分) 你接受了一项写作任务,为英语校报写一篇科技报道。 [写作内容] 请根据以下信息,介绍国外医疗行业出现的一项新技术。内容包括: 技术名称:DNA检测 检测方法:唾液样本分析 检测费用:125英镑 检测时长:4到 6周 检测用途: 1. 预测重大疾病 2. 预知食物偏好 3. 提示合适的锻炼方式 检测影响: 1. 增强健康意识 2. 易引起过度焦虑 *唾液样本:saliva sample [写作要求] 只能用 5个句子表达全部内容; [评分标准] 句子结构准确,信息内容完整,篇章连贯。 第二节 读写任务(共 1小题,满分 25分) 阅读下面短文,然后按照要求写一篇 150词左右的英语短文。 On the first day of her work, Sally found that a class full of problems was waiting for her. Six teachers had quit before her. When she walked in to the classroom, it was chaos: two boys were fighting in the far corner, yet the rest of the class seemed not to notice them; some girls were chatting and some were running about; paper, food packages and other garbage were littered around.... Just when she was about to speak, a student rushed in and pushed her aside! He was twenty minutes late! Sally walked onto the platform, picked up a piece of chalk and wrote on the blackboard: “Rule 1: We are family! “ All students stopped to look at her. And she continued with Rule 2, Rule 3… In the following weeks, Sally worked out 10 class rules and posted them on the walls of the classroom. She patiently explained all the rules to the students and require everyone to follow them. Surprisingly, Sally was not driven out like the former teachers; instead, she won respect from the students. Over the year, she witnessed gradual change in the class. At the graduation ceremony, just as she expected, she was very proud to stand with a class of care, manners and confidence. [写作内容] 1. 用约 30个词概括上文的主要内容。 2. 用约 120个词就班规谈谈你的想法,内容包括: (1) 你们班最突出的问题是什么? (2) 针对该问题你会设计一条什么班规? (3)你认为班规会带来什么影响? [写作要求] 1. 作文中可以使用亲身经历或虚构的故事,也可以参照阅读材料的内容,但不得直接引用原 文的句子。 2. 作文中不得出现真实姓名和学校名称。 [评分标准] 概括准确,语言规范,内容合适,篇章连贯。 2015年普通高等学校招生全国统一考试(江苏卷) 英 语 注 意 事 项 考生在答题前请认真阅读本注意事项及各题答题要求 1. 本试卷包含选择题(第 1 题~第 70题,共 70题)和非选择题(第 71题~第 81题,共 11题)两部分。 本试卷满分为 120分,考试时间为 120分钟。考试结束后,请将本试卷和答题卡一并交回。 2. 答题前,请务必将自己的姓名、准考证号用 0.5毫米黑色墨水笔填写在试卷及答题卡的规定位置上。 3. 请认真核对监考员在答题卡上所粘贴的条形码上的姓名、准考证号与本人是否相符。 4. 作答选择题,必须用 2B铅笔将答题卡上对应选项的方框涂满、涂黑;如需改动,请用橡皮檫干净 后,再选涂其他答案。作答非选择题,必须用 0.5毫米黑色墨水签字笔在答题卡上指定位置作答,在 其他位置作答一律无效。 第一部分 听力(共两节,满分 20 分) 做题时,先将答案标在试卷上。 录音内容结束后,你将有两分钟的时间将试卷上的答案转涂到 答题卡上。 第一节(共 5 小题;每小题 1 分,满分 5 分) 听下面 5 段对话。 每段对话后有一个小题,从题中所给的 A、B、C 三个选项中选出最佳选项, 并标在试卷的相应位置。 听完每段对话后,你都有 10 秒钟的时间来回答有关小题和阅读下一小 题。 每段对话仅读一遍。 例: How much is the shirt? A. £ 19.15 B. £ 9.18 C. £ 9.15 答案是 C。 1. 1. What time is it now? A. 9:10 B. 9:50 C. 10:00 2. What does the woman think of the weather? It’s nice. It’s warm It’s cold. 3. What will the man do? A. Attend a meeting. B. Give a lecture C. Leave his office. 4. What is the woman’s opinion about the course? A. Too hard B. Worth taking. C. Very easy. 5. What does the woman want the man to do? A. Speak louder B. Apologize to her. C. Turn off the radio. 第二节 (共 15 小题;每小题 1 分,满分 15 分) 听下面 5 段对话或独白。 每段对话或独白后有几个小题,从题中所给的 A、B、C 三个选项中选 出最佳选项,并标在试卷的相应位置。 听每段对话或独白前,你将有时间阅读各个小题,每小题 5 秒钟;听完后,各小题将给出 5 秒钟的作答时间。 每段对话或独白读两遍。 听第 6段材料,回答第 6、7题。 6. How long did Michael stay in China? A. Five days. B. One week. C. Two weeks. 7. Where did Michael go last year? A. Russia B. Norway C. India 听第 7段材料,回答第 8、9题。 8. What food does Sally like? A. Chicken. B. Fish. C. Eggs. 9. What are the speakers going to do? A. Cook dinner. C. Go shopping. C. Order dishes. 听第 8段材料,回答第 10至 12题。 10. Where are the speakers? A. In a hospital. B. In the office. C. At home. 11. When is the report due? A. Thursday. B. Friday. C. Next Monday. 12. What does George suggest Stephanie do with the report? A. Improve it. B. Hand it in later. C. Leave it with him. 听第 9段材料,回答第 13至 16题。 13. What is the probable relationship between the speakers? A. Salesperson and customer. B. Homeowner and cleaner. C. Husband and wife. 14. What kind of apartment do the speakers prefer? A. One with two bedroom. B. One without furniture. C. One near a market. 15. How much rent should one pay for the one-bedroom apartment? A. $350. B. $400. C. $415. 16. Where is the apartment the speakers would like to see? A. On Lake Street B. On Market Street. C. On South Street. 听第 8段材料,回答第 10至 12题。 17. What percentage of the world’s tea exports go to Britain? A. About 15%. B. About 30%. C. Over 40%. 18. Why do tea tasters taste tea with milk? A. Most British people drink that way. B. Tea tastes much better with milk. C. Tea with milk is healthy. 19. Who suggests a price for each tea? A. Tea tasters. B. Tea exporters. C. Tea companies. 20. What is the speaker talking about? A. The life of tea tasters. B. Afternoon tea in Britain. C. The London Tea Trade Centre. 第二部分: 英语知识运用 (共两节, 满分 35 分) 第一节: 单项填空 (共 15 小题; 每小题 1 分, 满分 15 分) 请阅读下面各题, 从题中所给的 A、 B、 C、 D 四个选项中, 选出最佳选项, 并在答题卡上将 该项涂黑。 例: It is generally considered unwise to give a child __________ he or she wants. A. however B. whatever C. whichever D. whenever 答案是 B。 21. The number of smokers, __________ is reported, has dropped by 17 percent in just one year. A. it B. which C. what D. as 22. Schools should be lively places where individuals are encouraged to _____ to their greatest potential. A. accelerate B. improve C. perform D. develop 23. —Jim, can you work this Sunday? —__________? I’ve been working for two weeks on end. A. Why me B. Why not C. What if D. So what 24. Much time __________ sitting at a desk, office workers are generally troubled by health problems. A. being spent B. having spent C. spent D. spending 25. __________ Li Bai, a great Chinese poet, was born is known to the public, but some won’t accept it. A. That B. Why C. Where D. How 26. It is so cold that you can’t go outside __________ fully covered in thick clothes. A. if B. unless C. once D. when 27. The university started some new language programs to __________ the country’s Silk Road Economic Belt. A. apply to B. cater for C. appeal to D. hunt for 28. It might have saved me some trouble __________ the schedule. A. did I know B. have I known C. do I know D. had I known 29. The whole team _____ Cristiano Ronaldo, and he seldom lets them down. A. wait on B. focus on C. count on D. call on 30. The real reason why prices __________ , and still are, too high is complex, and no short discussion can satisfactorily explain this problem. A. Were B. will be C. have been D. had been 31. The police officers decided to conduct a thorough and __________ review of the case. A. comprehensive B. complicated C. Conscious D. crucial 32. Some schools will have to make __________ in agreement with the national soccer reform. A. judgments B. adjustments C. comments D. achievements 33. —Why didn’t you invite John to your birthday party? —Well, you know he’s __________. A. an early bird B. a wet blanket C. a lucky dog D. a tough nut 34. Many of the things we now benefit from would not be around __________ Thomas Edison. A. thanks to B. regardless of C. aside from D. but for 35. —Go and say sorry to your Mom, Dave. —I’d like to, but I’m afraid she won’t be happy with my __________. A. requests B. excuses C. apologies D. regrets 第二节: 完形填空 (共 20 小题; 每小题 1 分, 满分 20 分) 请阅读下面短文, 从短文后各题所给的 A、B、C、D 四个选项中, 选出最佳选项, 并在答题卡 上将该项涂黑。 I was required to read one of Bernie Siegel’s books in college and was hooked on his positivity from that moment on. The stories of his unconventional 36 and the exceptional patients he wrote about were so 37 to me and had such a big 38 on how I saw life from then on.Who knew that so many years later I would look to Dr. Bernie and his CDs again to 39 my own cancer experience? I’m an ambitious 40 , and when I started going through chemo (化疗) , even though I’m a very 41 person, I lost my drive to write. I was just too tired and not in the 42 . One day,while waiting to go in for 43 , I had one of Dr. Bernie’s books in my hand. Another patient ___44___ what I was reading and struck up a conversation with me 45 he had one of his books with him as well. It 46 that among other things, he was an eighty-year-old writer. He was ___47___ a published author, and he was currently 48 on a new book. We would see each other at various times and 49 friends. Sometimes he wore a duck hat, and I would tell myself, he was definitely a(n) 50 of Dr. Bernie. He really put a ___51___ on my face.He unfortunately 52 last year due to his cancer, 53 he left a deep impression on me and gave me the 54 to pick up my pen again. I 55 to myself, “If he can do it, then so can I.” 36. A. tastes B. ideas C. notes D. memories 37. A. amazing B. shocking C. amusing D. strange 38. A. strike B. push C. challenge D. impact 39. A. learn from B. go over C. get through D. refer to 40. A. reader B. writer C. editor D. doctor 41. A. positive B. agreeable C. humorous D. honest 42. A. mood B. position C. state D. way 43. A. advice B. reference C. protection D. treatment 44. A. viewed B. knew C. noticed D. wondered 45. A. while B. because C. although D. providing 46. A. came out B. worked out C. proved out D. turned out 47. A. naturally B. merely C. hopefully D. actually 48. A. deciding B. investing C. working D. relying 49. A. became B. helped C. missed D. visited 50. A. patient B. operator C. fan D. publisher 51. A. sign B. smile C. mark D. mask 52. A. showed up B. set off C. fell down D. passed away 53. A. since B. but C. so D. for 54. A. guidance B. trust C. opportunity D. inspiration 55. A. promised B. swore C. thought D. replied 第三部分: 阅读理解 (共 15 小题; 每小题 2 分, 满分 30 分) 请阅读下列短文, 从短文后各题所给的 A、B、C、D 四个选项中, 选出最佳选项, 并在答 题卡上将该项涂黑. A 56. According to the Code, visitors should act __________. A. with care and respect B. with relief and pleasure C. with caution and calmness D. with attention and observation 57. What are you encouraged to do when travelling in New Zealand? A. Take your own camping facilities. B. Bury glass far away from rivers. C. Follow the track for the sake of plants. D. Observe signs to approach nesting birds. B In the United States alone, over 100 million cell-phones are thrown away each year. Cell-phones are part of a growing mountain of electronic waste like computers and personal digital assistants. The electronic waste stream is increasing three times faster than traditional garbage as a whole. Electronic devices contain valuable metals such as gold and silver. A Swiss study reported that while the weight of electronic goods represented by precious metals was relatively small in comparison to total waste, the concentration (含量 ) of gold and other precious metals was higher in So-called e-waste than in naturally occurring minerals. Electronic wastes also contain many poisonous metals. Even when the machines are recycled and the harmful metals removed, the recycling process often is carried out in poor countries, in practically uncontrolled ways which allow many poisonous substances to escape into the environment. Creating products out of raw materials creates much more waste material, up to 100 times more, than the material contained in the finished products. Consider again the cell-phone, and imagine the mines that produced those metals, the factories needed to make the box and packaging(包装) it came in. Many wastes produced in the producing process are harmful as well. The U.S. Environmental Protection Agency notes that most waste is dangerous in that “the production, distribution, and use of products — as well as management of the resulting waste — all result in greenhouse gas release.” Individuals can reduce their contribution by creating less waste at the start — for instance, buying reusable products and recycling. In many countries the concept of extended producer responsibility is being considered or has been put in place as an incentive (动机 ) for reducing waste. If producers are required to take back packaging they use to sell their products, would they reduce the packaging in the first place? Governments’ incentive to require producers to take responsibility for the packaging they produce is usually based on money. Why, they ask, should cities or towns be responsible for paying to deal with the bubble wrap (气泡垫) that encased your television? From the governments’ point of view, a primary goal of laws requiring extended producer responsibility is to transfer both the costs and the physical responsibility of waste management from the government and tax-payers back to the producers. 58. By mentioning the Swiss study, the author intends to tell us that __________. A. the weight of e-goods is rather small B. E-waste deserves to be made good use of C. natural minerals contain more precious metals D. the percentage of precious metals is heavy in e-waste 59. The responsibility of e-waste treatment should be extended __________. A. from producers to governments B. from governments to producers C. from individuals to distributors D. from distributors to governments 60. What does the passage mainly talk about? A. The increase in e-waste. B. The creation of e-waste. C. The seriousness of e-waste. D. The management of e-waste. C Suppose you become a leader in an organization. It’s very likely that you’ll want to have volunteers to help with the organization’s activities. To do so, it should help to understand why people undertake volunteer work and what keeps their interest in the work. Let’s begin with the question of why people volunteer. Researchers have identified several factors that motivate people to get involved. For example, people volunteer to express personal values related to unselfishness, to expand their range of experiences, and to strengthen social relationships. If volunteer positions do not meet these needs, people may not wish to participate. To select volunteers, you may need to understand the motivations of the people you wish to attract. People also volunteer because they are required to do so. To increase levels of community service, some schools have launched compulsory volunteer programs. Unfortunately, these programs can shift people’s wish of participation from an internal factor (e.g., “I volunteer because it’s important to me”) to an external factor (e.g., “I volunteer because I’m required to do so”). When that happens, people become less likely to volunteer in the future. People must be sensitive to this possibility when they make volunteer activities a must. Once people begin to volunteer, what leads them to remain in their positions over time? To answer this question, researchers have conducted follow-up studies in which they track volunteers over time. For instance, one study followed 238 volunteers in Florida over a year. One of the most important factors that influenced their satisfaction as volunteers was the amount of suffering they experienced in their volunteer positions. Although this result may not surprise you, it leads to important practical advice. The researchers note that attention should be given to “training methods that would prepare volunteers for troublesome situations or provide them with strategies for coping with the problem they do experience”. Another study of 302 volunteers at hospitals in Chicago focused on individual differences in the degree to which people view “volunteer” as an important social role. It was assumed that those people for whom the role of volunteer was most part of their personal identity would also be most likely to continue volunteer work. Participants indicated the degree to which the social role mattered by responding to statements such as “Volunteering in Hospital is an important part of who I am.”Consistent with the researchers’ expectations, they found a positive correlation (正相关 ) between the strength of role identity and the length of time people continued to volunteer. These results, once again, lead to concrete advice: “Once an individual begins volunteering, continued efforts might focus on developing a volunteer role identity.... Items like T-shirts that allow volunteers to be recognized publicly for their contributions can help strengthen role identity”. 61. People volunteer mainly out of __________. A. academic requirements B. social expectations C. financial rewards D. internal needs 62. What can we learn from the Florida study? A. Follow-up studies should last for one year. B. Volunteers should get mentally prepared. C. Strategy training is a must in research. D. Volunteers are provided with concrete advice. 63. What is most likely to motivate volunteers to continue their work? A. Individual differences in role identity. B. Publicly identifiable volunteer T-shirts. C. Role identity as a volunteer. D. Practical advice from researchers. 64. What is the best title of the passage? A. How to Get People to Volunteer B. How to Study Volunteer Behaviors C. How to Keep Volunteers’ Interest D. How to Organize Volunteer Activities D Freedom and Responsibility Freedom’s challenge in the Digital Age is a serious topic. We are facing today a strange new world and we are all wondering what we are going to do with it. Some 2,500 years ago Greece discovered freedom. Before that there was no freedom. There were great civilizations, splendid empires, but no freedom anywhere. Egypt and Babylon were both tyrannies, one very powerful man ruling over helpless masses. In Greece, in Athens (雅典), a little city in a little country, there were no helpless masses. And Athenians willingly obeyed the written laws which they themselves passed, and the unwritten, which must be obeyed if free men live together. They must show each other kindness and pity and the many qualities without which life would be very painful unless one chose to live alone in the desert.The Athenians never thought that a man was free if he could do what he wanted. A man was free if he was self-controlled. To make yourself obey what you approved was freedom. They were saved from looking at their lives as their own private affair. Each one felt responsible for the welfare of Athens, not because it was forced on him from the outside, but because the city was his pride and his safety. The essential belief of the first free government in the world was liberty for all men who could control themselves and would take responsibility for the state. But discovering freedom is not like discovering computers. It cannot be discovered once for all. If people do not prize it, and work for it, it will go. Constant watch is its price. Athens changed. It was a change that took place without being noticed though it was of the extreme importance, a spiritual change which affected the whole state. It had been the Athenian 爷 s pride and joy to give to their city. That they could get material benefits from her never entered their minds. There had to be a complete change of attitude before they could look at the city as an employer who paid her citizens for doing her work. Now instead of men giving to the state, the state was to give to them.What the people wanted was a government which would provide a comfortable life for them; and with this as the primary object, ideas of freedom and self-reliance and responsibility were neglected to the point of disappearing. Athens was more and more looked on as a cooperative business possessed of great wealth in which all citizens had a right to share. Athens reached the point when the freedom she really wanted was freedom from responsibility.There could be only one result. If men insisted on being free from the burden of self-dependence and responsibility for the common good, they would cease to be free. Responsibility is the price every man must pay for freedom. It is to be had on no other terms. Athens, the Athens of Ancient Greece, refused responsibility; she reached the end of freedom and was never to have it again. But, “the excellent becomes the permanent”, Aristotle said. Athens lost freedom forever, but freedom was not lost forever for the world. A great American, James Madison, referred to: “The capacity (能力 ) of mankind for self-government.” No doubt he had not an idea that he was speaking Greek. Athens was not in the farthest background of his mind, but once man has a great and good idea, it is never completely lost. The Digital Age cannot destroy it. Somehow in this or that man’s thought such an idea lives though unconsidered by the world of action. One can never be sure that it is not on the point of breaking out into action only sure that it will do so sometime. 65. What does the underlined word “tyrannies” in Paragraph 2 refer to? A. Countries where their people need help. B. Powerful states with higher civilization. C. Splendid empires where people enjoy freedom. D. Governments ruled with absolute power. 66. People believing in freedom are those who __________. A. regard their life as their own business B. seek gains as their primary object C. behave within the laws and value systems D. treat others with kindness and pity 67. What change in attitude took place in Athens? A. The Athenians refused to take their responsibility. B. The Athenians no longer took pride in the city. C. The Athenians benefited spiritually from the government. D. The Athenians looked on the government as a business. 68. What does the sentence “There could be only one result.” in Paragraph 5 mean? A. Athens would continue to be free. B. Athens would cease to have freedom. C. Freedom would come from responsibility. D. Freedom would stop Athens from self-dependence. 69. Why does the author refer to Aristotle and Madison? A. The author is hopeful about freedom. B. The author is cautious about self-government. C. The author is skeptical of Greek civilization. D. The author is proud of man’s capacity. 70. What is the author’s understanding of freedom? A. Freedom can be more popular in the digital age. B. Freedom may come to an end in the digital age. C. Freedom should have priority over responsibility. D. Freedom needs to be guaranteed by responsibility. 第四部分: 任务型阅读 (共 10 小题; 每小题 1 分, 满分 10 分) 请阅读下面短文,并根据所读内容在文章后表格中的空格里填入一个最恰当的单词。 注意: 请将答案写在答题卡上相应题号的横线上。 每个空格只填一个单词。 People select news in expectation of a reward. This reward may be either of two kinds. One is related to what Freud calls the Pleasure Principle, the other to what he calls the Reality Principle. For want of better names, we shall call these two classes immediate reward and delayed reward. In general, the kind of news which may be expected to give immediate reward are news of crime and corruption, accidents and disasters, sports, social events, and human interest. Delayed reward may be expected from news of public affairs, economic matters, social problems, science, education, and health. News of the first kind pays its rewards at once. A reader can enjoy an indirect experience without any of the dangers or stresses involved. He can tremble wildly at an axe-murder, shake his head sympathetically and safely at a hurricane, identify himself with the winning team, laugh understandingly at a warm little story of children or dogs. News of the second kind, however, pays its rewards later. It sometimes requires the reader to tolerate unpleasantness or annoyance — as, for example, when he reads of the threatening foreign situation, the mounting national debt, rising taxes, falling market, scarce housing, and cancer. It has a kind of “threat value.” It is read so that the reader may be informed and prepared. When a reader selects delayed reward news, he pulls himself into the world of surrounding reality to which he can adapt himself only by hard work. When he selects news of the other kind, he usually withdraws from the world of threatening reality toward the dream world. For any individual, of course, the boundaries of these two classes are not stable. For example, a sociologist may read news of crime as a social problem, rather than for its immediate reward. A coach may read a sports story for its threat value: he may have to play that team next week. A politician may read an account of his latest successful public meeting, not for its delayed reward, but very much as his wife reads an account of a party. In any given story of corruption or disaster, a thoughtful reader may receive not only the immediate reward of indirect experience, but also the delayed reward of information and preparedness. Therefore, while the division of categories holds in general, an individual’s tendency may transfer any story from one kind of reading to another, or divide the experience between the two kinds of reward. What news stories do you read? Division of news stories  People expect to get (71) ▲ from reading news.  News stories are roughly divided into two classes.  Some news will excite their readers instantly while others won’t. (72) ▲ of the two classes  News of immediate reward will seemingly take their readers to the very frightening scene without actual (73) ▲ .  Readers will associate themselves closely with what happens in the news stories and (74) ▲ similar feelings with those involved.  News of delayed reward will make readers suffer, or present a (75) ___▲___ to them.  News of delayed reward will induce the reader to (76) ▲ for the reality while news of immediate reward will lead the reader to (77) ___▲___ from the reality. Unstable boundaries of the two classes What readers expect from news stories are largely shaped by their (78) ___▲ . Serious readers will both get excited over what happens in some news stories and (79) ▲ themselves to the reality. Thus, the division, on the whole, (80) ▲ on the reader. 71. __________ 72. __________ 73. __________ 74. __________ 75. __________ 76. __________ 77. __________ 78. __________ 79. __________ 80. __________ 第五部分: 书面表达 (满分 25 分) 81.请阅读下面文字及图表,并按照要求用英语写一篇 150 词左右的文章. [写作内容] 1. 用约 30 个单词概述上述信息的主要内容; 2. 结合上述信息,简要分析导致交通问题的主要原因; 3. 根据你的分析,从社会规范(rules and regulations)和个人行为两方面谈谈你得到的启示(不 少于两点)。 [写作要求] 1. 写作过程中不能直接引用原文语句;摇 2. 作文中不能出现真实姓名和学校名称; 3. 不必写标题。 [评分标准] 内容完整,语言规范,语篇连贯,词数适当。 2015年普通高等学校招生全国统一考试(安徽卷) 英 语 本试卷分第Ⅰ卷(选择题)和第Ⅱ卷(非选择题)两部分。全卷满分 150分,考试时间 120分钟。 考生注意事项: 1.答题前,务必在试题卷、答题卡规定的地方填写自己的姓名、座位号,并认真核对答题卡 上所粘贴的条形码中姓名、座位号与本人姓名、座位号是否一致。务必在答题卡背面规定的地方 填写姓名和座位号后两位。 2.答第Ⅰ卷时,每小题选出答案后,用 2B铅笔把答题卡上对应题目的答案标号涂黑。如需 改动,用橡皮擦干净后,再选涂其他答案标号。 3.答第Ⅱ卷时,必须使用 0.5毫米的黑色墨水签字笔在答题卡上书写,要求字体工整、笔 迹清晰。作图题可先用铅笔在答题卡规定的位置绘出,确认后再用 0.5毫米的黑色墨水签字笔描 清楚。必须在题号所指示的答题区域作答,超出答题区域书写的答案无效,在试题卷、草稿纸上....................... 答题无效....。 4.考试结束,务必将试题卷和答题卡一并上交。 科网 第Ⅰ卷 第一部分 听力(共两节,满分 30 分) 做题时,请先将答案标在试卷上。录音内容结束后,你将有两分钟的时间将试卷上的答案转 涂到答题卡上。 第一节 (共 5小题;每小题 1.5分,满分 7.5分) 听下面 5段对话。每段对话后有一个小题,从题中所给的 A、B、C三个选项中选出最佳选项, 并标在试卷的相应位置。听完每段对话后;你都有 10秒钟的时间来回答有关小题和阅读下一小题。 每段对话仅读一遍。 例:How much is the shirt? A.£l9.15. B.£9.15 C.£9.18. 答案是 C。 1. What time is it now? A. 9:10. B. 9:50. C. 10:00. 2. What does the woman think of the weather? A. It's nice. B. It's warm. C. It's cold. 3. What will the man do? A. Attend a meeting. B. Give a lecture. C. Leave his office. 4. What is the woman's opinion about the course? A. Too hard. B. Worth taking. C. Very easy. 5. What does the woman want the man to do? A. Speak louder. B. Apologize to her. C. Turn off the radio. 第二节 (共 15小题;每小题 1. 5分,满分 22.5分) 听下面 5段对话。每段对话后有几个小题,从题中所给的 A、B、C三个选项中选出最佳选项, 并标在试卷的相应位置。听每段对话前,你将有时间阅读各个小题,每小题 5秒钟;听完后,各 小题给出 5秒钟的作答时间。每段对话读两遍。 听第 6段材料,回答第 6、7题。 6. How long did Michael stay in China? A. Five days. B. One week. C. Two weeks. 7. Where did Michael go last year? A. Russia. B. Norway. C. India. 听第 7段材料,回答第 8、9题。 8. What food does Sally like? A. Chicken B. Fish C. Eggs 9. What air the speakers going to do? A. Cook dinner. B. Go shopping. C. Order dishes. 听第 8段材料,回答第 10至 12题。 10. Where are the speakers? A. In a hospital. B. In the office. C. At home. 11. When is the report due? A. Thursday. B. Friday. C. Next Monday. 12. What does George suggest Stephanie do with the report? A. Improve it. B. Hand it in later. C. Leave it with him. 听第 9段材料,回答第 13至 16题。 13. What is the probable relationship between the speakers? A. Salesperson and customer. B. Homeowner and cleaner. C. Husband and wife. 14. What kind of department do the speakers prefer? A. One with two bedrooms. B. One without furniture C. One near a market. 15. How much rent should one pay for the one-bedroom apartment? A. $350. B. $400. C. $415. 16. Where is the apartment the speakers would like to see? A. On Lake Street. B. On Market Street. C. On South Street, 听第 10段材料,回答第 17至 20题。 17. What percentage of the world's tea exports go to Britain? A. Almost 15%. B. About 30%. C. Over 40%. 18. Why do tea tasters taste tea with milk? A. Most British people drink tea that way. B. Tea tastes much better with milk. C. Tea with milk is healthy. 19. Who suggests a price for each tea? A. Tea tasters. B. Tea exporters. C. Tea companies. 20. What is the speaker talking about? A. The life of tea tasters. B. Afternoon tea in Britain. C. The London Tea Trade Centre. 第二部分 英语知识运用(共两节,满分 45 分) 第一节 单项填空(共 15小题;每小题 1分,满分 15分) 从 A、B、C、D四个选项中,选出可以填入空白处的最佳选项,并在答题卡上将该项涂黑。 例: It is generally considered unwise to give a child ______ he or she wants. A. however B. whatever C. whichever D. whenever 答案是 B 21. — Can you come to a party on Saturday, Peter? — Oh, __________ I'm already going out, I'm afraid. A. what a pity! B. don't ask! C. how come? D. so what? 22. If you come to visit China, you will __________ a culture of amazing depth and variety. A. develop B. create C. substitute D. experience 23. __________ scientists haw learned a lot about the universe, there is much we still don't know. A. Once B. Since C. Though D. Unless 24. Just as I got to the school gate, I realized I __________ my bank in the cafe. A. have left B. had left C. would leave D. was leaving 25 A ship in harbor is safe, but that’s not __________ ships are built for. A. what B. whom C. why D. when 26. I’m so __________ to all those volunteers because they helped my terrible day end happily. A. special B. superior C. grateful D. attractive 27. __________ the difference between the two research findings will be one of the worst mistakes you make. A. Ignore B. Ignoring C. Ignored D. Having ignored 28. Some experts think reading is the fundamental skill upon __________ school education depends. A. it B. that C. whose D. which 29. It is reported that a space station __________ on the moon in years to come. A. will be building B. will he built C. has been building D. has been built 30. There is no need to tell me your answer now. Give it some __________ and then let me know. A. thought B. support C. protection D. authority 31. They gave money to the old people's home either __________ or through their companies. A. legally B. sincerely C. personally D. deliberately 32. It is lucky we booked a room, or we __________ nowhere to stay now. A. had B. had had C. would have D. would have had 33. They believe that there are transport developments __________ that will bring a lot of changes for the better, A. out of date B. out of order C. around the clock D. around the corner 34. __________ he once felt like giving up, he now has the determination to push further and keep on going. A. Where B. As C. In case D. Now that 35. — How is your table tennis these days? Still playing? —__________. I just don't seem to find the time these days. A. That's right B. No, not much C That's great D. Don’t worry 第二节 完形填空(共 20小题;每小题 1.5分,满分 30分) 阅读下面短文,从短文后各题所给的四个选项(A、B、C、和 D)中,选出可以填入空白处 的最佳选项,并在答题卡上将该项涂黑。 In our modern world, when something wears out, we throw it away and buy a new one. The ___36___ is that countries around the world have growing mountains of 37 because people are throwing out more rubbish than ever before. How did we 38 a throwaway society? First of all, it is now easier to 39 an object than to spend time and money to repair it. 40 modern manufacturing (制造业 ) and technology, companies are able to produce products quickly and inexpensively. Products are plentiful and 41 . Another cause is our 42 of disposable (一次性的) products. As 43 people, we are always looking for 44 to save time and make our lives easier. Companies ___45___ thousands of different kinds of disposable products: paper plates, plastic cups, and cameras, to name a few. Our appetite for new products also 46 to the problem. We are 47 buying new things. Advertisements persuade us that 48 is better and that we will be happier with the latest products. The result is that we 49 useful possessions to make room for new ones. All around the world, we can see the 50 of this throwaway lifestyle. Mountains of rubbish just keep getting bigger. To 51 the amount of rubbish and to protect the 52 , more governments are requiring people to recycle materials. 53 , this is not enough to solve (解 决) our problem. Maybe there is another way out. We need to repair our possessions 54 throwing them away. We also need to rethink our attitudes about 55 . Repairing our possessions and changing our spending habits may be the best way to reduce the amount of rubbish and take care of our environment. 36. A. key B. reason C. project D. problem 37. A. gifts B. rubbish C. debt D. products 38. A. face B. become C. observe D. change 39. A. hide B. control C. replace D. withdraw 40. A. Thanks to B. As to C. Except for D. Regardless of 41. A. safe B. funny C. cheap D. powerful 42. A. love B. lack C. prevention D. division 43. A. sensitive B. kind C. brave D. busy 44. A. ways B. places C. jobs D. friends 45. A. donate B. receive C. produce D. preserve 46. A. adapts B. returns C. responds D. contributes 47. A. tired of B. addicted to C. worried about D. ashamed for 48. A. newer B. stronger C. higher D. larger 49. A. pick up B. pay for C. hold onto D. throw away 50. A. advantages B. purposes C. functions D. consequences 51. A. show B. record C. decrease D. measure 52. A. technology B. environment C. consumers D. brands 53. A. However B. Otherwise C. Therefore D. Meanwhile 54. A. by B. in favour of C. after D. instead of 55. A. spending B. collecting C. repairing D. advertising 第三部分 阅读理解(共 20小题;每小题 2分,满分 40分) 阅读下列短文,从每题所给的四个选项(A、B、C和 D)中,选出最佳选项,并在答 题卡上将该项涂黑。 A Welcome to the Electronic Village to explore new ways of language teaching and learning. Electronic Village Program (Thursday, June 18, 2015) Nearpod ❖ 9:00 am to 10:00 am ❖ Room 501 Nearpod is a software program that creates a rich context ( 语 境 ) for students to learn vocabulary. The presenter will show how to use it. TEO ❖ 2:00 pin to 3:00 pm ❖ Room 502 Our students come from different backgrounds but have the same desire to learn on-line. The presenter will use examples from his first on-line class to explain how any teacher can begin teaching on-line with TEO. Kahoot ❖ 10:30 am to 11:30 am ❖ Room 601 Kahoot software can be used to create grammar tests which can be graded on a network. It can provide students with instant feedback (反 馈 ), including reports about their strengths and weaknesses. Prezi ❖ 3:30 pm to 4:20 pm ❖ Room 602 Uses of Prezi in listening and speaking courses draw students' attention to speaking more fluently. The presenter will show how students can use Prezi to confidently present on a variety of topics, including introducing family, friends, and hobbies. 56. Nearpod can be used to __________. A. offer grammar tests B. teach listening on-line C. help vocabulary learning D. gain fluency in speaking 57. If you want to improve your speaking skills, you can go to __________. A. Room 501 B. Room 502 C. Room 601 D. Room 602 58. Which of the following can assess your grammar learning? A. Nearpod. B. Kahoot. C. TEO. D. Prezi. 59. A teacher who wants to learn on-line teaching ia expected to arrive by __________. A. 9:00 am B. 10:30 am C. 2:00 pm D. 3:30 pm B When her five daughters were young, Helene An always told them that there was strength in unity (团结). To show this, she held up one chopstick, representing one person. Then she easily broke it into two pieces. Next, she tied several chopsticks together, representing a family. She showed the girls it was hard to break the tied chopsticks. This lesson about family unity stayed with the daughters as they grew up. Helene An and her family own a large restaurant business in California. However, when Helene and her husband Danny left their home in Vietnam in 1975, they didn't have much money. They moved their family to San Francisco. There they joined Danny's mother, Diana, who owned a small Italian sandwich shop. Soon afterwards, Helene and Diana changed the sandwich shop into a small Vietnamese restaurant. The five daughters helped in the restaurant when they were young. However, Helene did not want her daughters to always work in the family business because she thought it was too hard. Eventually the girls all graduated from college and went away to work for themselves, but one by one, the daughters returned to work in the family business. They opened new restaurants in San Francisco and Los Angeles. Even though family members sometimes disagreed with each other, they worked together to make the business successful. Daughter Elisabeth explains, "Our mother taught us that to succeed we must have unity, and to have unity we must have peace. Without the strength of the family, there is no business." Their expanding business became a large corporation in 1996, with three generations of Ans working together. Now the Ans' corporation makes more than $20 million each year. Although they began with a small restaurant, they had big dreams, and they worked together. Now they are a big success. 60. Helene tied several chopsticks together to show __________. A. the strength of family unity B. the difficulty of growing up C. the advantage of chopsticks D. the best way of giving a lesson 61. We can I earn from Paragraph 2 that the An family __________. A. started a business in 1975 B. left Vietnam without much money C. bought a restaurant in San Francisco D. opened a sandwich shop in Los Angeles 62. What can we infer about the An daughters? A. They did not finish their college education. B. They could not bear to work in the family business. C. They were influenced by what Helene taught them. D. They were troubled by disagreement among family members. 63. Which of the following can be the best title for the passage? A. How to Run a Corporation B. Strength Comes from Peace C. How to Achieve a Big Dream D. Family Unity Builds Success C As Internet users become more dependent on the Internet to store information, are people remember less? If you know your computer will save information, why store it in your own personal memory, your brain? Experts are wondering if the Internet is changing what we remember and how. In a recent study, Professor Betsy Sparrow conducted some experiments. She and her research team wanted to know the Internet is changing memory. In the first experiment, they gave people 40 unimportant facts to type into a computer. The first group of people understood that the computer would save the information. The second group understood that the computer would not save it. Later, the second group remembered the information better. People in the first group knew they could find the information again, so they did not try to remember it. In another experiment, the researchers gave people facts to remember, and told them where to find the information on the Internet. The information was in a specific computer folder (文件夹 ). Surprisingly, people later remember the folder location (位置) better than the facts. When people use the Internet, they do not remember the information. Rather, they remember how to find it. This is called "transactive memory (交互记忆)" According to Sparrow, we are not becoming people with poor memories as a result of the Internet. Instead, computer users are developing stronger transactive memories; that is, people are learning how to organize huge quantities of information so that they are able to access it at a later date. This doesn't mean we are becoming either more or less intelligent, but there is no doubt that the way we use memory is changing. 64. The passage begins with two questions to __________. A. introduce the main topic B. show the author's altitude C. describe how to use the Internet. D. explain how to store information 65. What can we learn about the first experiment? A. Sparrow's team typed the information into a computer. B. The two groups remembered the information equally well. C. The first group did not try to remember the formation. D. The second group did not understand the information. 66. In transactive memory, people __________. A. keep the information in mind B. change the quantity of information C. organize information like a computer D. remember how to find the information 67. What is the effect of the Internet according to Sparrow's research? A. We are using memory differently. B. We are becoming more intelligent. C. We have poorer memories than before. D. We need a better way to access information. D There are an extremely large number of ants worldwide. Each individual (个体的) ant hardly weigh anything, but put together they weigh roughly the same as all of mankind. They also live nearly everywhere, except on frozen mountain tops and around the poles. For animals their size, ants have been astonishingly successful, largely due to their wonderful social behavior. In colonies (群体) that range in size from a few hundred to tens of millions, they organize their lives with a clear division of labor. Even more amazing is how they achieve this level of organization. Where we use sound and sight to communicate, ants depend primarily on pheromone (外激素 ), chemicals sent out by individuals and smelled or tasted by fellow members of their colony. When an ant finds food, it produces a pheromone that will lead others straight to where the food is. When an individual ant comes under attack or is dying, it sends out an alarm pheromone to warn the colony to prepare for a conflict as a defense unit. In fact, when it comes to the art of war, ants have no equal. They are completely fearless and will readily take on a creature much larger than themselves, attacking in large groups and overcoming their target. Such is their devotion to the common good of the colony that not only soldier ants but also worker ants will sacrifice their lives to help defeat an enemy. Behaving in this selfless and devoted manner, these little creatures have survived on Earth, for more than 140 million years, far longer than dinosaurs. Because they think as one, they have a collective (集体的) intelligence greater than you would expect from its individual parts. 68. We can learn from the passage that ants are __________. A. not willing to share food B. not found around the poles C. more successful than all other animals D. too many to achieve any level of organization 69. Ants can use pheromones for __________. A. escape B. communication C. warning enemies D. arranging labor 70. What does the underlined expression "take on" in Paragraph 3 mean? A. Accept. B. Employ. C. Play with. D. Fight against. 71. Which of the following contributes most to the survival of ants? A. Their behavior. B. Their size. C. Their number. D. Their weight- E Food serves as a form of communication in two fundamental ways. Sharing bread or other foods is a common human tradition that can promote unity and trust. Food can also have a specific meaning, and play a significant role in a family or culture's celebrations or traditions. The foods we eat—and when and how we eat them—are often unique to a particular culture or may even differ between rural (农村的) and urban areas within one country. Sharing bread, whether during a special occasion (时刻 ) or at the family dinner table, is a common symbol of togetherness. Many cultures also celebrate birthdays and marriages with cakes that are cut and shared among the guests. Early forms of cake were simply a kind of bread, so this tradition hits its roots in the custom of sharing bread. Food also plays an important role in many New Year celebrations. In the southern United States, pieces of corn bread represent blocks of gold for prosperity (兴旺) in the New Year. In Greece, people share a special cake called vasilopita. A coin is put into the cake, which signifies (预示) success in the New Year for the person who receives it. Many cultures have ceremonies to celebrate the birth of a child, and food can play a significant role. In China, when a baby is one month old, families name and welcome their child in a celebration that includes giving red-colored eggs to guests. In many cultures, round foods such as grapes, bread, and moon cakes are eaten at welcome celebrations to represent family unity. Nutrition is necessary for life, so it is not surprising that food is such an important part of different cultures around the world. 72. According to the passage, sharing bread __________. A. indicates a lack of food B. can help to develop unity C. is a custom unique to rural areas D. has its roots in birthday celebrations 73. What does the coin in vasilopita signify for its receiver in the New Year? A. Trust. B. Success. C. Health. D. Togetherness. 74. The author explains the role of food in celebrations by __________. A. using examples B. making comparisons C. analyzing causes D. describing processes 75. What is the passage mainly about? A. The custom of sharing food. B. The specific meaning of food. C. The role of food in ceremonies. D. The importance of food in culture. 第Ⅱ卷 考生注意事项: 请用 0.5毫米黑色墨水签字笔在答题卡上作答,在试题卷上答题无效。 第四部分 写作 (共两节,满分 35 分) 第一节 任务型读写 (共 10小题;每小题 1分,满分 10分) 阅读下面短文,根据所读内容在表格中的空白处填入恰当的单词。注意:每个空格只填一个 单词。 Pup a group of strangers in a room together, and they'll probably start a conversation. "Hot today, isn't it?" one might say. "You said it." another replies. Why do we talk so much about the weather? When we meet new people, we don't begin by telling them our life story. We start with small talk, a polite conversation about something like traffic or weather. Research suggests that small talk can build new friendships. When we begin conversations with new people, we want to feel comfortable, and so do they. We use small talk to find common interests. Once we have a common interest, a friendship can begin. Small talk even helps people get hired. In order to impress at a job interview, you need to bond with the interviewer right away. Proper small talk can make that first impression get you the job. So, how can you make small talk lead to a new friendship or job? First off, find common ground. Select something around you that you share with the other person. Next, keep the conversation going. Compliment (赞美) the other person to make him or her feel comfortable, and ask questions to show interest. Third, keep eye contact (接触). When you look people in the eye, they feel you appreciate what they are saying. It makes you appear honest and builds trust. Naturally, shy people might not have enough confidence to start up conversations with strangers. Talking to someone you don't know is not the easiest thing to do! Some experts say with more practice, small talk does get easier. Some people avoid small talk because they dislike discussing things like traffic or weather. For them, they are just too small. However, when you think about it, small talk is anything but small. In fact, it is actually a very big deal! Title Small Talk: A Big (76)__________ Introduction We are likely to make small talk when we (77)__________ meet people. (78)__________ ❖ Small talk can help people form (79)__________ friendships. ❖ Small talk can also help people get a (80)__________. Advice ❖ Find some topics (81)__________ with the other person. ❖ Keep the talk going by making compliments and (82)__________ questions. ❖ Keep eye contact in conversation to build (83)__________. ❖ (84)__________ more in order to make small talk easier. Conclusion Small talk really (85)__________ a lot to us. 第二节 书面表达(满分 25分) 某英文杂志正在举办以 "Fancy yourself as an interviewer" 为主题的征文活动,请你 以“A Famous Chinese I Would Like to Interview" 为题, 写一篇英语短文。 内容包括: 1. 采访的对象; 2. 采访的原因; 3. 想提的问题。 注意: 1. 词数 120左右; 2. 可以适当增加细节,以使行文连贯; 3. 短文中不能出现与本人相关的信息; 4. 短文的标题已给出,不计人总词数。 A Famous Chinese I Would Like to Interview ____________________________________________________________________________________ ____________________________________________________________________________________ _______________ 2015年普通高等学校招生全国统一考试(浙江卷) 英 语 试 题 本试卷分选择题和非选择题部分。全卷满分 120分,考试时间 120分钟 请考生按规定用笔将所有试题的答案涂、写在答题纸上。 注意事项: 1. 答题前,考生务必将自己的姓名、准考证号用黑色笔迹的签字笔或钢笔分别填写在试卷和答题 纸规定的位置上。 2. 每小题选出答案后,用 2B铅笔把答题纸上对应的题目答案标号涂黑,如需改动,用橡皮檫干 净后,再选涂其他答案标号。不能答在试题卷上。 选择题部分(共 80分) 第一部分:英语知识应用(共两节,满分 30分) 第一节:单项填空(共 20小题,每小题 0.5分,满分 10分) 从 A、B、C 和 D四个选项中,选出可以填入空白处的最佳选项,并在答题纸上将该选项标 号涂黑。 1. ---Hi, John. Are you busy?. --- __________ A. Yes. I do agree. B. Yes. That would be nice C. No. Are you sure? D. No. What’s up? 2. Jane’s grandmother had wanted to write __________ children’s book for many years, but one thing or another always got in __________ way. A. a; 不填 B. the; the C. 不填; the D. a; the 3. Have you ever heard of the trees that are homes __________ animals both on land and sea?. A. about B. to C.with D. over 4. It was so noisy that we __________ hear ourselves speak. A. couldn’t B. shouldn’t C.mustn’t D. needn’t 5. Studies have shown that the right and left ear __________ sound differently. A. produce B. pronounce C. process D. download 6. If you swim in a river or lake, be sure to investigate __________ is below the water surface. Often there are rocks and branches hidden in the water. A. what B. who C. that D. whoever 7. Body language can __________ a lot about your mood, so standing with your arms folded can send out a signal that you are being defensive. A. take away B. throw away C. put away D. give away 8. Albert Einstein was born in 1879. As a child, few people guessed that he __________ a famous scientist whose theories would change the world. A. has been B. had been C. was going to be D. was 9. __________ a single word can change the meaning of a sentence, a single sentence can change the meaning of a paragraph. A. Just as B. In addition C. Until D. Unless 10. Most people work because it’s unavoidable. __________, there are some people who actually enjoy work. A. As a result B. Even though C. By contrast D. In conclusion 11. We tend to have a better memory for things that excite our senses or __________ our emotions than for straight facts. A. block off B. appeal to C. subscribe to D. come across 12. How would you like __________ if you were watching your favorite TV program and someone came into the room and just shut it off without asking you?. A. them B. one C. those D. it 13. Most of us, if we know even a little about where our food comes from, understand that every bite put into our mouths was __________ alive. A. steadily B. instantly C. formerly D. permanently 14. Listening is thus an active, not a __________, behavior consisting of hearing, understanding and remembering.. A. considerate B. sensitive C. reliable D. passive 15. One of the most effective ways to reduce __________ is to talk about feelings with someone you trust. A. production B. stress C. energy D. power 16. If steel is heavier than water, why are ships able to ___________ on the sea? A. float B. drown C. shrink D. split 17. These comments came __________ special questions often asked by local newsmen. A. in memory of B. in response to C. in touch with D. in possession of 18. Listening to music at home is one thing, going to hear it __________ live is quite another. A. perform B. performing C. to perform D. being performed 19. Creating an atmosphere __________ employees feel part of a team is a big challenge. A. as B. whose C. in which D. at which 20. —Why don’t you consider a trip to, say, Beijing or Hangzhou? —__________. A. I wouldn’t mind that B. Then we’ll get there quickly C. Let’s call it a day D. It’s not a requirement 第二节:完形填空(共 20小题;每小题 1分,满分 20分) 阅读下面短文,掌握其大意,然后从 21—40各题所给的四个选项中(A、B、C和 D)中, 选出最佳选项,并在答题纸上将该选项标号涂黑。 Since finishing my studies at Harvard and Oxford, I’ve watched one friend after another land high-ranking, high-paying Wall Street jobs. As executives(高级管理人员) with banks, consulting firms, established law firms, and major corporations, many are now 21 on their way to impressive careers. By society’s 22 , they seem to have it made. On the surface, these people seem to be very lucky in life. As they left student life behind, many had a 23 drink at their cheap but friendly local bar, shook hands with longtime roommates, and 24 out of small apartments into high buildings. They made reservations at restaurants where the cost of a bottle of wine 25 a college year’s monthly rent. They replaced their beloved old cars with expensive new sports cars. The thing is, a number of them have 26 that despite their success, they aren’t happy. Some 27 of unfriendly coworkers and feel sad for eight-hour workweeks devoted to tasks they ___28___. Some do not respect the companies they work for and talk of feeling tired and 29 . However, instead of devoting themselves to their work, they find themselves working to support the 30 to which they have so quickly become 31 . People often speak of trying a more satisfying path, and 32 in the end the idea of leaving their jobs to work for something they 33 or finding a position that would give them more time with their families almost always leads them to the same conclusion: it’s 34 . They have loans, bills, a mortgage(抵押贷款 ) to 35 , retirement to save for. They recognize there’s something 36 in their lives, but it’s 37 to step off the track. In a society that tends to 38 everything in terms of dollars and cents, we learn from a young age to consider the costs of our 39 in financial terms. But what about the personal and social costs 40 in pursuing money over meaning? These are exactly the kinds of costs many of us tend to ignore---and the very ones we need to consider most. 21. A. much B. never C. seldom D. well 22. A. policies B. standards C. experiments D. regulations 23. A. last B. least C. second D. best 24. A. cycled B. moved C. slid D. looked 25. A. shared B. paid C. equaled D. collected 26. A. advertised B. witnessed C. admitted D. demanded 27. A. complain B. dream C. hear D. approve 28. A. distribute B. hate C. applaud D. neglect 29. A. calm B. guilty C. warm D. empty 30. A. family B. government C. lifestyle D. project 31. A. accustomed B. appointed C. unique D. available 32. A. yet B. also C. instead D. rather 33. A. let out B. turn in C. give up D. believe in 34. A. fundamental B. practical C. impossible D. unforgettable 35. A. take off B. drop off C. put off D. pay off 36. A. missing B. inspiring C. sinking D. shining 37. A. harmful B. hard C. useful D. normal 38. A. measure B. suffer C. digest D. deliver 39. A. disasters B. motivations C. campaigns D. decisions 40. A. assessed B. involved C. covered D. reduced 第二部分 阅读理解(第一节 20 小题,第二节 5 小题,满分 50 分) 第一节:阅读下列材料,从每题所给的四个选项(A、B、C和 D)中,选出最佳选项,并在答题 纸上将该选项标号涂黑。 A From the very beginning of school we make books and reading a constant source of possible failure and public humiliation. When children are little we make them read aloud, before the teacher and other children, so that we can be sure they “know” all the words they are reading. This means that when they don’t know a word, they are going to make a mistake, right in front of everyone. After having taught fifth-grade classes for four years, I decided to try at all costs to rid them of their fear and dislike of books, and to get them to read oftener and more adventurously. One day soon after school had started, I said to them, “Now I’m going to say something about reading that you have probably never heard a teacher say before. I would like you to read a lot of books this year, but I want you to read them only for pleasure. I am not going to ask you questions to find out whether you understand the books or not. If you understand enough of a book to enjoy it and want to go on reading it, that’s enough for me. And I’m not going to ask you what words mean.” The children sat stunned and silent. Was this a teacher talking? One girl, who had just come to us from a school where she had had a very hard time, looked at me steadily for a long time after I had finished. Then, still looking at me, she said slowly and seriously, “Mr. Holt, do you really mean that?” I said just as seriously, “I mean every word of it.” During the spring she really astonished me. One day, she was reading at her desk. From a glimpse of the illustrations I thought I knew what the book was. I said to myself, “It can’t be,” and went to take a closer look. Sure enough, she was reading Moby Dick, in the edition with woodcuts. I said, “Don’t you find parts of it rather heavy going?” She answered, “Oh, sure, but I just skip over those parts and go on to the next good part.” This is exactly what reading should be and in school so seldom is --- an exciting, joyous adventure. Find something, dive into it, take the good parts, skip the bad parts, get what you can out of it, go on to something else. How different is our mean-spirited, picky insistence that every child get every last little scrap of “understanding” that can be dug out of a book. 41. According to the passage, children’s fear and dislike of books may result from __________. A. reading little and think little. B. reading often and adventurously C. being made to read too much D. being made to read aloud before others 42. The teacher told his students to read __________. A. for enjoyment B. for knowledge C. for a larger vocabulary D. for higher scores in exams 43. Upon hearing the teacher’s talk, the children probably felt that __________. A. it sounded stupid B. it was not surprising at all C. it sounded too good to be true D. it was no different from other teachers’ talk 44. Which of the following statements about the girl is TRUE according to the passage? A. She skipped over those easy parts while reading. B. She had a hard time finishing the required reading tasks. C. She learned to appreciate some parts of the difficult books. D. She turned out to be a top student after coming to this school. 45. From the teacher’s point of view, __________. A. children cannot tell good parts from bad parts while reading B. children should be left to decide what to read and how to read C. reading is never a pleasant and inspiring experience in school D. reading involves understanding every little piece of information B Graphs can be a very useful tool for conveying information, especially numbers, percentages, and other data. A graph gives the reader a picture to interpret. That can be a lot more efficient than pages and pages explaining the data. Graphs can seem frightening, but reading a graph is a lot like reading a story. The graph has a title, a main idea, and supporting details. You can use your active reading skills to analyze and understand graphs just like any other text. Most graphs have a few basic parts: a caption or introduction paragraph, a title, a legend or key, and labeled axes. An active reader looks at each part of the graph before trying to interpret the data. Captions will usually tell you where the data from(for example, a scientific study of 400 African elephants from 1980 to 2005). Captions usually summarize the author’s main point as well. The title is N um be ro fS tu de nt s very important. It tells you the main idea of the graph by stating what kind of information is being shown. A legend, also called a key, is a guide to the symbols and colors used in the graph. Many graphs, including bar graphs and line graphs, have two axes that form a corner. Usually these axes are the left side and the bottom of the graph. Each axis will always have a label tells you what each axis measures. Bar Graphs A bar graph has two axes and uses bars to show amounts. In Graph 1, we see that the x-axis shows grades students earned, and the y-axis shows how many students earned each grade. You can see that 6 students earned an A because the bar for A stretches up to 6 on the vertical measurement. There is a lot of information we can get from a simple graph like this (See Graph 1) Line Graphs A line graph looks similar to a bar graph, but instead of bars, it plots points and connects them with a line. It has the same parts as a bar graph---two labeled axes---and can be read the same way. To read a line graph, it’s important to focus on the points of intersection rather than the line segments between the points. This type of graph is most commonly used to show how something changes over time. Here is a graph that charts how far a bird flies during the first five days of its spring migration (See Graph 2). The unit of measurement for the x-axis is days. The unit of measurement for the y-axis is kilometers. Thus we can see that, on the first day, the pipit flew 20 kilometers. The line segment goes up between Day 1 and Day 2, which means that the bird flew farther on Day 2. If the line segment angled down, as between Day 4 and Day 5, it would mean that the bird flew fewer kilometers than the day before. This line graph is a quick, visual way to tell the reader about the bird’s migration. Pie Graphs A typical pie graph looks like a circular pie. The circle is divided into sections, and each section represents a fraction of the data. The graph is commonly used to show percentages; the whole pie represents 100 percent, so each piece is a fraction of the whole. A pie graph might include a legend, or it might use icons or labels within each slice. This pie graph shows one month’s expenses(See Graph 3). Food $25 Movies $12 Clothing $36 Savings $20 Books $7 46. When used in a graph, a legend is __________. A. a guide to the symbols and colors B. an introduction paragraph C. the main idea D. the data 47. What is the total number of students who earned a C or better? A. 4. B. 6. C. 10. D. 20. 48. The bird covered the longest distance on __________. A. Day 1. B. Day 2. C. Day 3. D. Day 4. 49. Which of the following cost Amy most? A. Food. B. Books. C. Movies. D. Clothing. C If humans were truly at home under the light of the moon and stars, we would go in darkness happily, the midnight world as visible to us as it is to the vast number of nocturnal(夜间活动的)species on this planet. Instead, we are diurnal creatures, with eyes adapted to living in the sun’s light. This is a basic evolutionary fact, even though most of us don’t think of ourselves as diurnal beings. Yet it’s the only way to explain what we’ve done to the night: We’ve engineered it to receive us by filling it with light. The benefits of this kind of engineering come with consequences---called light pollution--- whose effects scientists are only now beginning to study. Light pollution is largely the result of bad lighting design, which allows artificial light to shine outward and upward into sky. Ill-designed lighting washes out the darkness of night and completely changes the light levels---and light rhythms---to which many forms of life, including ourselves, have adapted. Wherever human light spills into the natural world, some aspect of life is affected. In most cities the sky looks as though it has been emptied of stars, leaving behind a vacant haze(霾 ) that mirrors our fear of the dark. We’ve grown so used to this orange haze that the original glory of an unlit night---dark enough for the planet Venus to throw shadows on Earth---is wholly beyond our experience, beyond memory almost. We’ve lit up the night as if it were an unoccupied country, when nothing could be further from the truth. Among mammals alone, the number of nocturnal species is astonishing. Light is a powerful biological force, and on many species it acts as a magnet(磁铁). The effect is so powerful that scientists speak of songbirds and seabirds being “captured” by searchlights on land or by the light from gas flares on marine oil platforms. Migrating at night, birds tend to collide with brightly lit tall buildings. Frogs living near brightly lit highways suffer nocturnal light levels that are as much as a million times brighter than normal, throwing nearly every aspect of their behavior out of joint, including their nighttime breeding choruses. Humans are no less trapped by light pollution than the frogs. Like most other creatures, we do need darkness. Darkness is as essential to our biological welfare, to our internal clockwork, as light itself. Living in a glare of our own making, we have cut ourselves off from our evolutionary and cultural heritage---the light of the stars and the rhythms of day and night. In a very real sense, light pollution causes us to lose sight of our true place in the universe, to forget the scale of our being, which is best measured against the dimensions of a deep night with the Milky Way---the edge of our galaxy---arching overhead. 50. According to the passage, human beings __________. A. prefer to live in the darkness B. are used to living in the day light C. were curious about the midnight world D.had to stay at home with the light of the moon 51. What does “it”(Paragraph 1) most probably refer to? A. The night B. The moon C. The sky D. The planet 52. The writer mentions birds and frogs to __________. A. provide examples of animal protection B. show how light pollution affects animals C. compare the living habits of other species D. explain why the number of certain species has declined 53. It is implied in the last paragraph that __________. A. light pollution does harm to the eyesight of animals B. light pollution has destroyed some of the world heritages C. human beings cannot go to the outer space D. human beings should reflect on their position in the universe 54. What might be the best title for the passage? A. The Magic Light B. The Orange Haze C. The Disappearing Night D. The Rhythms of Nature D In 2004, when my daughter Becky was ten, she and my husband, Joe, were united in their desire for a dog. As for me, I shared none of their canine lust. But why, they pleaded. “Because I don’t have tine to take care of a dog.” But we’ll do it. “Really? You’re going to walk the dog? Feed the dog? Bathe the dog?” Yes,yes and yes.”I don’t believe you.” We will. We promise. They didn’t. From day two (everyone wanted to walk the cute puppy that first day), neither thought to walk the dog. While I was slow to accept that I would be the one to keep track of her shots, to schedule her vet appointments, to feed and clean her, Misty knew this on day one. As she looked up at the three new humans in her life (small, medium, and large), she calculated, “The medium one is the sucker in the pack.” Quickly, she and I developed something very similar to a Vulcan mind meld(心灵融合). She’d look at me with those sad brown eyes of hers, beam her need, and then wait, trusting I would understand---which, strangely, I almost always did. In no time, she became my fifth appendage(附肢), snoring on my stomach as I watched television. Even so, part of me continued to resent walking duty. Joe and Becky had promised. Not fair, I’d balk(不心甘情愿地做) silently as she and I walked.“Not fair,” I’d loudly remind anyone within earshot upon our return home. Then one day---January 1, 2007, to be exact---my husband’s doctor uttered an unthinkable word: leukemia (白血病). With that, I spent eight to ten hours a day with Joe in the hospital, doing anything and everything I could to ease his discomfort. During those six months of hospitalizations, Becky, 12 at that time, adjusted to other adults being in the house when she returned from school. My work colleagues adjusted to my taking off at a moment’s notice for medical emergencies. Every part of my life changed; no part of my old routine remained. Save one: Misty still needed walking. At the beginning, when friends offered to take her through her paces, I declined because I knew they had their own households to deal with. As the months went by, I began to realize that I actually wanted to walk Misty. The walk in the morning before I headed to the hospital was a quiet, peaceful time to gather my thoughts or to just be before the day’s medical drama unfolded. The evening walk was a time to shake off the day’s upsets and let the worry tracks in my head go to white noise. When serious illness visits your household, it’s not just your daily routine and your assumptions about the future that are no longer familiar. Pretty much everyone you know acts differently. Not Misty. Take her for a walk, and she had no interest in Joe’s blood or bone marrow test results. On the street or in the park, she had only one thing on her mind: squirrels! She was so joyful that even on the worst days, she could make me smile. On a daily basis, she reminded me that life goes on. After Joe died in 2009, Misty slept on his pillow. I’m grateful---to a point. The truth is, after years of balking, I’ve come to enjoy my walks with Misty. As I watch her chase a squirrel, throwing her whole being into the here-and-now of an exercise that has never once ended in victory, she reminded me, too, that no matter how harsh the present or unpredictable the future, there’s almost always some measure of joy to be extracted from the moment. 55. Why didn’t the writer agree to raise a dog at the beginning of the story? A. She was afraid the dog would get the family into trouble. B. It would be her business to take care of the dog. C. Her husband and daughter were united as one. D. She didn’t want to spoil her daughter. 56. Which of the following is the closest in meaning to “The medium one is the sucker in the pack.” A. “The middle-aged person loves me most.” B. “The medium-sized woman is the hostess.” C. “The man in the middle is the one who has the final say.” D. “The woman is the kind and trustworthy one in the family.” 57. It can be inferred from Paragraph 3 that __________. A. Misty was quite clever B. Misty could solve math problems C. the write was a slow learner D. no one walked Misty the first day 58. The story came to its turning point when __________. A. Joe died in 2009 B. Joe fell ill in 2007 C. the writer began to walk the dog D. the dog tried to please the writer 59. Why did the writer continue to walk Misty while Joe was in hospital? A. Misty couldn’t live without her. B. Her friends didn’t offer any help. C. The walk provided her with spiritual comfort. D. She didn’t want Misty to be others’ companion. 60. What is the message the writer wants to convey in the passage? A. One should learn to enjoy hard times. B. A disaster can change everything in life. C. Moments of joy suggest that there is still hope ahead. D. People will change their attitude toward you when you are in difficulty.. 第二节:下面文章中有 5个段落需要添加首句(第 61—65题)。请从以下选项(A、B、C、D、 E和 F)中选出适合各段落的首句,并在答题纸上将相应选项的标号涂黑。选项中有一项是多余 选项。 A. Come in with something to say. B. Prepare general comments. C. Bring materials with you. D. Don’t make them wait. E. Have no fear. F. Go it alone. One of the best things you can do any time in the semester is go see the professor. So hoof on over to an office hour and have some one-on-one face time with someone who’ll help you master the material and improve your grade, to boot. But how should you have this conversation with the professor?Here are five insider tips about how to make that office hour really count:. 61. _________ No need to get all bent out of shape about going to see the professor. The professor would actually like to see you and answer your questions. Believe it or not, he or she is on your side and is eager to see you do well. And besides, he or she has seen many students stupider than you, so nothing you’re going to ask will set the record for stupidity. 62. _________ Even though you might feel more comfortable going with a friend or partner, the office hour will go better if it’s just you and the professor. You’ll get in more questions, the discussion will be tailored to what you need most help on, and two-party communication is almost always more productive than committee work. Your friend can wait outside for the discussion. 63. __________ If you can’t make the official office hours, most professors are willing to make individual appointments to help you out. If you’re lucky enough to land such an accommodation, though, be sure you’re 100 percent on time. There’s nothing that ticks off a professor more than making him-or herself available for a custom office hour only to find that you don’t care enough to come on time. And besides, the professor might leave after ten minutes, which would make your trip a total loss. 64. __________ If you’re meeting with the professor to go over a paper or test, or to ask questions about a particular lecture or reading,make sure you bring that paper or test, or your lecture notes or a copy of the article. The professor doesn’t remember the comments he or she wrote on your individual piece of work---though he or she will be able to recall them after just a brief glance at your work. And if you have your lecture notes or the article in hand, you and the professor will be able to examine specific points that are confusing to you, rather than just talking in a general way about the contents. 65. ___________ Office hours almost always go better if you bring a few specific questions to the meeting. It’s almost never good to start a meeting with general comments such as“I didn’t understand what you said about [main topic of the course]” or“I couldn’t understand any of your lectures last week.” Much better is to come in with two or three conversation-starters about a specific concept, point, or problem you didn’t understand. Keep in mind that in a fifteen-minute office hour (which is how long these things usually last), two or three questions are usually the most you’ll have time to discuss. 非选择题部分(共 40分) 第三部分:写作(共两节,满分 40 分) 第一节:短文改错(共 10小题;每小题 1分,满分 10分) 下面短文中有 10处语言错误。请在有错误的地方增加、删除或修改某个单词。 增加:在缺词处加一个漏字符号(∧),并在其下面写上该加的词。 删除:把多余的词用斜线(\)划掉。 修改:在错的词下划一横线,并在该词下面写上修改后的词。 注意:1.每处错误及修改均仅限一词; 2.只允许修改 10处,多者(从第 11处起)不计分。 例如: It was very nice to get your invitation to spend∧weekend with you. Luckily I was completely tree the am then, so I'll to say “yes”. I'll arrive in Bristol at around 8:00p.m. in Friday evening. on My old classroom was interesting because three side of the classroom were made from glass. I enjoyed sit close to the windows and looking at the view. On the left-hand side of the class, I could easy see the football field. In the morning ,it was full of the students exercising. The view from the back of the classroom is also splendid. Close to the school there was a beautiful park with many trees around them. Farther in the distance, I could not enjoy the view of snowy mountains. On the right side of the class was the road ,I was always interested to see the drivers in hurry in the morning. The position of the classroom with its view made me felt like I was dreaming. If I was only a child when I studied in that classroom, I will never forget it. 第二节:书面表达(满分 30 分) 在班级活动中,当你的想法与大多数同学不一致时,你是坚持自己的观点并说服别人,还是 尊重大多数同学的意见?请你以When I Have Different Opinion 为题,用英语写一篇 100~120个词 的短文。要求如下: 1、以上两种做法中选择一种; 2、以具体的实例阐述你选择的理由。 注意:短文中,不得以任何形式透露地区、学校、同学姓名等真实信息。否则按考试作弊行 为认定。 When I Have Different Opinion ____________________________________________________________________________________ 2015年普通高等学校招生全国统一考试(浙江卷) 自选模块英语科试题 题号:05 科目:英语 阅读理解(分两节,共 5小题;每小题 2分,共 10分) 阅读下面短文,并根据短文后的要求答题。 Children starting school this year will be retiring in 2070. No one has any idea of what the world will look like in ten years' time, let alone in 2070.There are two major drivers of change— technology and demography(人口状况). ① It is also contributing to what some pundits (权威人士)are calling the biggest generation gap since rock and roll. People over the age of thirty were born before the digital revolution really started. We’ve learned to use digital technology—laptops, cameras, personal digital assistants, the Internet—as adults, and it has been something like learning a foreign language. ② We do e-mails and PowerPoint, surf the Internet, and feel we’re at the cutting edge. But compared to most people under thirty and certainly under twenty, we are fumbling amateurs. People of that age were horn after the digital revolution began. They learned to speak digital as a mother tongue. But younger children who are growing up with even more sophisticated technologies are already outperforming teenagers of his generation. And this revolution is not over. ③ Some suggest that, in the near future, the power of laptop computers will match the computing power of the human brain. Before too long we may see the merging of information systems with human consciousness. If you think about the impact (影响) in the last twenty years of relatively simple digital technologies on the work we do and how we do it—and the impact these technologies have had on national economies— think of the changes that lie ahead. Don't worry if you can’t predict them: nobody can. Add to this the impact of population growth. The world population has doubled in the past thirty years, from three to six billion. It may be heading for nine billion by the middle, of the century. This great new mass of humanity will be using technologies that have yet to be invented in ways we cannot imagine and in jobs that don't yet exist. These driving cultural and technological forces are bringing about great changes in the world economies and increasing diversity and complexity in our daily lives, and especially in those of young people. The simple fact is that these are times of unprecedented (前所未有的) global change. ④ 第一节 根据短文内容,从 A、B、C、D和 E中选出最适合填入短文空白处的选项,并将序号及 相应答案写在答题纸上。选项中有一项是多余选项。 A. In fact, it's barely begun. B. Most of us are okay, and some are even expert. C. We need to think very differently about human resources. D. We can identify trends for the future, but accurate predictions are almost impossible. E. Technology — especially digital technology — is developing at a rate that most people cannot properly grasp. 第二节 根据短文所给的信息,用一个完整的句子回答下面的问题,并将序号及相应答案写在答 题纸上。 (5)What are the main causes of the great changes mentioned in this passage? 题号:06 科目:英语 填空(共 10小题;每小题 1分,共 10分) 阅读下面的短文,在标有序号的空白处填入一个适当的词,或填入括号中所给单词的正确形式, 并将序号及相应的答案写在答题纸上。 A year ago, my son Michael, who had just graduated from high school, had to choose between two summer jobs. He could work either at my basketball camp ① at one of the horse farms in the Lexington, Kentucky, area. It was his choice. He ② (choose) the horse farm,because he was tired of working my camps, and he thought he could make more money at the horse farm. But ____③____ only a few days he realized he hated it. Farm work is hard,and he was spending most of his time ④ (walk)horses on a treadmill and shoveling manure. One day a couple of weeks into the summer he came to me and said he’d made a mistake and ⑤ rather work at my basketball camp. I understood, but I told him that he’d made a commitment and now he was going to have to follow through with it, no matter ⑥ unpleasant. Quitting was not an option. Sometimes, of course, we fail to accomplish our goals. But we have to remember that failure is a part of life and failure is only fertilizer for future success. I’ve often heard people say that failure teaches us ⑦ . I don’t believe it. There are so many lessons we can learn ⑧ failure. Most of all, we learn what ⑨ to do .It’s like the child who touches the hot stove, and then keeps doing it. The first time is part of the learning process. The second time it’s stupid. The only time failure is truly bad is if you use it ⑩ an excuse to quit. ①__________ ②__________ ③__________ ④__________ ⑤__________ ⑥__________ ⑦__________ ⑧__________ ⑨__________ ⑩__________ 2015年普通高等学校招生全国统一考试(福建卷) 英 语 本试卷分为四个部分,共 12页。时量 120分钟。满分 150分 本试卷分第 I卷(选择题)和第 II卷(非选择题)两部分。满分 150分。考试时间 120 分钟。 注意事项: 1. 答题前,考生务必先将自已的姓名、准考证号填写在答题卡上。 2. 考生作答时,请将答案答在答题卡上,请按照题号在各题的答题区域(黑色线框)内作 答,超出答题区域书写的答案无效。在草稿纸、试卷上答题无效。 3. 选择题答案使用 2B铅笔填涂,如需改动,用橡皮擦干净后,再选涂其他答案标号;非 选择题答案使用 0.5毫米的黑色中性(签字)笔或碳素笔书写,字体工整、笔迹清楚。 4. 保持答题卡卡面清洁’不折叠,不破损。考试结束后,将本试卷和答题卡一并交回。 第Ⅰ卷(选择题 共 115分) 第一部分 听力(共两节,满分 30分) 回答听力部分时,请先将答案标在试卷上。听力部分结束前,你将有两分钟的时间将你的答 案转涂到客观题答题卡上。 第一节 (共 5 小题;每小题 1.5 分,满分 7.5 分) 听下面 5段对话,每段对话后有一个小题。从题中所给的 A,B,C三个选项中选出最佳选项, 并标在试卷的相应位置。听完每段对话后,你都有 10秒钟的时间来回答有关小题和阅读下一小题。 每段对话仅读一遍。 例: How much is the shirt? A. £ 19.15 B. £ 9.18 C. £ 9.15 答案是 C。 1. 1. What time is it now? A. 9:10 B. 9:50 C. 10:00 2. What does the woman think of the weather? It’s nice. It’s warm It’s cold. 3. What will the man do? A. Attend a meeting. B. Give a lecture C. Leave his office. 4. What is the woman’s opinion about the course? A. Too hard B. Worth taking. C. Very easy. 5. What does the woman want the man to do? A. Speak louder B. Apologize to her. C. Turn off the radio. 第二节(共 15 小题;每小题 1.5 分,满分 22.5 分) 听下面 5短话或独白,没段话或独白后有几个小题,从题中所给的 A、B、C三个选项中选出 最佳选项,并标在试卷的相应位置。听每段对话或独白,你将有时间阅读各个小题,每小题 5秒 钟;听完后,各小题将给出 5秒钟的做大时间,每段对话或独白读两遍。 听第 6段材料,回答第 6、7题。 6. How long did Michael stay in China? A. Five days. B. One week. C. Two weeks. 7. Where did Michael go last year? A. Russia B. Norway C. India 听第 7段材料,回答第 8、9题。 8. What food does Sally like? A. Chicken. B. Fish. C. Eggs. 9. What are the speakers going to do? A. Cook dinner. C. Go shopping. C. Order dishes. 听第 8段材料,回答第 10至 12题。 10. Where are the speakers? A. In a hospital. B. In the office. C. At home. 11. When is the report due? A. Thursday. B. Friday. C. Next Monday. 12. What does George suggest Stephanie do with the report? A. Improve it. B. Hand it in later. C. Leave it with him. 听第 9段材料,回答第 13至 16题。 13. What is the probable relationship between the speakers? A. Salesperson and customer. B. Homeowner and cleaner. C. Husband and wife. 14. What kind of apartment do the speakers prefer? A. One with two bedroom. B. One without furniture. C. One near a market. 15. How much rent should one pay for the one-bedroom apartment? A. $350. B. $400. C. $415. 16. Where is the apartment the speakers would like to see? A. On Lake Street B. On Market Street. C. On South Street. 听第 8段材料,回答第 10至 12题。 17. What percentage of the world’s tea exports go to Britain? A. About 15%. B. About 30%. C. Over 40%. 18. Why do tea tasters taste tea with milk? A. Most British people drink that way. B. Tea tastes much better with milk. C. Tea with milk is healthy. 19. Who suggests a price for each tea? A. Tea tasters. B. Tea exporters. C. Tea companies. 20. What is the speaker talking about? A. The life of tea tasters. B. Afternoon tea in Britain. C. The London Tea Trade Centre. 第二部分 英语知识运用(共两节,满分 45分) 第一节 单项填空(共 15 小题;每小题 1 分,满分 15 分) 从 A、B、C、D四个选项中,选出可以填入空白处的最佳选项,并在答题卡上将该项涂黑。 例: It is generally considered unwise to give a child __________ he or she wants. A. however B. whatever C. whichever D. whenever 答案是 B 21. The research group produced two reports based on the survey, but __________ contained any useful suggestions. A. all B. none C. either D. neither 22. A common memory they all have __________ their school days is the school uniform. A. of B. on C. to D. with 23. It was __________ of Michael to inform us of his delay in case we got worried. A. careless B. considerate C. patient D. generous 24. The failure was a big __________ to him, but he wasn't discouraged and soon got as enthusiastic as ever. A. blow B. issue C. excuse D. factor 25. It is said that body language __________ 55 per cent of a first impression while what you say just 7 per cent. A. lies in B. accounts for C. consists of D. goes with 26. To my delight, I __________ from hundreds of applicants to attend the opening ceremony. A. was chosen B. was being chosen C. would choose D. had chosen 27. —Sorry, Mum! I failed the job interview again. —Oh, it's too bad. You __________ have made full preparations. A. must B. can C. would D. should 28. __________ more about Chinese culture, Jack has decided to take Chinese folk music as an elective course. A. Learn B. Learned C. To learn D. To be learning 29. —I wonder __________ Mary has kept her figure after all these years. —By working out every day. A. where B. how C. why D. if 30. —Where is Peter? I can't find him anywhere. —He went to the library after breakfast and __________ his essay there ever since. A. wrote B. had written C. has been writing D. is writing 31. __________ the students came from different countries, they got along quite well in the summer camp. A. While B. Unless C. Since D. Until 32. Human life is regarded as part of nature and, as such, the only way for us to survive is to live __________ nature. A. in view of B. in need of C. in touch with D. in harmony with 33. In recent years an English word "infosphere" has appeared, __________the sense of “information” and " atmosphere". A. combine B. combined C. combing D. being combined 34. China Today attracts a worldwide readership, __________ shows that more and more people all over the world want to learn about China. A. who B. whom C. that D. which 35. —Hi, Dr Brown! I'm a little early. Should I wait outside? —No. __________. A. That's right B. My pleasure C. Come on in D. Take it easy 第二节 完形填空(共 20 小题;每小题 1.5 分,满分 30 分) 阅读下面短文,从短文后各题所给的四个选项(A、B、C、和 D)中,选出可以填入空白处 的最佳选项,并在答题卡上将该项涂黑。 One of the easiest things in the world is to become a fault-finder. However, life can be ___36___ when you are not busy finding fault with it. Several years ago I 37 a letter from seventeen-year-old Kerry, who described herself as a world-class fault-finder, almost always 38 by things. People were always doing things that annoyed her, and 39 was ever good enough. She was highly self-critical and also found fault with her friends. She became a really 40 person. Unfortunately, it took a horrible accident to change her 41 Her best friend was seriously hurt in a car crash. What made it almost 42 to deal with was that the day before the 43 , Kerry had visited her friend and had spent the whole time criticizing her 44 of boyfriends, the way she was living, the way she related to her mother, and various other things she felt she needed to ___45___ It wasn't until her friend was badly hurt that Kerry became 46 her habit of finding fault. Very quickly, she learned to appreciate life rather than to 47 everything so harshly (刻薄) . She was able to transfer her new wisdom to other parts of her 48 as well. Perhaps most of us aren't as extreme at fault-finding, 49 when we're honest, we can be sharply 50 of the world. I'm not suggesting you 51 problems, or that you pretend things are ___52___ than they are, but simply that you learn to allow things to be as they are— 53 most of the time, and especially when it's not a really big 54 . Train yourself to "bite your tongue" , and with a little 55 , you'll get really good at letting things go. And when you do, you'll get back your enthusiasm and love for life. 36. A. lonely B. great C. quiet D. uneasy 37. A. received B. answered C. expected D. rejected 38. A. threatened B. interrupted C. bothered I). spoiled 39. A. anything B. everything C. something D. nothing 40. A. caring B. boring C. interesting D. surprising 41. A. attitude B. plan C. measure D. explanation 42. A. urgent B. unnecessary C. certain D. impossible 43. A. occasion B. event C. accident D. adventure 44. A. memory B. notice C. evidence D. choice 45. A. hear B. contribute C. express D. admit 46. A. aware of B. afraid of C. curious about D. confused about 47. A. discuss B. realize C. judge D. settle 48. A. family B. life C. career D. education 49. A. so B. or C. but D. for 50. A. proud B. sure C. hopeful D. critical 51. A. face B. create C. solve D. ignore 52. A. rarer B. better C. stranger D. worse 53. A. at least B. at last C. by far D. so far 54. A. task B. deal C. result D. duty 55. A. practice B. speech C. rest D pity 第三部分 阅读理解(共 20小题;每小题 2分,满分 40分) 阅读下列短文,从每题所给的四个选项(A、B、C和 D)中,选出最佳选项,并在答 题卡上将该项涂黑。 A Food festivals around the world Stilton Cheese Rolling May Day is a traditional day for celebrations, but the 2,000 English villagers of Stilton must be the only people in the world who include cheese rolling in their annual plans. Teams of four,dressed in a variety of strange and funny clothes , roll a complete cheese along a 50-metre course. On the way, they must not kick or throw their cheese, or go into their competitors' lane. Competition is fierce and the chief prize is a complete Stilton cheese weighing about four kilos (disappointingly, but understandably the cheeses used in the race are wooden ones). All the competitors arc served with beer or port wine, the traditional accompaniment for Stilton cheese. Fiery Foods Festival—The Hottest Festival on Earth Every year more than 10,000 people head for the city of Albuquerque, New Mexico. They come from as far away as Australia, the Caribbean and China, but they all share a common addiction—food that is not just spicy(辛辣 ) , but hot enough to make your mouth burn, your head spin and your eyes water. Their destination is the Fiery Food and BBQ Festival which is held over a period of three days every March. You might like to try a chocolate-covered habanero pepper—officially the hottest pepper in the world—or any one of the thousands of products that are on show. But one thing's for sure—if you don't like the feeling of a burning tongue, this festival isn't for you! La Tomatina—The World's Biggest Food Fight On the last Wednesday of every August, the Spanish town of Bunol hosts Ea Tomatina—the world's largest food fight. A week-long celebration leads up to an exciting tomato battle as the highlight of the week's events. The early morning sees the arrival of large trucks with tomatoes—official fight-starters get things going by casting tomatoes at the crowd. The battle lasts little more than half an hour, in which time around 50,000 kilograms of tomatoes have been thrown at anyone or anything that moves, runs, or fights back. Then everyone heads down to the river to make friends again—and for a much-needed wash! 56. In the Stilton cheese rolling competition, competitors on each team must __________. A. wear various formal clothes B. roll a wooden cheese in their own lane C. kick or throw their cheese D. use a real cheese weighing about four kilos 57. Where is the Fiery Food and BBQ Festival held? A. In New Mexico. B. In the Caribbean. C. In Australia. D. In China. 58. The celebration of La Tomatina lasts __________. A. three days B. seven days C. less than three days D. more than seven days 59. Which of the following is TRUE according to the passage? A. The chief prize for the Stilton cheese rolling competition is beer or port wine. B. More than 10,000 Chinese take pail in the Fiery Food and BBQ Festival. C. Thousands of spicy foods are on show in the Fiery Food and BBQ Festival. D. An exciting tomato battle takes place at the beginning of La Tomatina. B Papa, as a son of a dirt-poor farmer, left school early and went to work in a factory, for education was for the rich then. So, the world became his school. With great interest, he read everything he could lay his hands on, listened to the town elders and learned about the world beyond his tiny hometown. "There's so much to learn," he'd say. "Though we're born stupid, only the stupid remain that way. " He was determined that none of his children would be denied (拒绝) an education. Thus, Papa insisted that we learn at least one new thing each day. Though, as children, we thought this was crazy, it would never have occurred to us to deny Papa a request. And dinner time seemed perfect for sharing what we had learned. We would talk about the news of the day; no matter how insignificant, it was never taken lightly. Papa would listen carefully and was ready with some comment, always to the point. Then came the moment—the time to share the day's new learning. Papa, at the head of the table, would push back his chair and pour a glass of red wine, ready to listen. "Felice," he'd say, "tell me what you learned today. " "I learned that the population of Nepal is .... " Silence. Papa was thinking about what was said, as if the salvation(拯救 ) of the world would depend upon it. "The population of Nepal. Hmm. Well . . . . " he'd say. "Get the map; let's see where Nepal is. " And the whole family went on a search for Nepal. This same experience was repeated until each family member had a turn. Dinner ended only after we had a clear understanding of at least half a dozen such facts. As children, we thought very little about these educational wonders. Our family, however, was growing together, sharing experiences and participating in one another's education. And by looking at us, listening to us, respecting our input, affirming(肯定) our value, giving us a sense of dignity, Papa was unquestionably our most influential teacher. Later during my training as a future teacher /1 studied with some of the most famous educators. They were imparting(传授 ) what Papa had known all along—the value of continual learning. His technique has served me well all my life. Not a single day has been wasted, though I can never tell when knowing the population of Nepal might prove useful. 60. What do we know from the first paragraph? A. The author's father was born in a worker's family. B. Those born stupid could not change their life. C. The town elders wanted to learn about the world. D. The poor could hardly afford school education. 61. The underlined word "it" in the second paragraph refers to “__________”. A. one new thing B. a request C. the news D. some comment 62. It can be learned from the passage that the author __________. A. enjoyed talking about news B. knew very well about Nepal C. felt regret about those wasted days D . appreciated his father's educational technique 63. What is the greatest value of "dinner time" to the author? A. Continual learning. B. Showing talents. C. Family get-together. D. Winning Papa's approval. 64. The author's father can be best described as __________. A. an educator expert at training future teachers B. a parent insistent on his children's education C. a participant willing to share his knowledge D. a teacher strict about everything his students did C SIGN YOUR CHILD UP FOR "FLYTO THE MOON CLUB" AND ENJOYAFREE * FLIGHT TOANY DESTINATION INASIA! With a registration fee of just $50 per child, children under the age of 12 can join Eagle Airways' FLY TO THE MOON CLUB as members. They can then enjoy the same benefits onboard Eagle Airways' newest Boeing-797 to any destination in the world! BENEFITS YOU CAN'T MISS! • A free * flight to any destination in Asia • 30% off any course at Tanya Language School • 20% off any purchase made at Ruby Bookstore • A free notebook with every purchase above $50 at Starlight Stationery • A free bowl of dessert for a family of four at Don's Diners dinner ordered • A birthday gift on your child's birthday • A free album containing pictures taken during the journey All bookings made before 12 September will receive free travel insurance for the entire family! ** Insurance is issued by Live Life Insurance Group. 10% OFFALL BOOKINGS for departures from 5 to 11 September * Child must be accompanied by two paying adults. ** Terms and conditions apply. 65. One of the benefits mentioned in, the advertisement is __________. A. a free flight to any destination in the world B. 30% off any book purchased at Ruby Bookstore C. a free bowl of dessert at any restaurant at the airport D. a discount on any course at Tanya Language School 66. Which of the following bookings may receive the most benefits? A. Booking date Departure date September 13 September 18 B. Booking date Departure date September 2 September 12 C. Booking date Departure date August 15 September 4 D. Booking date Departure date August 16 September 8 67. Which of the following is TRUE according to the advertisement? A. You need to pay $50 to sign up a child for the club. B. Club members enjoy free travel insurance for any flight. C. The advertisement is intended for students of all ages. D. Any child must be accompanied by at least one paying adult. D Life can be so wonderful, full of adventure and joy. It can also be full of challenges, setbacks and heartbreaks. Whatever our circumstances, we generally still have dreams, hopes and desires—that little something more we want for ourselves and our loved ones. Yet knowing we can have more can also create a problem, because when we go to change the way we do things, up come the old patterns and pitfalls that stopped us from seeking what we wanted in the first place. This tension between what we feel we can have and "what we're seemingly able to have is the niggling suffering, the anxiety we feel. This is where we usually think it's easier to just give up. But we're never meant to let go of the part of us that knows we can have more. The intelligence behind that knowing is us—the real us. It's the part that believes in life and its possibilities. If you drop that, you begin to feel a little "dead" inside because you're dropping "you". So, if we have this capability but somehow life seems to keep us stuck, how do we break these patterns? Decide on a new course and make one decision at a time. This is good advice for a new adventure or just getting through today's challenges. While, deep down, we know we can do it, our mind—or the minds of those close to us—usually says we can't. That isn't a reason to stop, it's just the mind, that little man or woman on your shoulder, trying to talk you out of something again. It has done it many times before. It's all about starting simple and doing it now. Decide and act before overthinking. When you do this you may feel a little, or large, release from the jail of your mind and you'll be on your way. 68. It can be inferred from the first two paragraphs that we should __________. A. slow down and live a simple life B. be careful when we choose to change C. stick to our dreams under any circumstances D. be content with what we already have 69. What is the key to breaking the old patterns? A. To focus on every detail. B. To decide and take immediate action. C. To listen to those close to us. D. To think twice before we act. 70. Which of the following best explains the underlined part in the last paragraph? A. Escape from your punishment. B. Realization of your dreams. C. Freedom from your tension. D. Reduction of your expectations. 71. What does the author intend to tell us? A. It's easier than we think to get what we want. B. It's important to learn to accept sufferings in life. C. It's impractical to change our way of thinking. D. It's harder than we expect to follow a new course. E Group exercise is one of the most effective ways to improve physical fitness and sustain a healthy lifestyle. Group exercise is challenging, yet fun and empowering! Of course everyone knows that exercise is good for the body. However, studies have shown that when exercise is performed in groups, it's not only great for improving physical health but for psychological health. It's an opportunity to be social, release endorphins(内啡肽 ), and improve your strength. Additionally, group exercise creates a community feel and the shared common goal motivates participants to work hard. The instrumental support of taking on a fitness journey with others proves more effective than going to the gym alone. Another beneficial aspect of group exercise is the informational support participants receive from the instructor. Many people fear the gym because they feel lost and don't want to embarrass themselves. If you feel you can relate, then group training is an even better option for you. It's a great opportunity to learn more about fitness through the clear instruction and supervision (监管) of a fitness instructor. If you're tired of wandering around the gym wasting time and becoming bored, you can attend an upbeat group fitness class that’ll keep your workout on track. Don't let fitness frighten you! If you're serious about wanting to live a healthy lifestyle, it's extremely important to surround yourself with people who'll provide you with the proper emotional support. I wouldn't scold anyone for deciding to party on weekends and in turn I wouldn't expect anyone to give offence to me for focusing on my health. Surround yourself with people who uplift, encourage and understand you! Make fitness even more fun by trying something new or any group fitness class, with a friend. Plan to go for a jog together. Then try a fun healthy restaurant or fresh juice bar! Fitness can be both fun and social! Surrounding yourself with people who'll provide you with respect support can be very beneficial while working towards reaching health and fitness goals. First, decide to do it for yourself and work towards staying positive. Then make sure the people you surround yourself with are supportive. Don't let negativity ruin your motivation. 72. The first paragraph focuses on __________. A. the greatest challenge of group exercise B. the most effective way to improve physical fitness C. the contribution of group exercise to psychological health D. the shared common goal in performing exercise in groups 73. The underlined word "upbeat" in the second paragraph probably means “___________”. A. cheerful B. average C. serious D. temporary 74. When it comes to emotional support, the author thinks it necessary __________. A. to sustain a colorful lifestyle B. to party on weekends with positive people C. to try a fun healthy restaurant regularly D. to surround yourself with supportive people 75. What would be the best title for the passage? A. Seeking Support B. Supporting Health C. Improving Your Strength D. Building Up Fitness 第Ⅱ卷(非选择题 共 35分) 第四部分 写作 (共两节,满分 35分) 第一节 短文填词(共 10 小题;每小题 1 分,满分 10 分) 阅读下面短文,根据以下提示:1)汉语提示,2)首字母提示,3)语境提示,在每个空格内填 入一个适当的英语单词,所填的词要求意义准确,拼写正确,并将该词完整地写在答题卡中响应 的横线上。 Sometimes we have disagreements with people. When this (76)h__________, the important thing is to try not to let a calm discussion turn into a heated argument. Here (77 )___________ my tips for you. The (78) f__________ thing I would say is that the way you begin the conversation is very important. Imagine you are a student and you share a flat (79) __________ another student who you think isn't doing her share of the housework. If you say, "Look, you never do your share of the housework. (80)__________ are you going to do about it?", the discussion will very soon turn into an argument. It's much more (81)__________(有帮助 ) to say something like, "I think we had (82) b __________have another look about how we divide up the housework. Maybe there is a better way of dealing with it. " My second piece of (83) a__________ is simple. If you're the person who is in the wrong, just admit it! This is the easiest and best way to avoid an argument. Just make an (84)__________(道歉), and move on. The other person will have more respect for you (85)__________ the future if you do that. 第二节 书面表达(满分 25 分) 请阅读下面图画,按要求用英语写一篇词数为 120左右的短文。 内容包括: 1. 描述画面; 2. 概述其含义; 3. 谈谈个人感想。 凿壁偷光 注意: 1. 短文开头已给出,不计入总词数; 2. 可以适当增加细节,以使行文连贯; 3. 短文中不能出现与本人相关的信息; 参考词汇:凿,钻:bore In the picture, ________________________________________________________________________ ____________________________________________________________________________________ ____________________________________________________________________________________ ____________________________________________________________________________________ 2015年普通高等学校招生全国统一考试(湖南卷) 英 语 本试卷分为四个部分,共 12页。时量 120分钟。满分 150分 PartⅠ Listening Comprehension (30 marks) Section A (22.5 marks) Directions: In this section, you will hear six conversations between two speakers. For each conversation, there are several questions and each question is followed by three choices marked A, B and C. Listen carefully and then choose the best answer for each question. You will hear each conversation TWICE. Example: When will the magazine probably arrive? A. Wednesday B. Thursday C. Friday The answer is B. Conversation 1 1. When does the woman usually get home from work? A. About 6:30. B. About 7:30. C. About 8:30. 2. What did the woman do last night? A. She watched TV. B. She recorded a program. C. She prepared for a lecture. Conversation 2 3. How often does the man exercise at the gym? A. Every day. B. Every two days. C. Once a week. 4. Where will the two speakers meet before doing exercise this Friday? A. At the park. B. At the cafe. C. At the cinema. Conversation 3 5. What is Mr. Chester doing? A. Telephoning someone. B. Speaking to the woman. C. Leaving the man a message. 6. What is the man's last name? A. Oliver. B. Horst. C. Robert. Conversation 4 7. Why will the woman be late? A. She didn't catch the train. B. She didn't finish her paper. C. She didn't wake up in time. 8. Where is the man? A. At the station. B. At home. C. At the office. 9. What is the probable relationship between the two speakers? A. Teacher and student. B. Parent and child. C. Husband and wife. Conversation 5 10. For whom does the woman buy the T-shirt? A. Herself. B. Her husband. C. Her friend. 11 .How much does the T-shirt normally cost? A. $54. B. $60. C. $70. 12. Why does the salesman agree to sell the T-shirt at $48? A. It is cheaper online. B. He is in a hurry. C. A button is lost. Conversation 6 13. When did the woman arrive? A. Friday. B. Saturday. C. Sunday. 14. What major did the man choose in the end? A. English. B. Biology. C . History. 15. What suggestion does the man give on reading the books? A. Making notes. B. Skimming first. C. Reading word by word. Section B(7.5 marks) Directions: In this section, you will hear a short passage. Listen carefully and then fill in the numbered blanks with the information you have heard. Fill in each blank with NO MORE THAN THREE WORDS. You will hear the short passage TWICE School Web Design 16 I. Meet on 17 from 2:30 to 3:15 II. Create a website for a competition ● Website ☆ on your ancestor 18 ☆ 19 long & well constructed ● Competition ☆ has a first prize of $300 for beginners & $ 20 for the advanced ☆ do it by yourself, list the software you used Part Ⅱ Language Knowledge (45 marks) Section A (15 marks) Directions: For each of the following unfinished sentences there are four choices marked A, B, C and D. Choose the one that best completes the sentence. 21. It was when we were returning home __________ I realized what a good feeling it was to have helped someone in trouble. A. which B. that C. where D. how 22. As you go through this book, you __________ that each of the millions of people who lived through World War II had a different experience. A. will find B. found C. had found D. have found 23. Only after talking to two students __________ that having strong motivation is one of the biggest factors in reaching goals. A. I did discover B. did I discover C. I discovered D. discovered 24. Video games can be a poor influence if __________ in the wrong hands. A. to leave B. leaving C. leave D. left 25. I wasn't able to hide my eagerness when I __________, "What do you wish me to do now?" A. ask B. have asked C. am asking D. asked 26. You have to know __________ you're going if you are to plan the best way of getting there. A. what B. that C. where D. who 27. It is important to remember that success __________ a sum of small efforts made each day and often __________ years of achieve. A. is; takes B. are; takes C. are; take D. is; take 28. He must have sensed that I __________ him. He suddenly glanced at me and said quietly, "Why are you staring at me like that?" A. would look at B. looked at C. was looking at D. am looking at 29. It is a truly delightful place, __________ looks the same as it must have done 100 years ago with its winding streets and pretty cottages.. A. as B. where C. that D. which 30. When the clerk saw a kind face wrinkled in an apologetic smile, she stood rooted to the ground, __________ whether to stay or leave. A. wondering B. wonder C. to wonder D. wondered 31. Always __________ in mind that your main task is to get this company running smoothly. A. to keep B. to have kept C. keep D. have kept 32. I had a strong desire to reach in and play with the toy, but __________ thankfully by the shop window. A. am held back B. held back C. hold back D. was held back 33. __________ the job takes a significant amount of time, most students agree that the experience is worth it. A. If only B. After C. Although D. In case 34. Sometimes I act as a listening ear for fellow students __________ what is bothering them. A. to take over B. talked over C. talk over D. having talked over 35. That’s why I help brighten people’s days. If you __________, who is to say that another person will? A. didn’t B. don’t C. weren’t D. haven’t Section B (18 marks) Directions: For each blank in the following passage there are four words or phrases marked A, B, C and D. Fill in each blank with the word or phrase that best fits the context. It was a rainy morning and the children, mainly boys with various learning difficulties, refused to settle for the start of the lesson. As an inexperienced teacher, I tried every means to get them to be ___36___, but in vain. my panic was rising and I could feel my heart beating wildly. This was the ___37___ of my job as a music teacher, I thought -- teaching was not for me. Then I had an idea. Hoping that no one would notice that I was 38 inside, I threw my voice as far as it would reach: "Put your heads on the desks and close your 39 ! We are going on a journey." 40 , the children fell silent. "Now what should I do?" I thought to myself. Reaching over to my collection of CDs, I blindly 41 , put it in the machine and played it. Obediently (顺从地), my class lay their heads on their desk, closed their eyes and 42 . When the music started, the room as filled with the most beautiful tones and musical colors I could have ever imagined. All the children were 43 . When the music finished, I asked them all to raise their ___44___ slowly so that we could share our musical journey. At this point, when all the children were willing to share their experiences, I began to learn how to 45 . The music allow me to learn that teaching is about sharing and respect, tears and smiles, the knowing and the 46 and most of all, an understanding of each other. This was the power that ___47___ in the classroom could have. 36. A. glad B. safe C. kind D. quiet 37.A. end B. aim C. rule D. plan 38. A. guessing B. shaking C. responding D. laughing 39. A. eyes B. mouths C. books D. doors 40. A. Punctually B. Importantly C. Amazingly D. Obviously 41. A. passed one on B. gave one back C. turned one in D. took one out 42. A. slept B. nodded C. waited D. continued 43. A. talking B. singing C. dancing D. listening 44. A. legs B. heads C. arms D. shoulders 45. A. teach B. imagine C. play D. understand 46. A. unprepared B. unspoken C. unknown D. unforgotten 47. A. games B. music C. tears D. knowledge Section C (12 marks) Directions: Complete the following passage by filling in each blank with one word that best fits the context. Research has become both simpler and more complex. It's simpler because, 48 you have computer, you can find information you need by searching the Internet. For all you information, you don't have to go to 49 library to find the relevant resource and take notes on it. Instead, you can find some sources from the Internet 50 print the copies needed. Remember, however, that you should usually consult different types of sources. That is, you 51 always rely just on the Internet for you research. While finding information is easier than ever, at the same time, researching has become ___52___ complex. There is a lot more material available, which means you may be overwhelmed ___53___ the amount of information. You need to learn 54 to sort through and find the relevant information for your particular project. Also, 55 need to check the accuracy of it. 48. ___________ 49. ___________ 50. ___________ 51. ___________ 52. ___________ 53. ___________ 54. ___________ 55. ___________ Part Ⅲ Reading Comprehension (30 marks) Directions: Read the following three passages. Each passage is followed by several questions or unfinished .statements For each of them there are four chokes marked A. B. C and D. Choose the one that fits best according to the information given in the passage. A Forget Cyclists, Pedestrians are Real Danger We are having a debate about this topic. Here are some letters from our readers. ■ Yes, many cyclists behave dangerously. Many drivers are disrespectful of cyclists. But pedestrians are probably the worse offenders. People of all ages happily walk along the pavement with eyes and hands glued to the mobile phone, quite unaware of what is going on around them. They may even do the same thing while crossing a road at a pedestrian crossing or elsewhere. The rest of us have to evade (避让) them or just stand still to wait for the unavoidable collision. The real problem is that some pedestrians seem to be, at least for the moment, in worlds of their own that are, to them, much more important than the welfare of others. ——Michael Horan ■I love the letter from Bob Brooks about cyclists (Viewpoints, May 29). I am afraid they seem to think they own the roads. I was walking across Altrincham Road one morning when a cyclist went round me and on being asked what he was doing he shouted at me. The government built a cycle lane on the road but it is hardly used. The police do nothing. What a laugh they are! The cyclists should all have to be made to use the cycle lanes and wear helmets, fluorescent (发荧 光的) jacket and lights at night and in the morning they should pay some sort of tax and be fined for not wearing them. ——Carol Harvey ■Cyclists jump on and off pavements (which are meant for pedestrians), ride at speed along the pavements, and think they have a special right to go through traffic lights when they are on red. I was almost knocked down recently by a cyclist riding on the pavement when there was a cycle lane right next to him. Other road users, including horse riders, manage to obey the rules so why not cyclists? It's about time they had to be registered and insured, so when they do hit a pedestrian or a vehicle, or cause an accident, at least they can be treated and there might be an opportunity to claim. ——JML Write to Viewpoints of the newspaper. 56. Michael Horan wrote the letter mainly to show that ___________. A. drivers should be polite to cyclists B. road accidents can actually be avoided C. sine pedestrians are a threat to road safety D. walking while using phones hurts one's eyes 57. Carol Harvey suggests that cyclists should ___________. A. be provided with enough roads B. be asked to ride on their own lanes C. be made to pay less tax for cycling D. be fined for laughing at policemen 58. What is a complaint of JML? A. Very few drivers are insured. B. Cyclists ride fast on pavements. C. Pedestrians go through red traffic lights. D. Horse riders disrespect other road users. 59. The underlined word "they" in the third letter refers to ___________. A. accidents B. vehicles C. pedestrians D. cyclists 60. The three letters present viewpoints on ___________. A. real source of road danger B. ways to improve road facilities C. measures to punish road offences D. increased awareness of road rules B In its early history, Chicago had floods frequently, especially in the spring, making the streets so muddy that people, horses, and carts got stuck. An old joke that was popular at the time went something like this: A man is stuck up to his waist in a muddy Chicago street. Asked if he needs help, he replies, "No, thanks. I've got a good horse under me." The city planner decided to build an underground drainage (排水 ) system, but there simply wasn't enough difference between the height of the ground level and the water level. The only two options were to lower the Chicago River or raise the city. An engineer named Ellis Chesbrough convinced me the city that it had no choice but to build the pipes above ground and then cover them with dirt. This raised the level of the city's streets by as much as 12 feet. This of course created a new problem: dirt practically buried the first floors of every building in Chicago. Building owners were faced with a choice: either change the first floors of their buildings into basements, and the second stories into main floors, or hoist the entire buildings to meet the new street level. Small wood-frame buildings could be lifted fairly easily. But what about large, heavy structures like Tremont Hotel, which was a six-story brick building? That's where George Pullman came in. He had developed some house-moving skills successfully. To lift a big structure like the Tremont Hotel, Pullman would place thousands of jackscrews (螺旋千斤 顶 ) beneath the building's foundation. One man was assigned to operate each section of roughly 10 jackscrews. At Pullman's signal each man turned his jackscrew the same amount at the same time, thereby raising the building slowly and evenly. Astonishingly, the Tremont Hotel stay open during the entire operation, and many of its guests didn't even notice anything was happening. Some people like to say that every problem has a solution. But in Chicago's early history, every engineering solution seemed to create a new problem. Now that Chicago's waste water was draining efficiently into the Chicago River, the city's next step was to clean the polluted river. 61. The author mentions the joke to show ______. A. horses were fairly useful in Chicago B. Chicago's streets were extremely muddy C. Chicago was very dangerous in the spring D. the Chicago people were particularly humorous 62. The city planners were convinced by Ellis Chesbrough to_______. A. get rid of the street dirt B. lower the Chicago River C. fight against heavy floods D. build the pipes above ground 63. The underlined word "hoist" in Paragraph 4 means "_______". A. change B. lift C. repair D. decorate 64. What can we conclude about the moving operation of the Tremont Hotel? A. It went on smoothly as intended. B. It interrupted the business of the hotel. C. It involved Pullman turning ten jackscrews. D. It separated the building from its foundation. 65. The passage is mainly about the early Chicago's ______. A. popular life styles and their influences B. environmental disasters and their causes C. engineering problems and their solutions D. successful businessmen and their achievements C Have your parents ever inspected your room to see if you cleaned it properly? Imagine having your entire houses, garage, and yard inspected at any time -- with no warning. Inspections were a regular part of lighthouse (灯塔) living, and a keeper's reputation depended on results. A few times each year, an inspector arrived to look over the entire light station. The inspections were supposed to be a surprise, but keeper sometimes had advance notice. Once lighthouses had telephones, keepers would call each other to warn that the inspector was approaching. After boats began flying special flags noting the inspector aboard, the keeper's family made it a game to see who could notice the boat first. As soon as someone spotted the boat, everyone would do last-minute tidying and change into fancy clothes. The keeper then scurried to put on his dress uniform and cap. Children of keepers remember inspectors wearing white gloves to run their fingers over door frames and windowsills looking for dust. Despite the serious nature of inspections, they resulted in some funny moments. Betty Byrnes remembered when her mother did not have time to wash all the dishes before an inspection. At the time, people did not have dishwashers in their homes. In an effort to clean up quickly, Mrs. Byrnes tossed all the dishes into a big bread pan, covered them with a cloth and stuck them in the oven. If the inspector opened the oven door, it would look like bread was baking. he never did. One day, Glenn Furst’s mother put oil on the kitchen floor just before the inspector entered their house. Like floor wax, the oil made the floors shiny and helped protect the wood. This time, though, she used a little too much oil. When the inspector extended his hand to greet Glenn's mother, he slipped on the freshly oiled surface. "He came across that floor waving his arms like a young bird attempting its first flight," Glenn late wrote. After he steadied himself, he shook Glenn's mother's hand, and the inspection continued as though nothing had happened. 66. What does Paragraph I tell us about the inspection at the light station? A. It was carried out once a year. B. It was often announced in advance. C. It was important for the keeper's fame. D. It was focused on the garage and yard. 67. The family began making preparations immediately after ___________. A. one of the members saw the boat B. a warning call reached the lighthouse C. the keeper put on the dress uniform and cap D. the inspector flew special flags in the distance 68. Mrs. Byrnes put the dishes in the oven because this would ___________. A. result in some fun B. speed up washing them C. make her home look tidy D. be a demand from the inspector 69. If the inspector had opened the oven door, he would have seen ___________. A. an empty pan B. many clean dishes C. pieces of baked bread D. a cloth covering something 70. The inspector waved his arms ___________. A. to try his best to keep steady B. to show his satisfaction with the floor C. to extend a warm greeting to Glenn's mother D. to express his intention to continue the inspection Part IV Writing (45 marks) Section A (10 marks) Directions: Read the following passage. Fill in the numbered blanks by using the information from the passage Write NO MORE THAN THREE WORDS for each answer. Not all print dictionaries are the same, as you will notice when you select one. To make a wise selection, you should know how to distinguish among three kinds of print dictionaries: pocket, desk, and unabridged. You should also know the copyright date of your dictionary, and check is special features. A pocket dictionary is small. Generally, it contains no more than 75000 entries, making it hardly to carry to class and efficient to use. However, a pocket dictionary doesn't contain enough entries to be adequate for college reference homework. In addition, the information about each word in a pocket dictionary is generally limited. A desk dictionary is medium sized, generally containing over 100,000 entries as well as extra features. For college work, you should own a current desk dictionary. An unabridged dictionary is a complete dictionary. Abridged dictionaries, such as pocket and desk dictionaries, are shortened. Because unabridged dictionaries contain nearly all English words, they are large and heavy. They are often used by schools and libraries. If the copyright date of your current dictionary shows that it was published five or more years ago, consider investing in a more recent edition. English is a dynamic language that admits new words and recognizes changes in meaning, spelling, and usage of familiar words. This is reflected in an up-to-date dictionary. In selecting a dictionary, check the features it offers besides vocabulary definitions. Many editions contain signs, symbols and foreign words. Some also contain CD-ROMs and access to special online features. 71. _____________ 72. _____________ 73. _____________ 74. _____________ 75. _____________ 76. _____________ 77. _____________ 78. _____________ 79. _____________ 80. _____________ Section B (10 mark.) Directions: Read the following passage. Answer the questions according to the information given in the passage. Walk Out of the Comfort Zone and Try New Things For most high school students, free periods are useless. From what I have seen, few do homework, instead many are on their phones and talking, making it impossible for those who actually want to do work to complete any. As a senior next year, I think extra periods should be used to take optional subjects. Our school offers many classes. Now is the time to experiment in different fields of study. We will never know if we are interested or talented in a subject if we don't try it. In my 8th grade, I was told that I had to take an art class as a graduation requirement; so in the 9th grade I took Studio and Art. One of the projects was to build a clay pot, but I built mine incorrectly, so it broke in the kiln (窑). I found out that I have no artistic ability at all, and now I know for sure that I Title: How to 71 a Print Dictionary Distinguish among three kinds pocket dictionary > small: with no more than 75,000 entries > 72 for college reference homework. > limited in 73 74 > medium-sized: with over 170,000 entries plus 75 > suitable for college work unabridged dictionary > complete: without shortening > 76 containing nearly all English words > suitable for 77 the latest edition:reflecting English as a dynamic language > 78 > recognizing changes in meaning, spelling, usage of familiar words · signs, symbols and foreign words · CD-ROMs · 80 special online features Know the copyright date 79 the special features do not want to be an artist. However, the class was one of my favorites that year. I was able to try new activities and test my ability. Walk out of our comfort zone and try new things! College is when we should focus on a specific major, but high school is when we have to figure it out. Half of all college students change their major at some point. By doing that hundreds of dollars are wasted on classes that they would have never needed to take. So use our extra periods to find out what we want to do in college. The classes we choose can impact us in future. Taking optional subjects will enrich our mind. It will also show colleges we are diverse students. 81. How should we use our extra periods in the author's opinion? (No more than 9 words) (2 marks) ___________________________________________________________________________________ 82. Why does the author think we should experiment in different fields of study? (No more than 17 words) (2 marks) ___________________________________________________________________________________ 83. Why did the clay pot show the author's lack of artistic ability? (No more than 10 words) (3 marks) ___________________________________________________________________________________ 84.According to the author, how will taking optional subjects impact up in the future? (No more than 13 words) (3 marks) ___________________________________________________________________________________ Section C (25 marks) Directions: Write an English composition according to the instructions given below. 请以老师当众表扬你为话题,用下面所给的句子开头,续写一篇英语短文。 As a student, I can tell you that there is nothing better than being praised by my teacher before my classmates. 注意: 1. 将所有句子写在答题卡上; 2. 续写词数不少于 120个; 3. 不能使用真实姓名和学校名称。 2015年普通高等学校招生全国统一考试(湖北卷) 英 语 本试题卷共 81题。全卷满分 150分。考试用时 120分钟。 ★祝考试顺利★ 注意事项: 1. 答卷前,先将自己的姓名、准考证号填写在试题卷和答题卡上,并将准考证号条形码粘贴 在答题卡上的指定位置。用统一提供的 2B铅笔将答题卡上试卷类型 A后的方框涂黑。 2. 选择题的作答:每小题选出答案后,用统一提供的 2B铅笔把答题卡上对应题目的答案标 号涂黑。写在试题卷、草稿纸和答题卡上的非答题区域均无效。 3. 完成句子和短文写作题的作答:用统一提供的签字笔直接答在答题卡上对应的答题区域 内。写在试题卷、草稿纸和答题卡上的非答题区域均无效。 4. 考试结束后,请将本试题卷和答题卡一并上交。 第一部分:听力(共两节,满分 30分) 做题时,现将答案标在试卷上,录音内容结束后,你将有两分钟的时间将试卷上的答案转涂 到答题卡上。 第一节(共 5小题;每小题 1.5分,满分 7.5分) 听下面 5段对话。每段对话后有一个小题,从题中所给的 A、B、C三个选项中选出最佳选项,并 标在试卷的相应位置。听完每段对话后,你都有 10秒钟的时间来回答有关小题和阅读下一小题。 每段对话仅读一遍。 例: How much is the shirt? A. £ 19.15 B. £ 9.15 C. £ 9.18 答案:B 1. What does Tom plan to do next week? A. To make a robot. B. To enter a competition. C. To surprise the woman. 2. What is the woman? A. A waitress. B. A tour guide. C. A saleswoman. 3. What is the woman’s problem? A. She forgot her seat number. B. She got on the wrong train. C. She lost her train ticket. 4. Why has the man stopped exercising? A. He is busy moving house. B. He lives far away from the gym. C. He dislikes going to the gym alone. 5. How is Janet probably feeling now? A. Disappointed. B. Confused. C. Stressed. 第二节(共 15小题;每小题 1.5分,满分 22.5分) 听下面 5段对话或独白。每段对话或独白后有几个小题,从题中所给的 A、B、C三个选项中 选出最佳选项,并标在试卷的相应位置。听每段对话或独白前,你将有时间阅读各个小题,每小 题 5秒钟;听完后,各小题将给出 5秒钟的作答时间。每段对话或独白读两遍。 听第 6段材料,回答第 6、7题。 6. How long is the woman going to stay at the hotel? A. Two nights. B. Three nights. C. Four nights. 7. Why are the prices for the two rooms different? A. The rooms are different in size. B. The rooms have different views. C. The rooms are of different shapes. 听第 7段材料,回答第 8、9题。 8. How did the man come to the place? A. By car. B. On foot. C. By train. 9. Why is the man here? A. To have an interview. B. To discuss traffic problems. C. To share his work experience. 听第 8段材料,回答第 10至 12题。 10. Where are the speakers? A. In a bank. B. On a lake. C. At a hotel. 11. Why does David’s father want to spend Christmas in the countryside? A. He can enjoy the lake view. B. He can be freed from housework. C. He can have some peace and quiet. 12. Why is David unhappy? A. He can’t attend the party. B. He isn’t allowed to skate. C. He has to cook for his parents. 听第 9段材料,回答第 13至 16题。 13. How does Steven describe his music? A. It’s hard rock. B. It’s folk music. C. It’s modern pop. 14. Why does Steven love Australian audiences? A. They never throw things at him. B. They buy more of his records. C. They are more friendly. 15. What’s Steven’s attitude towards people downloading his music? A. He doesn’t mind. B. He feels upset. C. He is puzzled. 16. What advice does Steven give to songwriters? A. Listen to more music. B. Stick to their own style. C. Make use of the Internet. 听第 10段材料,回答第 17至 20题。 17. Which can best describe a tension headache? A. It occurs repeatedly. B. It lasts over a month. C. It causes severe pain. 18. Which is a symptom of tension headaches? A. Loss of consciousness. B. Increased blood pressure. C. A pain at the back of the head. 19. What is the cause of rebound headaches? A. Stressful events. B. Tension in the muscles. C. Too much use of pain medicine. 20. Why is a headache diary helpful for the treatment? A. It helps one find the cause of his headaches. B. It reminds one to take medicine on time. C. It helps remove constant pain. 第二部分:词汇知识运用(共两节,满分 30分) 第一节:多项选择(共 10小题;每小题 1分,满分 10分) 从 A、B、C、D四个选项中,选出可以填入空白处的最佳选项,并在答题卡上将该项涂黑。 例:To make members of a team perform better, the trainer first of all has to know their __________ and weaknesses. A. strengths B. benefits C. techniques D. values 答案:A 21. When he was running after his brother, the boy lost his __________ and had a bad fall. A. balance B. chance C. memory D. place 22. He gave himself a new name to hide his __________ when he went to carry out the secret task. A. emotion B. talent C. identity D. treasure 23. The team are working hard to __________ the problem so that they can find the best solution. A. face B. prevent C. raise D. analyze 24. In order not to be heard, she pointed her finger upwards to __________ that someone was moving about upstairs. A. whisper B. signal C. declare D. complain 25. There is no doubt that this candidate’s advantage __________ his ability to communicate with foreigners in English. A. leaves out B. goes against C. lies in D. makes up 26. Don’t worry. I’m sure your missing glasses will __________ sooner or later. A. stand out B. break up C. get out D. turn up 27. Even though the conference hall is near his apartment, he has to hurry a little if he wants to be __________. A. accurate B. punctual C. efficient D. reasonable 28. I don’t think what he said is __________ to the topic we are discussing . He has missed the point. A. faithful B. parallel C. relevant D. similar 29. The girl used to be shy, but is __________ getting active in group work and is more willing to express herself. A. gradually B. usually C .previously D. merely 30. This meeting room is a non-smoking area. I would like to warn you __________ that if you smoked here you would be fined. A. in advance B. in detail C. in total D. in general 第二节:完形填空(共 20小题;每小题 1分,满分 20分) 阅读下面短文,从短文后各题所给的四个选项(A、B、C和 D)中,选出可以填入空白处 的最佳选项,并在答题卡上将该项涂黑。 “Daily Star, sir” called Jason, carrying some newspapers under his arm. The little boy had been running up and down the street, but there were still twenty 31 left. His voice was almost gone and his heart was 32 . The shops would soon close, and all the people would go home. He would have to go home too, carrying the papers 33 money. He had hoped to sell more papers tonight to make more money to buy a 34 for his mother and some seeds for his bird. That was why he had bought the papers with all his money. He 35 as he thought of his failure to sell all his papers. “You don’t know the 36 of selling papers. You must shout, “Hot news! Bomb bursting!” another newsboy Chad told Jason. “ 37 it’s not in the paper at all,” replied Jason. “Just run away quickly 38 they have time to see, and you’ll 39 out and get your money,”Chad said. It was a new 40 to Jason. He thought of his bird with no 41 and the cake he wanted to buy for his mother, but was 42 that he would not tell a lie. Though he was 43 a poor newsboy, he had been 44 some good things. The next afternoon Jason went to the office for his papers 45 . Several boys were crowding around Chad, who declared with a 46 smile that he sold six dozen the day before. He added that Jason 47 money because he would not tell a lie. The boy 48 at Jason. “You wouldn’t tell a lie yesterday, my boy?” A gentleman at the office came up and patted Jason’s shoulder ___49___.”You’re just the boy I am looking for.” A week later Jason started his new 50 . He lost sale of twenty papers because he would not tell a lie, but got a well-paid job because he told the truth. 31. A. shops B. coins C. people D. papers 32. A. open B. heavy C. pure D. weak 33. A.instead of B. in return for C. regardless of D. in exchange for 34. A. cup B.card C. comb D. cake 35. A. gave in B. broke down C. got away D. showed up 36. A. difficulty B. process C. goal D. secret 37. A. And B. But C. For D. So 38. A. before B. since C. though D. unless 39. A. call B. drop C. sell D. reach 40. A. edition B. idea C. policy D. task 41. A. bread B. insects C. seeds D. water 42. A. concerned B. amazed C. excited D. determined 43. A. still B. already C. just D. also 44. A. taught B. handed C. awarded D. allowed 45. A. at once B. by chance C. as usual D. on purpose 46. A. proud B. gentle C. warm D. polite 47. A. borrowed B. lost C. made D. saved 48. A. laughed B. shouted C. nodded D. started 49. A. bravely B. grateful C. fondly D. modestly 50. A. duty B. business C. job D. method 第三部分:阅读理解(共 20小题;每小题 2分,满分 40分) 阅读下列短文,从每篇短文后各题所给的四个选项(A、B、C和 D)中,选出最佳选项,并 在答题卡上将该项涂黑。 A “I see you’ve got a bit of water on your coat,” said the man at the petrol station. “Is it raining out there?””No, it’s pretty nice,” I replied, checking my sleeve. “Oh, right. A pony(马驹) bit me earlier.” As it happened, the bite was virtually painless: more the kind of small bite you might get from a naughty child. The pony responsible was queuing up for some ice cream in the car park near Haytor, and perhaps thought I’d jumped in ahead of him. The reason why the ponies here are naughty is that Haytor is a tourist-heavy area and tourists are constantly feeding the ponies foods, despite sighs asking them not to. By feeding the ponies, tourists increase the risk of them getting hit by a car, and make them harder to gather during the area’s annual pony drift(迁移). The purpose of a pony drift is to gather them up so their health can be checked, the baby ones can be stooped from feeding on their mother’s milk, and those who’ve gone beyond their limited area can be returned to their correct area. Some of them are also later sold, in order to limit the number of ponies according to the rules set by Natural England. Three weeks ago, I witnessed a small near-disaster a few mils west of here. While walking, I noticed a pony roll over on his back. “Hello!” I said to him, assuming he was just rolling for fun, but he was very still and, as I got closer, I saw him kicking his legs in the air and breathing heavily. I began to properly worry about him. Fortunately, I managed to get in touch with a Dartmoor’s Livestock Protection officer and send her a photo. The officer immediately sent a local farmer out to check on the pony. The pony had actually been trapped between two rocks. The farmer freed him, and he began to run happily around again. Dartmoor has 1,000 or so ponies, who play a critical role in creating the diversity of species in this area. Many people are working hard to preserve these ponies, and trying to come up with plans to find a sustainable(可持续的) future for one of Dartmoor’s most financially-troubled elements. 51. Why are tourists asked not to feed the ponies? A. To protect the tourists from being bitten B. To keep the ponies off the petrol station C. To avoid putting the ponies in danger D. To prevent the ponies from fighting 52. One of the purposes of the annual pony drift is ___________. A. to feed baby ponies on milk B. to control the number of ponies C. to expand the habitat for ponies D. to sell the ponies at a good price 53. What as the author’s first reaction when he saw a pony roll on its back? A. He freed it from the trap B. He called a protection officer C. He worried about it very much D. He thought of it as being naughty 54. What does the author imply about the preservation of Dartmoor’s ponies? A. It lacks people’s involvement. B. It costs a large amount of money C. It will affect tourism in Dartmoor. D. It has caused an imbalance of species B What Theresa Loe is doing proves that a large farm isn’t a prerequisite for a modern grow-your-own lifestyle. On a mere 1/10 of an acre in Los Angeles, Loe and her family grow, can(装罐) and preserve much of the food they consume. Loe is a master food preserver, gardener and canning expert. She also operates a website, where she share her tips and recipes, with the goal of demonstrating that everyone has the ability to control what’s on their plate. Loe initially went to school to become an engineer, but she quickly learned that her enthusiasm was mainly about growing and preparing her own food. “I got into cooking my own food and started growing my own herbs(香草) and foods for that fresh flavor,” she said. Engineer by day, Loe learned cooking at night school. She ultimately purchased a small piece of land with her husband and began growing their own foods. “I teach people how to live farm-fresh without a farm,”Loe said. Through her website Loe emphasizes that “anybody can do this anywhere.” Got an apartment with a balcony(阳台)? Plant some herbs. A window? Perfect spot for growing. Start with herbs, she recommends, because “they’re very forgiving.” Just a little of the herbs“can take your regular cooking to a whole new level,” she added. “I think it’s a great place to start.” Then? Try growing something from a seed, she said, like a tomato or some tea. Canning is a natural extension of the planting she does. With every planted food,Loe noted, there’s a moment when it’s bursting with its absolute peak flavor. “I try and keep it in a time capsule in a canning jar.”Loe said. “Canning for me is about knowing what’s in your food, knowing where it comes from.” In addition to being more in touch with the food she’s eating, another joy comes from passing this knowledge and this desire for good food to her children:“Influencing them and telling them your opinion on not only being careful what we eat but understanding the bigger picture,” she said, “that if we don’t take care of the earth, no one will.” 55. The underlined word “prerequisite”(Para. 1) is closest in meaning to “__________”. A. recipe B. substitute C. requirement D. challenge 56. Why does Loe suggest starting with herbs? A. They are used daily. B. They are easy to grow. C. They can grow very tall. D. They can be eaten uncooked. 57. According to Loe, what is the benefit of canning her planted foods? A. It can preserve their best flavor. B. It can promote her online sales. C. It can better her cooking skills. D. It can improve their nutrition. 58. What is the “the bigger picture”(Para. 6) that Loe wishes her children to understand? A. The knowledge about good food. B. The way to live a grow-our-own life. C. The joy of getting in touch with foods. D. The responsibility to protect our earth. C Hilversum is a medium-sized city between the major cities of Amesterdam and Utrecht in theGooi area of North Holland, the Netherlands. Unlike most of the Netherlands, Hilversum is actually in a hilly area with the soil mostly consisting of sand. Once called the Garden of Amsterdam, it still attracts travelers to come over to cycle and walk through the surrounding forests. They visit it for a relaxing day off from the urban madness. For Dutch people, Hilversum is all about textile(纺织)and media industries, and modern architecture. In history, Hilversum was largely an agricultural area. Daily life was marked by farming, sheep raising and wool production. A railway link to Amsterdam in 1874 attracted rich traders from Amsterdam to Hilversum. They built themselves large villas(别墅)in the wooded surroundings of the town. One of the families moving in was the Brenninkmeijers, currently the wealthiest family of the Netherlands. They moved in after big success in the textile industry and aided a substantial textile industry in Hilversum. But the textile boom lasted only several decades. The last factory closed in the 1960s. The change to a media economy started in 1920, when the Nederlandse Seintoestellen Fabriek (NSF) established a radio factory in Hilversum.Most radio stations settled in the large villas in the leafy areas of the town. Television gave another push to the local economy. Hilversum became the media capital of the Netherlands, and Dutch television stars moved into the leafy neighborhoods surrounding the town. In the early 1900s, modern architects W. M. Dudok and J. Duiker placed hundreds of remarkable buildings in Hilversum. These modern architectural masterpieces(杰作) are so many that Hilversum almost feels like an open air museum. Dudok alone shaped most of 20th century Hilversum and approximately 75buildings still bear his unique characteristics. His masterpiece, Hilversum Town Hall,was built in 1928-1931. It has wide international fame and is included in many architecture textbooks. The building has remarkable shape and looks like a combination of “blocks”. Actually, one may start his journey of modern architecture by walking or biking the W.M. Dudok Architectural Route in Hilversum. 59. Hilversum is different form most of the Netherlands in that __________. A. it has a large population B. it is cut off form big cities C. is has many beautiful gardens D. it is in a hilly area with sandy soil 60. What was the greatest contribution of the Brennikmeijers to Hilversum? A. Building a railway link to Amsterdam. B. Helping its textile industry to develop. C. Constructing large villas for the poor. D. Assisting its agricultural industry. 61. The beginning of the media industry in Hilversum was marked by the establishment of __________. A. a radio factory B. the media capital C. a radio station D. a TV station 62. What is known about W.M. Dudok’s Hilversum Town Hall? A. It consists of approximately 75 buildings. B. It looks like an open air museum in the city. C. It is a classic example in architecture textbooks. D. It has shaped most of 20th century Hilversum. D The oddness of life in space never quite goes away. Here are some examples. First consider something as simple as sleep. Its position presents its own challenges. The main question is whether you want your arms inside or outside the sleeping bag. If you leave your arms out, they float free in zero gravity, often giving a sleeping astronaut the look of a funny ballet (芭蕾) dancer. “I’m an inside guy,” Mike Hopkins says, who returned from a six-month tour on the International Space Station.“I like to be wrapped up.” On the station, the ordinary becomes strange. The exercise bike for the American astronaut has no handlebars. It also has no sear. With no gravity, it’s just as easy to pedal violently. You can watch a movie while you pedal by floating a microcomputer anywhere you want. Without gravity to help circulate air, the carbon dioxide you exhale(呼气)has a tendency to form an invisible(隐形的)cloud around your head. You can end up with what astronauts call a carbon-dioxide headache. Leroy Chiao, 54, an American retired astronaut after four flights, describes what happens even before you float out of your seat. “Your inner ear thinks you’re falling. Meanwhile your eyes are telling you you’re standing straight. That can be annoying—that’s why some people feel sick.” Within a couple of days—truly terrible days for some—astronauts’ brains learn to ignore the panicky signals from the inner ear, and space sickness disappears. Space travel can be so delightful but at the same time invisibly dangerous. For instance, astronauts lose bone mass. That’s why exercise is considered so vital that National Aeronautics and Space Administration(NASA) puts in right on the workday schedule. The focus on fitness is as much about science and the future as it is about keeping any individual astronaut healthy. NASA is worried about two things: recovery time once astronauts return home, and, more importantly, how to maintain strength and fitness for the two and a half years or more that it would take to make a round-trip to Mars. 63. What is the major challenge to astronauts when they sleep in space? A. Deciding on a proper sleep position. B. Choosing a comfortable sleeping bag. C. Seeking a way to fall asleep quickly. D. Finding a right time to go to sleep. 64. The astronauts will suffer from a carbon-dioxide headache when __________? A. they circle around on their bikes B. they use microcomputers without a stop C. They exercise in one place for a long time D. they watch a movie while pedaling 65. Some astronauts feel sick on the station during the first few days because __________? A. their senses stop working B. they have to stand up straight C. they float out of their seats unexpectedly D. their brains receive contradictory messages 66. One of NASA’s major concerns about astronauts is __________. A. how much exercise they do on the station B. how they can remain healthy for long in space C. whether they can recover after returning home D. whether they are able to go back to the station E Science has a lot of uses. It can uncover laws of nature, cure diseases, make bombs, and help bridges to stand up. Indeed science is so good at what it does that there’s always a temptation(诱惑) to drag it into problems where it may not be helpful. David Brooks, author of The Social Animal: The Hidden Sources of Love, Character,and Achievement, appears to be the latest in a long line of writers who have failed to resist the temptation. Brooks gained fame for several books. His latest book The Social Animal, however, is more ambitious and serious than his earlier books. It is an attempt to deal with a set of weighty topics. The book focuses on big questions: What has science revealed about human nature? What are the sources of character? And why are some people happy and successful while others aren’t? To answer these questions, Brooks surveys a wide range of disciplines(学科). Considering this, you might expect the book to be a dry recitation of facts. But Brooks has structured his book in an unorthodox(非常规的), and perhaps unfortunate, way. Instead of introducing scientific theories, he tells a story, within which he tries to make his points, perhaps in order to keep the reader’s attention.So as Harold and Erica, the hero and heroine in his story, live through childhood, we hear about the science of child development and as they begin to date we hear about the theory of sexual attraction. Brooks carries this through to the death of one of his characters. On the whole, Brooks’s story is acceptable if uninspired. As one would expect, his writing is mostly clear and, to be fair, some chapters stand out above the rest. I enjoyed, for instance, the chapter in which Harold discovers how to think on his own. While Harold and Erica are certainly not strong or memorable characters, the more serious problems with The Social Animal lie esle where. These problems partly involve Brooks’s attempt to translate his tale into science. 67. The author mentions the functions of science at the beginning of the passage to __________. A. illustrate where science can be applied B. demonstrate the value of Brooks’s new book C. remind the reader of the importance of science D. explain why many writers use science in their works 68. According to the author, which of the following could be a strength of the book? A. Its strong basis. B. Its convincing points. C. Its clear writing. D. Its memorable characters. 69. What is the author’s general attitude towards the book? A. Contradictory. B. Supportive. C . Cautious. D. Critical. 70. What is the author likely to write about after the last paragraph? A. Problems with the book. B. Brooks’s life experience. C. Death of the characters. D. Brooks’s translation skills. 第四部分:书面表达(共两节,满分 50分) 第一节:完成句子(共 10 小题;每小题 2 分,满分 20 分) 阅读下列各小题,根据汉语提示,用句末括号内的英语单词完成句子,并将答案写在答题卡 上的相应题号后。 例:_________________________ was that the young player performed extremely well in the table tennis tournament. (delight) 令球迷欣喜的是那位年轻的球员在乒乓球锦标赛中表现得极为出色。 答案:What delighted the fans/made the fans delighted 71. If she had been aware that the mushroom were poisonous, she _________________________ them for dinner.(pick) 要是知道这些蘑菇有毒,她不会采来做晚餐了。 72. _________________________ the beef being cooked on the stove,the mother pictured the whole family having meals together.(look) 看着炉子上炖着的牛肉,妈妈想象着一家人在一起吃饭的画面。 73. The director, _________________________a dorm-room in the Film Academy, has already make four films.(share) 这位导演已经执导了四部电影,在电影学院求学时杰克与他同住在一间寝室。 74. Into the complete silence of the waiting class _________________________, “Good morning, children.”(come) 在全班同学静静的等待中传来了老师甜甜的嗓音:“孩子们,早上好”。 75. _________________________ will not make any difference to our arrangements.(say) 无论她说什么,都是不会对我们的安排有影响。 76. We must act as quickly as possible now. Just tell us _________________________ the task or not.(undertake). 现在我们必须采取行动。你就告诉我们,你能否承担这项任务吧。 77. The professor was delighted to find that two thirds of the project _________________________ by the students independently.(finish) 教授高兴地发现学生们已独立完成了该项目三分之二的工作。 78. _________________________ that they found the long lost sword of the Ming Dynasty. (be) 就是在这个湖里,他们发现了失踪已久的明代宝剑。 79. _________________________ to only a few people here,his reputation abroad is very great.(know). 虽然这里只有几个人知道他,但他在国外的名气却很大。 80. Many bad habits can be especially difficult to cure because they are likely _______________ __________ at a very young age. (from). 许多坏习惯特殊气味难以纠正,因为它们可能是人们在年幼时就形成了的。 第二节 短文写作 请根据以下提示,并结合具体事例,有英语写一篇短文。 Questioning is a bridge to learning. When you begin to doubt something and search for an answer,you will learn. 注意:1.无须写标题; 2.除诗歌外,文体不限; 3.内容必须结合你生活中的具体事例; 4.文中不得透露个人姓名和学校名称; 5.词数不少于 120,如引用提示语则不计入总词数。 2015年普通高等学校招生全国统一考试(四川卷) 英 语 本试题卷分第Ⅰ卷(选择题)和第Ⅱ卷(非选择题)。 满分 150 分。考试时间 120 分钟。考生作答时,须将答案答在答题卡上,在本试题卷、草纸上答题不小。考试结 束后,将本试题卷和答题卡一并交回。 第Ⅰ卷 (选择题 共 90分) 注意事项: 1. 必须使用 2B铅笔在答题卡上将所选答案对应的标号涂黑 2. 第Ⅰ卷共两部分,共计 90分。 第一部分 英语知识运用(共两节,共 40分) 第一节 单项填空 从 A、B、C、D四个选项中,选出可以填入空白处的最佳选项,并在答题卡上将该项涂黑。 (共 10小题;每小题 1分,共 10分) 1. —Sorry, I forgot to lock the door. —__________ Mike can do it later.. A. No way B. Take your time C. Nothing serious D. You're welcome 2. You __________ be careful with the camera. It costs! A. must B. may C. can D. will 3. The books on the desk, __________ covers are shiny,are prizes for us. A. which B. what C. whose D. that 4. More expressways __________ in Sichuan soon to promote the local economy. A .are being built B. will be built C. have been built D. had been built 5. Brian is gifted in writing music;he is very likely to be __________ Beethoven. A. a B. an C. the D. 不填 6. There is only one more day to go __________ your favorite music group play live. A. since B. until C. when D. before 7. Andy is content with the toy.It is __________ he has ever got. A. a better B. the better C. a best D. the best 8. The exhibition tells us __________ we should do something to stop air pollution. A. where B. why C. what D. which 9. Little Tom sat __________ watching the monkey dancing in front of him. A. amaze B. amazing C. amazed D. to amaze 10. Niki is always full of ideas,but __________ is useful to my knowledge. A. nothing B. no one C. neither D. none 第二节 完形填空 阅读下面短文,从短文后各题所给的四个选项(A、B、C、D)中,选出可以填入空白处的 最佳选项,并在答题卡上将该项涂黑。(共 20小题;每小题 1.5分,共 30分) My previous home had a stand of woods behind it and many animals in the backyard. That first year, I 11 feeding peanuts to the blue jays, then the squirrels. The squirrels had no ___12___ coming up right to me for them. As the months went by, the rabbits saw that I was no 13 and didn’t escape When I threw carrot slices(薄片), they even came for a nibble(啃). Slowly they came to ___14___ me, and by the end of the year they were eating out of my hand. That second year, the rabbits 15 me,and one would even sit up for slices!While I was feeding them, I 16 that a groundhog who used to run away was now talking an ___17___ interest in this food situation.I carefully extended a long 18 , with a keen eye on those teeth,and ___19___, there were times I would have the groundhog sitting next to a rabbit,both munching(津津有 味的咀嚼 ) On carrots. A few months later, while 20 , she would even turn her back to me. ___21___ when she was facing away,I reached out and 22 scratched(搔)her back with my finger, She didn’t move. By year three, the rabbits and the groundhog were back.The ground hog 23 didn’t have a problem with me scratching her back,and I got an ides,I’d always 24 , while slicing up carrots,that the end looked like a cap. 25 one day,just to see what she would do ,I gently ___26___ one on top of the groundhog’s head.Again,not a 27 , The next time, I had my camera ready to record what you see here,one of several dozen such pictures, 28 she had a slice to eat,she never 29 the one of her head.It was a fair 30 —I got a pleasure,and she had yet another tasty treat. 11. A. avoided B. started C. canceled D. suggested 12. A. business B. fun C. problem D. privilege 13. A. help B. cheat C. threat D. exception 14. A. trust B. miss C. admire D. appreciate 15. A. feared B. ignored C. discovered D. remembered 16. A. proved B. decided C. noticed D. understood 17. A. extreme B. increasing C. additional D. inspiring 18. A. squirrel B. rabbit C. peanut D. carrot 19. A. before long B. long ago C. over and over D. all over again 20. A. eating B. playing C. sitting D. sleeping 21. A. Next B. Once C. Soon D. Lately 22. A. carefully B. suddenly C. violently D. patiently 23. A. also B. thus C. just D. still 24. A. thought B. doubted C. admitted D. recognised 25. A. While B. Or C. So D. For 26. A. fixed B. placed C. hung D. kept 27. A. tremble B. move C. delay D. hesitation 28. A. Even if B. Ever since C. As far as D. So long as 29. A. welcomed B. required C. bothered D. expected 30. A. trade B. competition C. task D. affair 第二部分 阅读理解(共两节,共 50分) 第一节 阅读下列短文,从每题所给的四个选项(A、B、C、D)中,选出最佳选项,并在答题 卡上将该项涂黑。(共 20小题;每小题 2分,共 40分) 31. The card above is __________. A. a ticket B. a postcard C. an invitation D. an advertisement 32. The party is for __________. A. a birthday B. the Queen C. bee watching D. the National Day 33. According to the card, if you are unable to go ,you can __________. A.return the card B.visit the Museum C.ignore the message D.contact Alice’mother B Nothing could stop Dad. After he was put on disability for a bad back, he bought a small farm in the country, just enough to grow food for the family. He planted vegetables, fruit trees and even kept bees for honey. And every week he cleaned Old Man McColgin's chicken house in exchange for manure(肥料). The Smell really burned the inside of your nose. When we complained about the terrible smell, Dad said the stronger the manure, the healthier the crops, and he was right. For example, just one of his cantaloupes filled the entire house with its sweet smell, and the taste was even sweeter. As the vegetables started coming in, Dad threw himself into cooking. One day, armed with a basket of vegetables, he announced he was going to make stew(炖菜).Dad pulled out a pressure cooker and filled it up with cabbages, eggplants, potatoes, corns, onions and carrots. For about half an hour. the pressure built and the vegetables cooked. Finally, Dad turned off the stove, the pot began to cool and the pressure relief valve sprayed out a cloud of steam. If we thought Dad's pile of chicken manure。was bad, this was 10 times worse. When Dad took off the lid, the smell nearly knocked us out. Dad carried the pot out and we opened doors and windows to air out the house. Just how bad was it? The neighbors came out of their houses to see if we had a gas leak! Determined, Dad filled our plates with steaming stew and passed them around it didn’t look that bad, and after the first wave had shut down my ability to smell, it didn’t offend the nose so much, edible, and we drank up every last drop of soup. 34. Why did Dad clean Old Man Mocolgin’s chicken house regularly? A. To earn some money for the family. B. To collect manure for his crops. C. To get rid of the terrible smell. D. To set a good example to us. 35. What can we infer about Dad’s stew? A. It is popular among the neighbors. B. It contains honey and vegetables. C. It looks very wonderful. D. It tastes quite delicious. 36. What does the underlined word “offend” in the last paragraph mean? A. To attract B. To upset C. To air D. To shut 37. What can we learn about Dad form the text? A. He is an experienced cook. B. He is a troublesome father. C. He has a positive attitude to life. D. He suffers a lot from his disability. C Across Britain,burnt toast will be served to mothers in bed this morning as older sons and daughters to deliver their supermarket bunches of flowers,But,according to a new study,we should be placing a higher value on motherhood all year. Mothers have long known that their home worked was just heavy as paid work.Now,the new study has shown that if they were paid for their parental labours,they would earn as much as$172,000 a year. The study looked at the range of jobs mothers do,as well as the hours they are working,to determine the figure.This would make their yearly income $3000 more than the Prime Minister earns. By analysing the numbers,it found the average mother works 119 hours a week,40 of which would usually be paid at a standard rate 79 hours as overtime.After questioning 1,000 mothers with children under 18,it found that ,on most days,mums started their routine work at 7am and finished at around 11pm. To calculate just how much mothers would earn from that labour, it suggested some of the roles that mums could take on,including housekeeper,part-time lawyer,personal trainer and entertainer.being a part-time lawyer,at $ 48.98 an hour,would prove to be the most profitable of the “mum jobs”, with psychologist(心理学家)a close second. It also asked mothers about the challenges they face,with 80 percent making emotional(情感 的) demand as the hardest thing about motherhood. Over a third of mums felt they needed more training and around half said they missed going out with friends. The study shoes mothers matter all year long and not just on, Mother’s Day.The emotional, physical and mental energy mothers devote to their children can be never-ending, but children are also sources of great joy and happiness. Investing(投入)in time for parenting and raising relationships is money well spent. 38. How much would a mother earn a year if working as the Prime Minister? A. £30,000. B. £142,000. C. £172,000. D. £202,000. 39. The biggest challenge for most mothers is from __________. A. emotional demand B. low pay for work C. heavy workload D. lack of training 40. What is stressed in the last paragraph? A. Mothers’ importance shows in family all year long. B. The sacrifices mothers make are huge but worthwhile. C. Mothers’ devotion to children can hardly be calculated. D. Investing time in parenting would bring a financial return. 41. What can we conclude from the study? A. Mothers’ working hours should be largely reduced. B. Mothers should balance their time for work and rest. C. Mothers’ labour is of a higher value than it is realised. D. Mothers should be freed from housework for social life. D Their cheery song brightens many a winter's day. But robins are in danger of wearing themselves out by singing too much. Robins are singing all night 一 as well as during the day,British-based researchers say. David Dominoni, of Glasgow University, said that light from street lamps, takeaway signs and homes is affecting the birds' biological clock, leading to them being wide awake when they should be asleep. Dr Dominoni, who is putting cameras inside nesting boxes to track sleeping patterns, said lack of sleep could put the birds' health at risk.His study shows that when robins are exposed to light at night in the lab, it leads to some genes being active at the wrong time of day. And the more birds are exposed to light, the more active they are at night. He told people at a conference, "There have been a couple of studies suggesting they are increasing their song output at night and during the day they are still singing. Singing is a costly behavior and it takes energy.So by increasing their song output, there might be some costs of energy." And it is not just robins that are being kept awake by artificial light. Blackoiids and seagulls are also being more nocturnal. Dr Dominoni said, "In Glasgow where I live, gulls are a serious problem.I have people coming to me saying `You are the bird expert. Can you help us kill these gulls?'.During the breeding(繁殖 )season,between April and June,they are very active at night and very noisy and people can't sleep." Although Dr Dominoni has only studied light pollution,other research concluded that robins living in noisy cities have started to sing at night to make themselves heard over loud noise. However, some birds thrive(兴旺)in noisy environments. A study from California Polytechnic University found more hummingbirds in areas with heavy industrial machinery. It is thought that they are capitalising on their predators(天敌)fleeing to quieter areas. 42. According to Dr Dominoni's study, what causer robins to sing so much? A. The breeding season. B. The light in modem C. The dangerous environment. D. The noise from heavy machinery. 43. What is the researchers' concern over the increase of birds' song output? A. The environment might be polluted. B. The birds' health might be damaged. C. The industry cost might be increased. D.The people's hearing might be affected. 44. What does the underlined word "nocturnal" in Paragraph 5 mean? A. Active at night. B. Inactive at night. C. Active during the day. D. Inactive during the day. 45. Why do some birds thrive in noisy environments? A. Because there are fewer dangers. B. Because there is more food to eat. C. Because there is less light pollution D. Because there are more places to take shelter. E No one is sure how the ancient Egyptians built the pyramids near Cairo. But a new study suggests they used a little rock ‘n’ roll. Long-ago builders could have attached wooden pole s to the stones and rolled then across the sand, the scientists say. “Technically, I think what they’re proposing is possible,” physicist Daniel Bonn said. People have long puzzled over how the Egyptians moved such huge rocks. And there’s no obvious answer. On average, each of the two million big stones weighed about as much as a large pickup truck. The Egyptians somehow moved the stone blocks to the pyramid site from about one kilometer away. The most popular view is that Egyptian workers slid the blocks along smooth paths. Many scientists suspect workers first would have put the blocks on sleds(滑板). Then they would have dragged them along paths. To make the work easier, workers may have lubricated the paths either with wet clay or with the fat from cattle. Bonn has now tested this idea by building small sleds and dragging heavy objects over sand. Evidence from the sand supports this idea. Researchers found small amounts of fat, as well as a large amount of stone and the remains of paths. However , physicist Joseph West think there might have been a simpler way , led the new study . West said , “I was inspired while watching a television program showing how sleds might have helped with pyramid construction . I thought , ‘Why don’t they just try rolling the things ? ‘ ” A square could be turned into a rough sort of wheel by attaching wooden poles to its sides , he realized . That , he notes , should make a block of stone “a lot easier to roll than a square”. So he tried it. He and his students tied some poles to each of four sides of a 30-kilogram stone block.That action turned the block into somewhat a wheel.Then they placed the block on the ground. They wrapped one end of a rope around the block and pulled.The researchers found they could easily roll the block along different kinds of paths.They calculated that rolling the block required about as much force as moving it along a slippery(滑的) path. West hasn’t tested his idea on larger blocks,but he thinks rolling has clear advantages over sliding.At least,workers wouldn’t have needed to carry cattle fat or water to smooth the paths. 46. It’s widely believed that the stone blocks were moved to the pyramid site by __________. A. rolling them on roads B. pushing them over the sand C. sliding them on smooth paths D. dragging them on some poles 47. The underlined part “lubricated the paths” in Paragraph 4 means __________. A. made the path wet B. made the path hard C. made the path wide D. made the path slippery 48. What does the underlined word “it”in Paragraph 7 refer to? A. Rolling the blocks with poles attached. B. Rolling the blocks on wooden wheels. C. Rolling poles to move the blocks. D. rolling the blocks with fat. 49. Why is rolling better than sliding according to West ? A. Because more force is needed for sliding. B. Because rolling work can be done by fewer cattle. C. Because sliding on smooth road is more dangerous. D. Because less preparation on path is needed for rolling. 50. What is the text mainly about ? A. An experiment on ways of moving blocks to the pyramid site. B. An application of the method of moving blocks to the pyramid site. C. An argument about different methods of moving blocks to the pyramid site. D. An introduction to a possible new way of moving blocks to the pyramid site. 第二节 根据对话内容,从对话后的选项中选出能填入空白处的最佳选项,并在答题卡上将该项涂 黑。选项中有两项多余选项。(共 5小题;每小题 2分,共 10分) A: I need to lose some weight. My clothes don’t fit anymore. 51 B: Well, instead of watching TV all day, you could do more exercise. A: 52 B: Like cycling, or you could work out at the gym. A: I don’t have time. 53 B: OK, then you could change your diet. 54 A: You mean no more hamburgers! Oh no! B: OK. 55 第 II卷 (非选择题 共 60分) 注意事项: 1.必须使用 0.5毫米黑色墨迹签字笔在答题卡上题目所指示的答题区域内作答, A. Like what? B. I’m too busy C. What should I do? D. Some more hamburgers? E. Buy some larger clothes then. F. That’s a good idea. Thanks a lot. G. Eat something healthier, like fruit. 答题在试题卷上无效 2.第 I卷共三节,共计 60分。 第三部分 写作(共三节,满分 60分) 第一节 阅读表达:(共计 5 小题每小题 2 分,共 10 分) 阅读下面短文,并用英语回答问题,请清楚问题后的词数要求。 It was a hot summer day that found me running down the street with a dollar in my sweaty hand. The closer I got to Sam & Joe's Stationery Store, the more excited I was. After all, a dollar was a tidy sum for an 11-year-old boy in 1961. Out of breath, I swung open the door and got a blast of cold air that instantly cooled me off. I was in a place of pure happiness. Sam & Joe's had it all一 books, candy, toys, baseball cards,puzzles and games. I went directly to the bar and ordered a bottle of soda. There I sat, considering my next move. Books always came first.I picked out five titles.Next came the candy, rows and rows of it. I chose five candy bars, which brought my total to 25 cents. Then a bottle of cold orange juice, a bag of chips and a pack of Ted Williams baseball cards. After paying the clerk,I still had 5 cents. I hurried directly to my backyard, where an apple tree patiently waited for me. There, sitting in the shade against its strong trunk, I read my books, ate my candy and drank my orange juice. As I sat in my own personal heaven,I started thinking about having more.That's when a fantastic life-changing idea popped into head: If I could get more money, I could buy more books l liked, buy anything I wanted. That's how it all started. 56. How did the boy fell on his way to Sam & Joe’s? (no more than 5 words) 57. What does”a tidy sum”mean in Paragraph 1? (no more than 5 words) 58. What did the boy consider doing first after he ordered soda? (no more than 5 words) 59. What did the boy do while reading? (no more than 5 words) 60. Where was the boy’s”personal heaven”? (no more than 9 words) 第二节 短文改错(共 10小题;每小题 1.5分,共 15分) 下面短文中有 10处语言错误。请在有错误的地方增加、删除或修改某个单词。 增加:在缺词处加一个漏词符号(ʌ),并在其下面写出该加的词; 删除:把多余的词用斜线(\)划掉。 修改:在错的词下划一横线,并在该词下面写出修改后的词。 注意: 1. 每句不超过两个错误; 2. 每处错误及其修改均仅限一次; 3. 3.只允许修改 10出,多者(从第 11处起)不计分。 Hi, Janice, It's been a month since I came to this new school and I really want share with you some of the problems I have been experiencing. As I tell you last time, I made three new friend here. We hang out together during lunch and after school. We've been spending a lot of time sing in karaoke bars. It's been three Saturdays now and it really costs me many. And I started to see this as a time—wasting activity! In fact ,I don’t like to go anymore, so I’m afraid I’ll lose their friendship. How do you think I should do? If you are me ,would you talk to him? Please help with me and give me some advice. Grave 第三节 书面表达(共 35分) 假如你是李夏。你看到关国留学生 Sharon 在网上发贴,.希望有人能帮助她提高普通话(Mandarin) 水平,她可以教英语作为回报。请银据以下提示用英语给她写一封电于哪件。 1.表达给她提供帮助的意愿; 2.说明你能胜任辅导的理由; 3.给出讲好普通话的两点建议; 4.提出你学习英语的具体需求。 注意: 1. 词数 120左右,开头语已为你写好; 2. 可以适当增加细节,以使行文连贯; 3. 文中不得透漏个人姓名和学校名称; Hi Sharon, This is Li Xia. I learned from your post that you want to improve your Mandarm _______________ ____________________________________________________________________________________ ____________________________________________________________________________________ Li Xia 2015年普通高等学校招生全国统一考试(重庆卷) 英语试题卷 英语试题卷满分 120分。考试时间 l00分钟。 注意事项: 1. 答题前,务必将自己的姓名、准考证号填写在答题卡规定的位置上。 2. 答选择题时,必须使用 2B铅笔将答题卡上对应题目的答案标号涂黑。如需改动,用 橡皮擦擦干净后,再迭涂其它答案标号。 3. 答非选择题时,必须使用 0.5毫米黑色签字笔,将答案书写在答题卡规定的位置上。 4. 所有题目必须在答题卡上作答,在试题卷上答题无效。 5. 考试结束后,将试题卷和答题卡一并交回。 一、单项埴空(共 15小题;每小题 1分,满分 15分) 请从 A、B、c、D四个选项中,选出可以填人空白处的最佳选项,井在答属卡上将该选项的 标号涂黑。 例: A computer can only do __________ you have instructed it to do. A. how B. after C. what D. when 答案是 C。 1. —Is Peter coming? —No, he __________ his mind after a phone call at the last minute. A. changes B. changed C. was changing D. had changed 2. The meeting will be held in September, but __________ knows the date for sure. A. everybody B. nobody C. anybody D. somebody 3. I just heard __________ bank where Dora works was robbed by __________ gunman wearing a mask. A. the; / B. a; / C. the; a D. a; the 4. If you miss this chance, it may be years __________ you get another one. A. As B. before C. since D. after 5. —I can drive you home. —__________ , but are you sure it’s not too much trouble? A. That would be great B. Don’t bother C. I’m afraid not D. Take care 6. ___________ in the poorest area of Glasgow, he had a long, hard road to becoming a football star. A. Being raised B. Raising C. Raised D. To raise 7. Without his wartime experiences, Hemingway __________ his famous novel A Farewell to Arms. A. Didn’t write B. hadn’t written C. wouldn’t write D. wouldn’t have written 8. We must find out __________ Karl is coming, so we can book a room for him. A. when B. how C. where D. why 9. Bach died in 1750, but it was not until the early 19th century __________ his musical gift was fully recognized. A. while B. though C. that D. after 10. Last year was the warmest year on record, with global temperature 0.68 ℃ __________ the average. A. below B. on C. at D. above 11. Like ancient sailors, birds can find their way ___________ the sun and the stars. A. used B. having used C. using D. use 12. You __________ be Carol. You haven’t changed a bit after all these years. A. must B. can C. will D. shall 13. In my hometown, there is always a harvest supper for the farmers after all the wheat __________ cut. A. will have been B. will be C. was D. has been 14. He wrote many children’ s books, nearly half of __________ were published in the 1990s. A. whom B. which C. them D. That 15. —Hello Jenny, can I see Ms. Lewis? —__________. I’ll tell you’re here. A. With pleasure B. Never mind C. You’re welcome D. Just a minute 二、完形填空(共 20小题;每小题 1.5分,满分 30分) 请阅读下面两篇短文,掌握大意,然后从 16—35各题所给的四个选项(A、B、C和 D)中, 选出最佳选项,并在答题卡上将该选项的标号涂黑。 A When Alice was sixteen, I was the one who wanted to run away from home. It was ___16___ to see the changes coming over her. She skipped school, and refused to communicate. I tried being firm, but it didn‘t 17 . I saw a dark future for my once sweet daughter. One school day Alice returned home very late. With a quarrel in view. I was surprised to see Alice was 18 . “I hope I did the right thing, Mom,”“Alice said. I saw a cat, all bloody but alive. I ___19___ it to the vet’s(宠物医院), and was asked to make payment 20 . As I couldn’t reach anyone at the phone number on the cat’s tag(标牌), I had to pay the bill.” In the following days, the owner still couldn’t be 21 . Alice paid the vet to continue treatment. I grew 22 : what if the family had simply left the cat behind? Aweek went by. A woman called to speak to Alice. “She is at school,” I said. “You have a 23 daughter,” she said, apparently in tears. Her family had just returned from abroad, and got a (n) 24 from the vet. Their cat was recovering, thanks to Alice’s 25 . “We can’t wait to hug Cuddles again,” she sobbed. Upon her return home, Alice was filled with 26 at the news. So was I. I learned through another woman’s eyes that my daughter was still a good person despite her ___27___ teenage years. Her warm heart would surely guide her in the right direction. 16. A. pleasant B. painful C. unwise D. inspiring 17. A. remain B. match C. appear D. work 18. A. annoyed B. amused C. worried D. interested 19. A. carried B. followed C. returned D. guided 20. A. monthly B. honestly C. generously D. immediately 21.A. trusted B. contacted C. persuaded D. satisfied 22. A. active B. rude C. anxious D. proud 23. A. pretty B. grateful C. wonderful D. curious 24. A. apology B. invitation C. message D. reply 25. A. suggestion B. donation C. encouragement D. help 26. A. love B. anger C. regret D. joy 27. A. troubled B. long C. boring D. quiet B Imagine the first days in a new time zone. Slow to respond to the 28 , your body clock is confused. You’re sleepy all day, but when it’s time for bed, you can hardly fall asleep. Obviously you are 29 jet lag(时差反应). Travelers have traditionally fought this 30 with sleeping pills or alcohol. There are actually healthier ways that can work just as 31. For example, the moment you get on the airplane, start 32 your biological block to the destination’s time. If it’s daytime in your destination, try to stay 33 . Walking around the cabin(客舱) can be of help. When it’s nighttime, try to sleep. In that case, eat before the flight, ___34___ an empty stomach will prevent you from sleeping. These tips will help you start a new ____35____ of sleep and wakefulness. 28. A. flight B. change C. demand D. climate 29. A. suffering from B. working on C. looking into D. leading to 30. A. danger B. problem C. waste D. fear 31. A. briefly B.slowly C. suddenly D. effectively 32. A. checking B. sending C. adjusting D. stopping 33. A. awake B. alone C. hungry D.calm 34. A. though B. so C. whole D. or 35. A. understanding B. cycle C. research D. trend 三、阅读理解(共 20小题;每小题 2分,满分 40分) 请阅读下列短文,从每题所给的四个选项(A、B、c和 D)中,选出最佳选项,并在答题卡上 将该选项的标号涂黑。 A At thirteen, I was diagnosed(诊所) with kind of attention disorder. It made school difficult for me. When everyone else in the class was focusing on tasks, I could not. In my first literature class, Mrs.Smith asked us to read a story and then write on it, all within 45 minutes. I raised my hand right away and said,“Mrs.Smith, you see, the doctor said I have attention problems. I might not be able to do it.” She glanced down at me through her glasses, “you are no different from your classmates, young man.” I tried, but I didn’t finish the reading when the bell rang. I had to take it home. In the quietness of my bedroom, the story suddenly all became clear to me. It was about a blind person, Louis Braille. He lived in a time when the blind couldn’t get much education. But Louis didn’t give up. Instead, h invented a reading system of raised dots(点), which opened up a whole new world of knowledge to the blind. Wasn’t I the “blind” in my class, being made to learn like the “sighted” students? My thoughts spilled out and my pen started to dance. I completed the task within 40 minutes. Indeed, I was no different from others; I just needed a quieter place. If Louis could find his way out of his problems, why should I ever give up? I didn’t expect anything when I handled in my paper to Mrs.Smith, so it was quite a surprise when it came back to me the next day- with an“A” on it. At the bottom of the paper were these words:“ See what you can do when you keep trying?” 36. The author didn’t finish the reading in class because __________. A. He was new to the class B. He was tried of literature C. He had an attention disorder D. He wanted to take the task home 37. What do we know about Louis Braille from the passage? A. He had good sight B. He made a great invention. C. He gave up reading D. He learned a lot from school 38.What was Mrs.Smith ‘s attitude to the author at the end of the story? A. Angry B. Impatient C. Sympathetic D. Encouraging 38. What is the main idea of the passage? A. The disabled should be treated with respect. B. A teacher can open up a new world to students. C. One can find his way out of difficulties with efforts. D. Everyone needs a hand when faced with challenges. B In ancient Egypt, a shopkeeper discovered that he could attract customers to his shop simply by making changes to its environment. Modern businesses have been following his lead,with more tactics (策略). One tactic involves where to display the goods. Foe example, stores place fruits and vegetables in the first section. They know that customers who buy the healthy food first will feel happy so that they will buy more junk food(垃圾食品)later in their trip. In department stores, section is generally next to the women’s cosmetics(化妆品) section:while the shop assistant is going back to find the right size shoe, bored customers are likely to wander over cosmetics they might want to try later. Besides, businesses seek to appeal to customers’ senses. Stores notice that the smell of baked goods encourages shopping, they make their own bread each morning and then fan the bread smell into the store throughout the day. Music sells goods, too. Researchers in Britain found that when French music was played, sales of French wine went up. When it comes to the selling of houses, businesses also use highly rewarding tactics. They find that customers make decision in the first few second upon walking in the door, and turn it into a business opportunity. A California builder designed the structure of its houses smartly. When entering the house, the customer would see the Pacific Ocean through the windows, and then the poll through an open stairway leading to the lower level. The instant view of water on both levels helped sell these $10 million houses. 40. Why do stores usually display fruits and vegetables in the first section? A. To save customers times. B. To show they are high quality foods. C. To help sell junk food. D. To sell them at discount prices. 41. According to Paragraph 3, which of the following encourages customers to buy? A. Opening the store early in the morning. B. Displaying British wines next to French ones. C. Inviting customers to play music. D. Filling the store with the smell of fresh bread. 42. What is the California builder’s story intended to prove? A. The house structure is a key factor customers consider. B. The more costly the house is, the better it sells. C. An ocean view is much to the customers’ taste. D. A good first impression increases sales. 43. What is the main purpose of the passage? A. To explain how businesses turn people into their customers. B. To introduces how businesses have grown from the past. C. To report researches on customer behavior. D. To show dishonest business practices. C LakeLander ·2 hours ago Today, a man talked very loud on his phone on a train between Malvern and Reading, making many passengers upset. I wonder how he would react if I were to read my newspaper out loud on the train, I have never had the courage to do it, though. Pak50 ·57 minutes ago Why not give it a try? Perhaps you should take lessons on a musical instrument. The late musician Dennis Brian is said to have asked a fellow train passenger to turn off his radio. When his request was refused, he took out his French horn(号) and started to practice. Angie O’Edema ·42 minutes ago I don’t see how musical instruments can help improve manners in public. Don’t do to others what you wouldn’t like to be done to yourself. Once, a passenger next to me talked out loud on his mobile phone. I left my seat quietly, giving him some privacy to finish his conversation. He realized this and apologised to me. When his phone rang again later, he left his seat to answer it. You see, a bit of respect and cooperation can do the job better. Taodas ·29 minutes ago Join the discussion… I did read my newspaper out loud on a train, and it turned out well. The guy took it in good part, and we chatted happily all the way to Edinbergh. Sophie 76 ·13minutes ago I have not tried reading my newspaper out loud on a train, but ,several years ago, I read some chapters from Harry Porter to my bored and noisy children. Several passengers seemed to appreciate what I did. 44. The passenger made an apology to Angie O’Edema because____. A. he offered his seat to someone else B. he spoke very loudly on his phone C. he refused to talk with Angie D. he ignored Angie’s request 45. Who once read a newspaper out loud on a train? A. Pak50 B. Angie O’Edema C. Taodas D. Sophie76 46. What is the discussion mainly about? A. How to react to bad behavior. B. How to kill time on a train. C. How to chat with strangers. D. How to make a phone call. 47. Where is the passage most probably taken from? A. Awebpage. B. A newspaper. C. A novel. D. A report. D There are many places to go on safari(观赏野生动物)in Africa, but riding a horse through the flooded waters of Botswana's Okavango Delta must rank as one of the world's most exciting wildlife journeys. Several safari camps operate as the base for this adventure, providing unique rides twice a day to explore deep into the delta. The camps have excellent horses, professional guides and lots of support workers. They have a reputation for providing a great riding experience. The morning ride, when the guides take you to beautiful, shallow lakes full of water lilies, tends to be more active. It is unlike any other riding experience. With rainbows forming in the splashing water around you and the sound of huge drops of water bouncing off your body and face. It is truly exciting. You are very likely to come across large wild animals, too. On horseback it is possible to get quite close to elephants, giraffes and many other animals. The sense of excitement and tension levels rise suddenly though, as does your heart rate, as you move closer to them. In the evening, rides are usually at a more relaxed and unhurried pace. With golden light streaming across the grassy delta and the animals coming out to eat and drink. Sedate though they are, rides at this time of day are still very impressive. As the sun's rays pass through the dust kicked up by the horses, the romance of Africa comes to life. Back at the camp you can kick off your boots and enjoy excellent food and wine. Looking back on your day, you will find it hard to deny that a horseback Safari is as close as you will ever come to answering the call of the wild. 48. What does the underlined word "They" refer to? A. Flooded waters B. Wildlife journey C. Safari camps D. Unique rides 49. What does the author find most exciting about a horse safari? A. Seeing and feeling the real African life. B. Enjoying good food and wine at the camp. C. Hunting large animals just as our ancestors did. D. Being part of the scene and getting close to animals. 50. What does the underlined word "sedate" probably mean? A. Wild and romantic B. Slow and peaceful C. Hurry and thirsty D. Active and excited 51. The author introduced the riding experience in the OKavango Delta mainly by________. A. Following space order B. Following time order C. Making classifications D. Giving examples E The values of artistic works, according to cultural relativism(相对主义 ), are simply reflections of local social and economic conditions. Such a view, however, fails to explain the ability of some works of art to excite the human mind across cultures and through centuries. History has witnessed the endless productions of Shakespearean plays in every major language of the world. It is never rare to find that Mozart packs Japanese concert halls, as Japanese painter Hiroshige does Paris galleries, Unique works of this kind are different from today’s popular art, even if they began as works of popular art. They have set themselves apart in their timeless appeal and will probably be enjoyed for centuries into the future. In a 1757 essay, the philosopher David Hume argued that because“the general principles of taste are uniform(不变的) in human nature,”the value of some works of art might be essentially permanent. He observed that Homer was still admired after two thousand years. Works of this type, he believed, spoke to deep and unvarying features of human nature and could continue to exist over centuries. Now researchers are applying scientific methods to the study of the universality of art. For example, evolutionary psychology is being used by literary scholars to explain the long-lasting themes and plot devices in fiction. The structures of musical pieces are now open to experimental analysis as never before. Research findings seem to indicate that the creation by a great artist is as permanent an achievement as the discovery by a great scientist. 52. According to the passage, what do we know about cultural relativism? A. It introduces different cultural values. B. It explains the history of artistic works. C. It relates artistic values to local conditions. D. It excites the human mind throughout the world. 53. In Paragraph 2, the artists are mentioned in order to show that __________. . A. great works of art can go beyond national boundaries B. history gives art works special appeal to set them apart C. popular arts are hardly distinguishable from great arts D. great artists are skilled at combining various cultures 54. According to Hume, some works of art can exist for centuries because __________. A. they are results of scientific study B. they establish some general principles of art C. they are created by the world’s greatest artists D. they appeal to unchanging features of human nature 55. Which of the following can best serve as the title of the passage? A. Are Artistic Values Universal? B. Are Popular Arts Permanent? C. Is Human Nature Uniform? D. Is Cultural Relativism Scientific? 四、写作(共两个写作任务,满分 35 分) 写作一(满分 15分) 请就以下话题,按要求用英文写作。 If you could give your younger self one piece of advice, what would you say? 要求: (1)作出回答并说出理由; (2)词数不少于 60; (3)在答题卡上做答。 写作二(满分 20分) Bookcrossing. com 是一个面向全球的图书分享网站。该网站组织的分享活动有两种参与 方式:一是自由分享(wild release),即把书放在指定地点,由其他参与者自由获取;二是 定向分享(controlled release),即直接传递给另一位参与者。假设你是李华,请用英文写信 申请参加。内容应包括: · 表明写信目的 · 选择一种分享方式 · 简述作出该选择的理由 · 希望了解更多信息 注意:(1)词数不少于 80; (2)在答题卡上做答 (3)书信格式及开头已给出(不计入总词数) 2015年普通高等学校招生全国统一考试 英 语(陕西卷) 本试卷分第Ⅰ卷(选择题)和第Ⅱ卷(非选择题)两部分。考试结束后,将本试卷和答题卡一 并交回。 第Ⅰ卷 注意事项: 1. 答第Ⅰ卷前,考生务必将自己的姓名、准考证号填写在答题卡上。 2. 选出每小题答案后,用 2B铅笔把答题卡上对应题目的答案标号涂黑。如需改动, 用橡皮擦干净后,再选涂其他答案标号,不能答在本试卷上,否则无效。 第一部分:英语知识运用(共四节,满分 55分) 第一节 语音知识(共 5小题;每小题 1分,满分 5分) 从每小题的 A、B、C、D四个选项中,找出其划线部分与所给单词的划线部分读音相同的选项, 并在答题卡上将该选项涂黑。 1. stomach A. technology B. exchange C. chair D. church 2. uncle A. concert B. strange C. blanket D. autumn 3. exact A. excellent B. example C. exercise D. expensive 4. table A. cottage B. basket C. wake D. family 5. double A. soul B. group C. pronounce D. couple 第二节 情景对话(共 5小题;每小题 1分,满分 5分) 根据对话情境和内容,从对话后所给的选项中选出能填入每一空白处的最佳选项,并在答题卡上 将该选项涂黑。选项中有两个为多余选项。 Doctor: Good morning, Sir. 6 Johnson: Yes, doctor. I'm always tired but when I go to bed I can't sleep. Doctor: 7 Johnson: Since I started my new job two months ago. Doctor:What is your job ? Johnson: I’m in advertising. 8 Doctor: It depends. How many hours do you work? Johnson: About 80 hours a week Doctor: 9 . Do you often take exercise? Johnson: Not very often. 10 . Doctor:Well, you do need to find some time. Try to work less, or look for a more stress-free job. A. That's a lot! B. I like my job. C. Does it matter? D. How can I help you? E. I don't have the time. F. What time do you usually go to bed? G.. How long have you had this problem? 第三节 语法和词汇知识(共 15小题;每小题 1分,满分 15分) 从每小题的 A、B、C、D四个选项中,选出可以填入空白处的最佳选项,并在答题卡上将该 选项涂黑。 11. ---Let’s go to the New Year’s Eve party, shall we? ---__________ I guess it will be fun. A. Forget it! B. No way! C. Why not? D. What for? 12. The little pupil took his grandma __________ the arm and walked her across the street. A. on B. by C. in D. at 13. To warm himself, the sailor sat in front of the fire rubbing one bare foot against __________. A. another B. the other C. other D. either 14. __________ more learned a man is, __________more modest be usually become. A. The ; the B. A; a C. The ; a D. A; the 15. As the smallest child of his family, Alex is always longing for the time __________ he should be able to be independent. A. which B. where C. whom D. when 16. —Sir, could I hand in my homework a bit late? — __________, since you’ve been unwell these days. A. You can’t be serious B. I’m afraid not C. Good idea D. Well, all right 17. After receiving the Oscar for Best Supporting Actress, Anne Benedict went on __________ all the people who had helped in her career. A. to thank B. thanking C. having thanked D. to have thanked 18. Back from his two-year medical service in Africa, Dr. Lee was very happy to see his mother __________ good care of at home. A. taking B. taken C. take D. be taken 19. Reading her biography, I was lost in admiration for __________ Doris Lessing had achieved in literature. A. what B. that C. why D. how 20. I believe you will have a wonderful time here __________ you get to know everyone else. A. though B. as if C. once D. so that 21. You __________ feel all the training a waste of time, but I’m a hundred percent sure later you’ll be grateful you did it. A. should B. need C. shall D. may 22. Mary__________ really hard on his book and thinks he’ll have finished it by Friday. A. worked B. has been working C. had worked D. has worked 23. Ellen is a fantastic dancer. I wish I __________ as well as her. A. dance B. will dance C. had danced D. danced 24. At college, Barack Obama didn’t know that he __________ the first black president of the United States of America. A. was to become B. becomes C. is to become D. became 25. Peter will __________ his post as the head of the travel agency at the end of next month. A. take up B. put up C. add up D. break up 第四节 完形填空(共 20小题;每小题 1.5分,满分 30分) 阅读下面短文, 从短文后各题的 A、B、C、D四个选项中,选出适合填入对应空白处的 最佳选项,并在答题卡上将该选项涂黑。 Dad had a green comb. He bought it when he married Mum. Every night, he would hand me his ___26___ and say, “Good girl, help Daddy clean it, OK?” I was 27 to do it. At age five , this dull task brought me such 28 . I would excitedly turn the tap 29 and brush the comb carefully. Satisfied that I’d done a good job, I would happily return the comb to Dad. He would 30 affectionately at me and place the comb on his wallet. Two years later, Dad started his own 31 , which wasn’t doing so well. That was when things started to 32 . Dad didn’t come home as early and as much as he used to. Mum and I became 33 with him for placing our family in trouble. With 34 , an uncomfortable silence grew between us. After my graduation, Dad’s business was getting back on track. On my 28th birthday, Dad came home 35 . As usually I helped him carry his bags into his study. When I turned to leave, he said ,“Hey, would you help me 36 my comb?” I looked at him a while, then 37 the comb and headed to the sink. It hit me then: why, as a child, 38 Dad clean his comb was such a pleasure. That routine(习惯) meant Dad was home early to 39 the evening with Mum and me. It 40 a happy and loving family. I passed the clean comb back to Dad. He smiled at me and 41 placed his comb on his wallet. But this time , I noticed something 42 . Dad had aged. He had wrinkles next to his eyes when he smiled, 43 his smile was still as 44 as before, the smile of a father who just wanted a good 45 for his family. 26. A. bag B. wallet C. comb D. brush 27. A. annoyed B. relieved C. ashamed D. pleased 28. A. joy B. sadness C. courage D. pain 29. A. out B. over C. in D. on 30. A. stare B. smile C. shout D. laugh 31. A. family B. business C. task D. journey 32. A. progress B. change C. improve D. form 33. A.satisfied B. delighted C. mad D. strict 34. A. time B. patience C. speed D. ease 35. A. occasionally B. early C. frequently D. rarely 36. A. sharply B. repair C. clean D. keep 37. A. dropped B. took C. handed D. threw 38. A. watching B. letting C. helping D. hearing 39. A. find B. lose C. waste D. spend 40. A. affected B. broke C. meant D. supported 41. A. firmly B. hurriedly C. casually D. carefully 42. A. different B. exciting C. interesting D. urgent 43. A. for B. or C. so D. yet 44. A. convincing B. heartwarming C. cautious D. innocent 45. A. origin B. life C. reputation D. education 第二部分:阅读理解(共两节,满分 40分) 第一节:(共 15小题;每小题 2分,满分 30) 阅读下列四篇短文,从每小题后所给的 A、B、C、D四个选项中,选出最佳选项,并在答 题卡上将该选项涂黑。 A Share with us! Would you like to have your writing published in this magazine? Then let us know! Anecdotes and Jokes $50 What’s made you laugh recently? A funny sign? A colleague’s be haviour? Got a joke? Send it in for Laughter is the Best Medicine! Email: Juliet@sws.com Smart Animals Up to $100 Send us a tale about the strange behaviour of unique pets or wildlife in up 300 words. Email: audry@sws.com Power of Love Up to $150 Acts of generosity can change lives or just give you that warm feeling full of love. Share your moments 100—500 words. Email: susan@sws.com My Story $350 Do you have an inspiring or life-changing story to tell? Your story must be true, unpublished, original and 800-1000words. Email: nanjc@sws.com For more information, please visit: www.sws.com./share 46. How much will the magazine pay for a joke to be published? A. $50 B. $100 C. $150 D.$350 47. If you want to share a story of your pets with the readers, you need to submit it to __________. A. Anecdotes and Jokes B. Smart Animals C. Power of Love D. My Story 48. A story showing people’s generosity should be emailed to the editor at __________. A. juliet@sws.com B. susan@sws.com C. Audrey@sws.com D. nanjc@sws.com 49. An inspiring story sent to the magazine should __________. A. describe strange behaviour B. contain less than 800 words C. be real and original D. be published before B When the dog named Judy spotted the first sheep in her life, she did what comes naturally. The four-year-old dog set off racing after the sheep across several fields and, being a city animal, lost both her sheep and her sense of direction. Then she ran along the edge of cliff( 悬崖 ) and fell 100 feet, bouncing off a rock into the sea. Her owner Mike Holden panicked and celled the coastguard of Cornwall, who turned up in seconds . Six volunteers slid down the cliff with the help of a rope but gave up all hope of finding her alive after a 90-minute search. Three days later, a hurricane hit the coast near Cornwall. Mr. Holden returned home from his holiday upset and convinced his pet was dead. He comforted himself with the thought she had died in the most beautiful part of the country. For the next two weeks, the Holdens were heartbroken . Then, one day, the phone rang and Steve Tregear, the coastguard of Cornwall, asked Holder if he would like his dog bark. A birdwatcher, armed with a telescope, found the pet sitting desperately on a rock. While he sounded the alarm, a student from Leeds climbed down the cliff to collect Judy. The dog had initially been knocked unconscious(失去知觉的 )but had survived by drinking water from a fresh scream at the base of the cliff. She may have fed on the body of a sheep which had also fallen over the edge. “The dog was very thin and hungry,” Steve Tregear said , “It was a very dog. She survived because of a plentiful supply of fresh water,” he added. It was ,as Mr. Holden admitted, “a minor miracle(奇迹)”. 50. The dog Jody fell down the cliff when she was __________. A. rescuing her owner B. caught in a hurricane C. blocked by a rock D. running after a sheep 51. Who spotted Judy after the accident? A. A birdwatcher B. A student from Leeds C. Six volunteers D. The coastguard of Cornwall 52. What can we infer from the text? A. People like to travel with their pets. B. Judy was taken to the fields for hunting. C. Luck plays a vital role in Judy's survival. D. Holden cared little where Judy was buried. 53. Which of the following can be the best title for the text? A. Miracle of the Coastguard. B. Surviving a Hurricane. C. Dangers in the Wild D. Coming Back from the Dead. C The production of coffee beans is a huge, profitable business, but, unfortunately, full-sun production is taking over the industry and bringing about a lot of damage. The change in how coffee is grown from shade-grown production to full-sun production endangers the very existence of, certain animals and birds, and even disturbs the world’s ecological balance. On a local level, the damage of the forest required by full-sun fields affects the area’s birds and animals. The shade of the forest trees provides a home for birds and other special(物种) that depend on the trees’ flowers and fruits. Full-sun coffee growers destroy this forest home. As a result, many special are quickly dying out. On a more global level, the destruction of the rainforest for full-sun coffee fields also threatens(威胁)human life. Medical research often makes use of the forests' plant and animal life, and the destruction of such species could prevent researchers from finding cures for certain diseases. In addition, new coffee-growing techniques are poisoning the water locally, and eventually the world's groundwater. Both locally and globally, the continued spread of full-sun coffee plantations (种植园 )could mean the destruction of the rainforest ecology. The loss of shade trees is already causing a slight change in the world's climate, and studies show that loss of oxygen-giving trees also leads to air pollution and global warming. Moreover, the new growing techniques are contributing to acidic( 酸 性 的 ) soil conditions. It is obvious that the way much coffee is grown affects many aspects many aspects of life, from the local environment to the global ecology. But consumers do have a choice. They can purchase shade-grown coffee whenever possible, although at a higher cost. The future health of the planet and mankind is surely worth more than an inexpensive cup of coffee. 54. What can we learn about full-sun coffee production from Paragraph 4? A. It limits the spread of new growing techniques. B. It leads to air pollution and global warming. C. It slows down the loss of shade trees. D. It improves local soil conditions. 55. The purpose of the text is to . A. entertain B. advertise C. instruct D. persuade 56. Where does this text probably come from ? A. An agricultural magazine. B. A medical journal. C. An engineering textbook. D. A tourist guide. 57. Which of the following shows the structure of the whole text A. B. C. D. (P : Paragraph) D Parents who help their children with homework may actually be bringing down their school grades. Other forms of prenatal involvement, including volunteering at school and observing a child's class, also fail to help, according to the most recent study on the topic. The findings challenge a key principle of modern parenting(养育子女 ) where schools except them to act as partners in their children's education. Previous generations concentrated on getting children to school on time, fed, dressed and ready to learn. Kaith Robinson, the author of the study, said, "I really don't know if the public is ready for this but there are some ways parents can be involved in their kids' education that leads to declines in their academic performance. One of the things that was consistently negative was parents' help with homework." Robinson suggested that may be because parents themselves struggle to understand the task." They may either not remember the material their kids are studying now, or in some cases never learnt it themselves, but they're still offering advice." P1 P2 P3 P5 P4 P1 P2 P3 P4 P5 P2 P3 P4 P1 P5 P1 P2 P5 P4 P3 Robinson assessed parental involvement performance and found one of the most damaging things a parent could do was to punish their children for poor marks. In general, about 20% of parental involvement was positive, about 45% negative and the rest statistically insignificant. Common sense suggests it was a good thing for parents to get involved because "children with good academic success do have involved parents ", admitted Robinson. But he argued that this did not prove parental involvement was the root cause of that success." A big surprise was that Asian-American parents whose kids are doing so well in school hardly involved. They took a more reasonable approach, conveying to their children how success at school could improve their lives." 58. The underlined expression "parental involvement " in Paragraph 1 probably means __________. A. parents' expectation on children's health B. parents' participation in children's education C. parents' control over children's life D. parents' plan for children's future 59. What is the major finding of Robinson's study ? A. Modern parents raise children in a more scientific way. B. Punishing kids for bad marks is mentally damaging. C. Parental involvement is not so beneficial as expected. D. Parents are not able to help with children’s homework. 60. The example of Asian-American parents implies that parents should __________. A. help children realize the importance of schooling B. set a specific life goal for their children C. spend more time improving their own lives D. take a more active part in school management 第二节 (共 5小题;每小题 2分,满分 10分) 根据短文内容,从下框的 A~F选项中选出能概括每一段主题的最佳选项,并在答题卡上将 该选项涂黑。选项中有一项为多余项。 A. The mistaken belief B. The need for tolerance C. Unpunctuality at dinners D. Punctuality and confidence E. Self-discipline and punctuality F. Avoid anxiety by being punctual 61. __________ There is the belief that, if you arrive at an appointment late, you will be considered important. This is a mistaken view. Being unpunctual, we are not respectful of others ; we are interfering (扰 乱)with another man's time. We must realise that keeping appointments or being punctual is a contract that is silently agreed and we are expected to respect this contract. It is only natural that we lose faith, trust and confidence in a person who is tardy(延迟的). 62. __________ To be punctual one has to have self-discipline(自律), and the lack of it affects others. A school boy or girl is unpunctual because he or she does not have the necessary human virtue of self-discipline. It is also a mark of disrespect for a system or an institution. Unpunctual people seldom realise that their habit cause problems to others. A salesman who is not punctual may not make a sale if he arrives late for an appointment. If one is late for a job interview, it is not likely that he will get the job. 63. __________ Being punctual, we can avoid anxiety. Imagine the anxiety if you do not want to be considered unpunctual. You will be anxious if you set out for a dinner late. The person who sets out late might be careless in driving. He will ignore traffic rules. A traffic jam, flat tyres, etc. can delay him further. Happy and calm is the man who takes all these possibilities into consideration and arrives at the appointment either early or on time. 64. __________ Many of those who attend dinners are notorious (声名狼藉的)for unpunctuality. They ignore the appointed time and leave their homes only after the fixed time. They are indifferent to the inconvenience caused to others. If the self-centred guest arrives late, the nine others at the table set for ten will have to wait. The host is put in an unpleasant situation and this man seldom thinks of the inconvenience caused to all -the waiters, the management staff, etc. It is necessary for us to think of others and be considerate to them. 65. __________ However, we cannot always be intolerant (无法容忍的 ) of tardiness, for ordinary living requires some tolerance. There can be a busy executive who fights to keep to his schedule. Such a person may be forgiven if he is late, but not those who are deliberately late to create impression. In modern society, punctuality is a necessary virtue. It is a recognition of the importance of other people. 第 II卷 第三部分 写作(共三节,满分 55分) 第一节 单词拼写(共 10题;每题 1分,满分 10分) 根据下列各句句意和空白之后的汉语提示词,在答题卡指定区域的横线上写出对应单词的正 确、完整形式,每空只写一词。 66. Cecily, I shall _______________ (想念) our pleasant talks when you leave. 67. Hans Zhang was _______________ (自豪的) of himself for not giving up. 68. The teacher asked one student to _______________ (重复 ) what he had heard from the tape recorder. 69. Young people should always _______________ (对待) the elders with respect. 70. Tourism is the major source of _______________(收入) for this area. 71. Our class president went up to Cambridge at the age of 15 to study _______________ (化学) . 72. For your own _______________ (安全), please keep away from the tiger cage. 73. It was pouring with rain so I _______________ (接受) his offer of a lift. 74. In a week, Lucy has already been interviewed by six _______________ (医院). 75. After knocking _______________ (礼貌地 ) at the door, the applicant entered the office of the general manager. 第二节 短文改错(共 10小题;每小题 1.5分,满分 15分) 假定英语课上老师要求同桌之间交换修改作文,请你修改你同桌系的以下作文。文中共有 10处语 言错误,每句中最多有两处。错误涉及一个单词的增加、删除或修改。 增加:在缺词处加一个漏词符号(∧),并在其下面写出该加的词。 删除:把多余的词用斜线( )划掉。 修改:在错的词下划一横线( ),并在该词下面写出修改后的词。 注意:1.每处错误及修改均仅限一词; 2.只允许修改 10处,多者(从第 11处起)不计分。 My soccer coach retired in last week. I wanted to do anything special for him at his retirement party. My mum makes the better biscuits in the world, so I decide to ask her for help. Mum taught me some basic step of baking. I insisted on doing most of the baking myself. I thought the biscuits were really well. My only mistake was that I dropped some on the floor after I was packing them up. At a party, my coach, with a biscuit in his mouth, asked surprisingly who made them and joked, “I might have to retire again next year just get some more of these biscuits.” My favorite picture at the party is of my coach and me enjoy the biscuits with happy laughter! 第三节 书面表达(满分 30分) 假定你是李华。在校报英语专栏看到了学校“英语文化节”的一则招募启事,请阅读启事,并 根据写作要点和写作要求写一封应征邮件。 Volunteers Wanted Our annual English Festival, which will be held on June 15-17, 2015, is now looking for 20 student volunteers to provide service for Talent Show, Speech Contest, and English Debate. If you are interested, please send an application email at your earliest convenience to Ms.Chen at chenlaoshi@aef.com. 写作要点: 1.表示写信意图; 2.陈述应征目的; 3.说明应征条件(性格、能力等)。 写作要求: 1.邮件词数不少于 100; 2.开头和结尾部分已写好,不计入总词数; 3.可根据情况增加细节,使行文连贯 4.不能使用真实姓名和学校名称。 Dear MS Chen , I’m Li Hua, a student from Class2, Grade 3.______________________________________________ ____________________________________________________________________________________ ____________________________________________________________________________________ ____________________________________________________________________________________ ___________________________________________________________________________________ Looking forward to your reply. Yours truly, Li Hua 参考答案 2015年普通高等学校招生全国统一考试(新课标 I)参考答案 第 I卷 1~5 ACABC 6~10 BABCB 11~15 ABCAB 16~20 CBAAC 21~25 ADCBD 26~30 BABAD 31~35 CDCAB 36~40 BGACE 41~45 CCBCD 46~50 ADBAD 51~55 AABBC 56~60 DCDAB 第 II卷 61. arrived 62. before / earlier 63. its 64. that / which 65. paintings 66. by 67. is 68. conducted 69. regularly 70. living 短文改错 When I was a child, I hoped to live in the city. I think I would be happy there. Now I am living in thought a city, but I miss my home in ∧countryside. There the air is clean or the mountains are green. the and Unfortunately, on the development of industrialization, the environment has been polluted. Lots of with studies have been shown that global warming has already become a very seriously problem. The airs serious air we breathe in is getting dirtier and dirtier. Much rare animals are dying out. We must found ways to Many find find protect your environment. If we fail to do so, we‟ll live to regret it. our / the 写作 Dear Peter, I‟d like to ask you to write an article for our school‟s English newspaper. The“Foreign Cultures” section in our newspaper is very popular among us students. It carries articles written by foreign friends about the cultures of their home countries. Would you please write something about the culture in your part of the United States? And we would especially welcome articles about how Americans spend their holidays and festivals, and the life of American high school students. You can write anything relevant so long as it‟s interesting and informative, 400 words would be fine. Could we have your article before June 28? I‟m looking forward to hearing from you! Yours, Li Hua 2015年普通高等学校招生全国统一考试(新课标 II)参考答案 第一部分: 1~5 ACABC 6-10 BABCB 11-15ABCAB 16-20 CBAAC 第二部分 21-25 CADDB 25-30 BCACA 31-35 BCBDA 36-40 CEADG 第三部分 41-45 BCDCB 46-50 ADDBC 51-55 ABDAC 56-60 ABDAC 61. built 62. the 63. ability 64. using 65. slowly 66. to cool 67. at 68. goes 69. natural 70. how 第四部分 第一节 1.parent—parents 2.on---in 3.删去 very 4.looks---looking 5.where—that 或删去 6.begun—began 7.telling—told 8.a---the 9.saw 和 parents 之 间 插 入 his 10.terrible---terribly 第二节 Dear Lucy, I’d like to invite you to join us for a visit to the nearby home next Saturday for the Double Ninth Festival. It is the day for the elderly in our culture. We’ll go and make dumplings and cakes with the elderly people there. We’ll also spend some fun time together singing, dancing and playing games, which we hope will make them happy. We should be back around 4 o’clock in the afternoon. If you are able to come with us, please let us know and we’ll wait for you at the school gate at 9 in the morning. Looking forward to your reply. Yours, Li Hua 2015年普通高等学校招生全国统一考试(北京卷)参考答案 1-5 BAABC 6-10 CACBA 11-15 BACCB 16. Tarshis 17. 627935 18. May 19. black 20. open 21-25 CACDB 26-30 CACDB 31-35 DCADB 36-40 ACBAD 41-45 ADABC 46-50 BCBDD 51-55 ADCBC 56-60 ADBAB 61-65 CABDC 66-70 DCCAB 71-75 BCAEG 书面表达 第一节 One possible version: Dear Jim I am writing to tell you my exciting plan for the summer holiday. I am going to join the Dragon Boat Training Camp. I wonder if you would like to go with me. This camp will open on July 20th. If offers us a lot of interesting activities. We can enjoy sports and learn about Chinese culture. If you are interested in Chinese culture and want to join it, I will send you more information to it. Hope to meet you at the training camp this summer. 第二节 Last Tuesday, our class invited an old craftsman to teach us how to make dough figurines. When the craftsman came into the classroom, we gave him a warm welcome and two boys helped him with the tool box. First, he showed us the steps and skills of making dough figurines. We stood around him and watched him making . Then we put the figurines we made on the table and took pictures with the old craftsman. Looking at the figurines, we were all very excited. We hope we can have more activities of this kind 2015年普通高等学校招生全国统一考试(天津卷)参考答案 第 I卷 第一、二部分(Key 1 to 55) 1. B 2. A 3. D 4. A 5. C 6. D 7. C 8. D 9. A 10. B 11. B 12. C 13. D 14. B 15. A 16. B 17. A 18. C 19. A 20. C 21. B 22. D 23. B 24. D 25. A 26. C 27. B 28. D 29. B 30. D 31. D 32. C 33. A 34. B 35. A 36. D 37. A 38. A 39. C 40. D 41. B 42. D 43. C 44. B 45. D 46. C 47. B 48. B 49. A 50. A 51. C 52. B 53. D 54. D 55. A 第 II卷 第三部分 第一节 阅读表达 Some possible answers: 56. He was a postman. Or: He worked in a post-office. 57. He advised him to slow down (and take things easy). / He advised he/his grandfather (should) slow down. 58. The process is more important than the result. Or:Aman should not slow down however old he is. Or: Life isn't having it made; it's getting it made. / It's the journey, not the arrival that counts. 59. My/His/The grandfather is still busy doing meaningful things. Or: The grandfather is still living an active life. Or: The grandfather is still involved in whatever he can do. 60. Yes. One should always be full of passion in his life no matter how old he is. Or: People should make full use of their time to do something meaningful No. It is more sensible for people to slow down and enjoy an easy life in their old age. Or: Slowing down contributes to better health and longer life for people in old age. 第二节 书面表达 Dear Chris, I am Li Jin, Chairman of the Student Union of Chenguang High School. I am writing to inform you of our donation. In order to enhance the friendship between our two schools, our school has appealed to the students to donate books to your Chinese class. The books, which can be used as reading materials or reference books in you Chinese class, include 30 story books, 25 magazines and 20 Chinese-English dictionaries. We will take the books over to you when we take part in the summer camp to be held in your school in July. Should you need any other things, please do not hesitate to tell us. Yours, Li Jin 2015年普通高等学校招生全国统一考试(上海卷)参考答案 第Ⅰ卷 第一大题第 1至第 10小题,每题 1分;第 11至第 16小题,每题 2分;第 17至第 24小题,每题 1分。共 30分。 1、A 2、C 3、D 4、C 5、D 6、B 7、B 8、C 9、A 10、B 11、C 12、B 13、A 14、 B 15、D 16、D 17、XW94702 18、electricity19、engineer 20、Wednesday 21、disabled 22、 the human spirit 23、inspiring 24、an online diary/a diary online 第二大题每小题 1分。共 26分。 25、As/Because/Since 26、emptied 27、an 28、looking 29、might/may 30、like 31、Shocked 32、the nicest 33、to check 34、that 35、as if/as though 36、who 37、ignoring 38、 someone/somebody 39、has bean interrupted/is being interrupted/is interrupted 40、which 41、I 42、B 43、H 44、 D 45、C 46、J 47、A 48、E 49、G 50、F 第三大题第 51至第 65小题,每题 1分;第 66至第 77小题,每题 2分;第 78至第 81小题,每 题 2分。共 47分。 51、A 52、C 53、B 54、D 55、C 56、C 57、A 58、B 59、B 60、A 61、A 62、D 63、B 64、D 65、C 66、C 67、B 68、C 69、A 70、D 71、C 72、A 73、A 74、B 75、 A 76、D 77、D 78、Improved physical health and psychosocial development. 79、participate in sports during adulthood. 80、They change rules to suit their needs and their environment. 81、improving performance 第Ⅱ卷 1、 翻译 (第 1-3题,每题 4分;第 4-5 题,每题 5分。共 22分。) 1、 Delicious food is one of the pleasures for people to visit Shanghai. 2、 Street artists have brought brilliant colours to old neighbourhoods with their creativity. 3、 If there is someone is your life to whom you need to say sorry, go ahead and make an apology. 4、 What makes the game unique is that it helps children learn how to cope with problems in real life. 5、 The applications should be carefully prepared in order that the school you like can have an overall and accurate knowledge of your abilities. II 书面表达 Word came that a reading festival would be held by our school and there is a need for a picture for advertisement and I am writing to give my opinion. From my perspective , I am in favor of the picture in the top right corner. As is vividly depicted in that picture, around the table sit three kids with a couple of books in their hands, having a heated discussion about the contents and sharing their thoughts for the themes, which shows their dedication and passion for reading. Basically, several reasons are listed for my preference. First, nothing is more attractive than a proper and realistic picture. This picture serves as an epitome of our daily reading scene in the school and it is apt to let students recall the joy and harvest while reading. Second, this picture stands for the fun of sharing and exchanging ideas. With regard to reading, there is a saying that there are a thousand Hamlets in a thousand people's eyes. After reading, bits and pieces of thoughts will appear in our mind, so it is beneficial to have a better understanding after comparing notes about what we have read. From what is mentioned above, I, undoubtedly, approve of the picture in the top right corner. Thus, I would appreciate it if you could take my proposal into consideration and I wish this festival a complete success. 2015年普通高等学校招生全国统一考试(广东卷)参考答案 1-5 DACDB 6-10 DCBAB 11-15 ABADC 16 a 17 Luckily 18 for 19 was left 20 when 21 fell 22 without 23 to sell 24 where 25 him 26-30 ABCBC 31-35 ACDBC 36-40 BCDAD 41-50 DCDAB 46-50 ECADB 写作 第一节 The DNA examination, a new technology appearing in medical field abroad as good news for some people, is designed to test DNA by means of analyzing the saliva samples. All they have to do is to pay 125 pounds, give testers their saliva samples with a time ranging from 4 to 6 weeks to get an exact results of the exam. What’s more, not only is it of great benefit to predict some serious diseases, but it also can provide people with a prediction of the preference for food, and taking exercises. The medical report may raise their awareness of keeping fit. However, it may give rise to unnecessary anxiety in some people as well. 第二节 Confronted with the misbahavior of her new class, Sally emphasized the idea of “we are family” and afterwards worked out 10 rules to regulate the students’ conduct. Amazingly, she earned respect from her students, all of whom later grew to be confident, considerate and well-behaved. The most obvious problem with our class is lack of interaction or teamwork. Everyone is minding his or her own business without the slightest attention to the people around., thus giving rise to inefficiency in their study and even putting a limit to the play of their potential. To address the issue concerned, I would make it a rule that every Friday a project of a certain subject should be assigned by teachers and completed by every study group consisting of four students. The final presentation on the following Monday will be a result of group work. From where I stand, this rule will definitely exert a profound and far-reaching effect on students’ academic performance. First and foremost, a harmonious but competitive interaction in a group will help everyone’s learning. Additionally, we can combine our skills and knowledge as well as motivate each other greater accomplishments. Last but not least, studying in group can share our experiences in solving problems more efficiently. It is for those reasons that I firmly believe that this rule will promote the learning atmosphere and our teamwork spirit. With continuous efforts, our class, a big family, will compose an extraordinary harmonious symphony. 2015年普通高等学校招生全国统一考试(江苏卷)参考答案 第一部分(共 20 小题;每小题 1 分,共 20 分) 1. A 2. C 3. A 4. B 5. C 6. B 7. A 8. B 9. C 10. B 11. A 12. B 13. C 14. A 15. B 16. C 17. B 18. A 19. A 20. C 第二部分(共 35 小题;每小题 1 分,共 35 分) 21. D 22. D 23. A 24. C 25. C 26. B 27. B 28. D 29. C 30. A 31. A 32. B 33. B 34. D 35. C 36. B 37. A 38. D 39. C 40. B 41. A 42. A 43. D 44. C 45. B 46. D 47. D 48. C 49. A 50. C 51. B 52. D 53. B 54. D 55. C 第三部分(共 15 小题;每小题 2 分,共 30 分) 56. A 57. C 58. B 59. B 60. D 61. D 62. B 63. C 64. A 65. D 66. C 67. A 68. B 69. A 70. D 第四部分(共 10 小题;每小题 1 分,共 10 分) 71. rewards/rewarded 72. Explanations 73. involvement 74. share 75. threat 76. prepare 77. Withdraw 78. profession(s)/intention 79. adapt 80. depends 第五部分(满分 25 分) One possible version: The traffic issue is a hard nut to crack. It not only affects our everyday life, but may also threaten people’s lives. The three selections presented above are typical examples. Quite a few things give rise to the traffic problem. In spite of the large-scale construction of roads and highways, there is still much room for improvement, because of the ever increasing number of cars these years. What’s worse, some drivers, cyclists and pedestrians do not think it vital to obey traffic rules. In fact, traffic rules are part of the rules and regulations closely related to public order. Without them, people could not enjoy harmony or the country would be in chaos. But rules alone don’t secure an orderly society. It is the people who obey the rules that matter. It is everybody’s duty to observe them to keep our society in order and going on the right track. 2015年普通高等学校招生全国统一考试(安徽卷)参考答案 1~5 ACABC 6-10 BABCB 11-15ABCAB 16-20 CBAAC 21-25 ADCBA 26-30 CBDBA 31-35 CCDAB 36-40 DBBCA 41-45 CADAC 46-50 DBADD 51-55 CBADC 56-60 CDBCA 61-65 BCDAC 66-70 DABBD 71-75 ABBAD 76. Deal 77. First 78. Advantages/Benefits 79. New 80. job/position/post 81. Shared 82. asking/rasing 83. Trust 84. Practice/Practise 85. matters/means 书面表达 A Famous Chinese I Would Like to Interview The person I would like to interview is Yang Liwei. I would really like to interview him because he is not only the first Chinese to go to space but also one of the greatest astronauts in the world. I have long been interested in space exploration and I believe I could learn a great deal from him about it. If I could interview him, I would ask him what made him an astronaut and how he was trained. I would also like to know how he felt in space and whether space travel is such great fun as I have read. Finally, I would like to ask a few questions about his personal life, which must be very interesting. 2015年普通高等学校招生全国统一考试(浙江卷)参考答案 01-05 DDBAC 06-10 ADCAC 11-15 BDCDB 16-20 ABDCA 21-25 DRARC 26-30 CABDC 31-35 AADCD 36-40 ABADB 41-44 DACC 45-49 BADCD 50-54 BABDC 55-60 BDABCC 61-65 EFDCA 短文改错: 书面表达: One possible student version: When I Have Different Opinion We may have different opinions in organizing class activities. We may have various ways to deal with such a situation. When I have a better idea, I would choose to stick to it. By doing so, I can not only share good ideas with others but also learn to express myself clearly. Once we were discussing where to go for an outing. Most of my classmates wanted to go to a park while I had an idea of going to a nicer place. I managed to persuade my classmates into accepting my idea. We did have a good time that day. Good opinions are worth sticking to because they can benefit us all. An alternative student version: We may have different opinions in organizing class activities. When I have a different opinion, I may choose to give it up and respect the opinion of the majority. The main reason for my choice is that being brought up in a culture emphasizing collectivism, I tend to sacrifice my own interest for the group benefit. Once we were left to decide whether to have a picnic in a park or go to a museum. I would love to go to a museum, but most of my classmates wanted to go for a picnic. Without hesitation, I decided to follow them and we did have lots of fun that day. Sometimes giving up a little can mean getting more. 2015年普通高等学校招生全国统一考试(浙江卷)自选模块英语部分试题 参考答案 题号:05 科目:英语 第一节((8分,每小题 2分) ①E ②B ③A ④D 第二节(2分) ⑤The main causes are technology and demography. 题号:06 科目:英语 (10分,每小题 1分) ①or ②chose ③after ④walking ⑤would ⑥how ⑦nothing(little) ⑧from ⑨not ⑩as 2015年普通高等学校招生全国统一考试(福建卷)参考答案 1~5 ACABC 6-10 BABCB 11-15ABCAB 16-20 CBAAC 21【答案】D 【解析】考查副词辨析,该句中 A,all全部;B,none没有一个;C,either两个中的任何一个; D,两个都不。根据该句话的意思:那个团队在哪个调查上得出的两个报告,但是两个报告里面 都没有任何有用的参考价值。所以选择 D. 22【答案】A 【解析】考察介词辨析。A.of表示关于,表示从属关系;B.on表示在…之上;C.to 表示对于; D.with表示和…在一起。该句意思为:他们共有的关于他们学校的记忆是校服。故选 A. 23【答案】B 【解析】考查形容词辨析 A.careless 不小心的;B.considerate 体贴的;C.Patient 有耐心的; D.generous慷慨的。该句意思为:麦克真是体贴,他通知我们他会迟点来以防我们着急。故选 B. 24【答案】A 【解析】考查名词辨析 A.blow打击;B.issue 问题;C.excuse借口;D.factor要素。该句意思为:这 个失败对他来说是一个打击,但是他没有泄气,很快振作恢复了原来的热情。故选 A 考点:考查名词辨析。 25【答案】B 【解析】考查短语辨析。A项 lies in在于;B项 accounts for说明,引起;C项 consisits of 包 括;D项,goes with 相伴。该句意思为:据说身体语言占了百分之五十五。而你所说的话只占百 分之七。故选择 B。 26【答案】A 【解析】考查时态和语态。根据句意:让我高兴的是,我从成百上千的参加者中被选中参加开 幕式。可以知道这里应该是被动语态,又因为被选中是过去发生的事情,这里只是对过去发生的 事情的一般描述,故用一般过去式,故选 A 考点:考查时态和语态辨析。 27【答案】D. 【解析】考查情态动词-have done 结构。A,must必须;B,can 可以;C,should 应该。这里 是情态-have done 的结构的虚拟语气,can-have-done”表示对过去的行为的怀疑,”用于疑问句, 译成“可能做过......吗?”;could have done表示对过去的事情进行假设,意思本来能够做的事情 而没有做;must have done 表示对过去的事情的肯定的推测,译成“一定做过某事”,该结构只 用于肯定句;should have done意思是“本来应该做某事,而实际没做”。该对话的意思是---对不 起妈妈我失败了。----哦,那太糟糕了,你应该做好充分准备的。故选择 D项。 28【答案】C 【解析】考查非谓语动词。这里是动词不定式形式的非谓语动词做目的状语。该句意思为:为 了更多了解到中国文化,杰克决定选择中国传统音乐作为选修课。故选 C 29【答案】B 【解析】考查宾语从句。句中 I是主语,wonder是谓语,how引导的宾语从句在整个句子中做 宾语。连接副词 How是宾语从句中的方式状语。句子是用表示方式的介词 by回答的,所以是针 对方式题提问的,故用 how。根据句意---我好奇玛丽在这些年是怎么保持着身材的。--通过每天 锻炼。故选 B。 30【答案】C 【解析】考查现在完成时。根据句意“---Peter 在哪里,我哪都找不到他”“---他在早餐的时候 就去了图书馆,然后自从那以后就一直在那里写论文”。从句意中我们可以知道 Peter是吃了早餐 之后去了图书馆,并且写论文,这个动作一直持续到仙子啊,所以用现在完成时。现在完成时表 示动作从某一时间开始,一直持续到现在。故选择 C。 31【答案】A 【解析】 试题分析:考查连词辨析。这里是一个 while引导的让步状语从句。A.while 尽管(引导让步状语 从句);B.unless 除非(通常用于条件状语从句);C.since 自从(引导时间状语从句);D.until 直到…才…(通常用在时间状语从句中)。结合句意:尽管学生都来自不同的国家,他们在夏令 营里面都相处得很好。故选 A 32【答案】D 【解析】考查短语辨析 A.in view of在…眼里;B.in need of需要;;C.in touch with和…接触; D. In harmony with 与…和谐相处。这四个短语后面都应该接名词,该句意思为:人的生活是自然的 一部分,因此,我们生存下来的唯一的方法就是与自然和谐相处。根据句意,故选 D 33【答案】C 【解析】考查非谓语动词。非谓语动词主要有动词不定式、现在分词和过去分词形式。其中现 在分词表示主动的关系,而过去分词通常表示被动的关系。根据该句意思:最近几年,一个叫做 infosphere的英语单词出现了,结合了“信息”和“气氛”这两个单词。这里的结合与前面新出现的单 词的关系是主动的,所以故选 C 考点:考查非谓语动词 34【答案】D 【解析】考查非限制性定语从句。根据句式结构看来出现了逗号,说明这是一个非限制性定语从 句。首先 That不能引导非限制性定语从句,who在引导非限制性定语从句的时候,通常在句中做 主语和宾语,指人。而 whom也指人,但是只能做宾语。Which引导非限制性定语从句的时候做 主语或者是并与。关系词呆滞前面的中国日报,在定语从句中做主语。该句的意思为:中国日报 有着全球的读者群,这说明全球越来越多的人想要了解中国。故选 D。 35【答案】C 【解析】 试题分析:考查句子衔接。A. That's right 对了;B. My pleasure 我的荣幸;C. Come on in 请进; D. Take it easy放松。该句意思为:--你好,布朗医生,我来的有点早,我应该在外面等么?---不, 进来吧。结合语境来看,布朗医生并不觉得病人来得早,所以让病人进来了。故选 C 完形填空 36. 【答案】B 【解析】考查形容词辨析。A,lonely 孤独的;B,great 伟大的 C,quiet 安静的;D,uneasy 不舒服的。直接上最简单的事情就是成为一个挑错者,然而,人生可以变得伟大当你不忙与 找错误。故选择 B。 37.【答案】A 【解析】考查动词辨析 A.received 接受;B.answered回答;C.expected 期望;D.rejected排斥。 几年以前,我收到一个来自 17岁女孩 kerry 的信。故选 A 38.【答案】C 【解析】考查动词辨析 A.threatened威胁;B.interrupted 打扰;C.bothered麻烦;D.spoiled 娇 惯。这个叫做 Kerry的女孩说自己是一个世界级的找错者,她总是被各种事情困扰着。故选 C 39.【答案】D考查不定代词辨析 A.anything任何事;B.everything每一件事;C.something 某事; D.nothing没有事。人们总是做着那些惹她恼怒的事情,没有一件事是好的,故选 D 40.【答案】B 【解析】考查形容词辨析 A.caring关心的;B.boring糟糕的;C.interesting有趣的;D.surprising 令人惊奇的。 她具有高度的自我批评精神,她也会对自己的朋友挑毛病。她成为了一个非常 糟糕的人。故选 B 41.【答案】A 【解析】考查名词辨析 A.attitude 态度;B.plan 计划;C.measure 措施;D.explanation 解释。 不幸的,一场恐怖的事故发生改变了她的态度。故选 A 42.【答案】D 【解析】考查形容词辨析 A.urgent 经济的;B.unnecessary 不必要的;C.certain 确定的; D.impossible不可能的。她最好的朋友在一场车祸中受伤很严重,让事情处理起来变得不可能 的事情是,发生事故的前一天 Kerry拜访了她的朋友…故选 D 43.【答案】C 【解析】考查名词辨析 A.occasion场合;B.event事项;C.accident事故;D.adventure 冒险。 她最好的朋友在一场车祸中受伤很严重,让事情处理起来变得不可能的事情是,发生事故的 前一天,Kerry拜访了她的朋友,在这期间全程批评了她选择男朋友的眼光…故选 C 44.【答案】D 【解析】考查名词辨析 A.memory记忆;B.notice 注意;C.evidence证据;D.choice选择。发 生事故的前一天,Kerry拜访了她的朋友,在这期间全程批评了她选择男朋友的眼光、她生活 的方式、她与母亲联络时候的方式,还有各种其他她认为她需要表达的…故选 D 45.【答案】C 【解析】考查动词辨析 A.hear听;B.contribute贡献;C.express 表达;D.admit承认、许可。 发生事故的前一天,Kerry拜访了她的朋友,在这期间全程批评了她选择男朋友的眼光、她生 活的方式、她与母亲联络时候的方式,还有各种其他她认为她需要表达的。故选 C 46.【答案】A 【解析】考查短语辨析 A.aware of注意;B.afraid of担心;C.curious about对…好奇;D.confused about对…迷惑。直到他的朋友受了重伤,Kerry才注意到她挑毛病的习惯。故选 A 47.【答案】C 【解析】考查动词辨析 A.discuss讨论;B.realize 意识到;C.judge 评价;D.settle 解决,非常快 的,他学会了珍惜生命二不是对每件事情都刻薄。 48.【答案】B 【解析】考查名次辨析。A.family 家庭,B.life 生命;C.career事业;D.education教育。她能够 将她的新智慧用在她生活的其他方面。故选择 B. 49.【答案】C 【解析】考查介词辨析。A,so于是;B,or或者;C,but但是;D,for对于。可能我们中的 大多数人都不会极端的寻找错误,但是当我们诚实的面对的时候,我们可以尖锐的批判世界。 故选择 C。 50.【答案】D 【解析】考查形容词辨析 A.proud骄傲的;B.sure 确定的;C.hopeful有希望的;D.critical 批 判的。可能我们中的大多数都不会极端的的寻找错误,但是当我们诚实的面对的时候,我们 可以尖锐的批判这个世界。故选 D 51.【答案】D 【解析】考查动词辨析 A.face 面对;B.create 创造;C.solve 解决;D.ignore 忽略。我并不是 在建议你无视问题。故选 D 52.【答案】B 【解析】考查形容词比较级辨析A.rarer更稀罕的;B.better更好的;C.stranger更奇怪的;D.worse 更坏的。我也不是在建议你假装事物比它们自身更好。故选 B 53.【答案】A 【解析】考查短语辨析 A.at least 至少;B.at last最后;C.by far到现在为止;D.so far迄今为 止。至少在大多数时间里,你学着容忍事物的原态。故选 A 54.【答案】B 【解析】考查名词辨析 A.task任务;B.deal处理、交易;C.result计划;D.duty责任。尤其是 当这不是一个特别大的事情。故选 B 55.【答案】A 【解析】考查名词辨析 A.practice 实践;B.speech演讲;C.rest休息;D.pity 遗憾。训练你自 己忍着不说那些刻薄的话,小小的实践一下,你会变得很擅长放手。故选 A 阅读 A 56.【答案】B 【解析】细节理解题。根据第二段第二句我们就可以知道,四个队伍,穿着滑稽搞笑的服装, 在 50米的路程中卷一个完整的奶酪。以及后面倒数第二句括号里的内容,这个奶酪是木头的。 故选择 B。 57.【答案】A 【解析】细节理解题。根据第二段第一句 Every year more than 10,000 people head for the city of Albuquerque, New Mexico.可以知道这是在新墨西哥发生的事情,从段落中第三句 Their destination is the Fiery Eood and BBQ Festival 可以知道人们到新墨西哥是为了参加 Fiery food festival。故选 A 58.【答案】B 【解析】细节理解题。根据文章第三段第二句 A week-long celebration leads up to an exciting tomato battle as the highlight of the week's events.可以知道,西红柿大战这个西班牙的节日的持 续时间是一周,故选 B 59.【答案】C 【解析】细节理解题。根据文章第二段倒数第二个破折号后面的文字 or any one of the thousands of products that are on show.可以知道你可以任意品尝展出的上千种的食物。故选 C 阅读 B 60.【答案】D 【解析】细节理解题。根据第一段第一句话 最后半句, for education was for the rich then ,the word became his school.因为教育是对有钱人开放的,所以世界成了爸爸的学校。可以推断出 作为一个穷人的爸爸付不起学费,所以选择 D。 61.【答案】C 【解析】分析推断题。这里的 it代指的是 news,代指的是前面的主语。根据该词附近的语境 可以知道我们会谈论一天的新闻,不论重要与否,都不是可以等闲视之的。故选 C 62.【答案】D 【解析】细节理解题。根据最后一段前两句 Later during my training as a future teacher studied with some of the most famous educators. They were imparting what Papa had known all along:在 我成为未来教师的实习中,我和一些很有名的教育者一同学习,他们传授着父亲所知道的—— 持续教育的意义。故选 D 63.【答案】A 【解析】分析推断题。根据最后一段倒数第二句 His technique has served me well all my life。他 的教育方式使我受用终生。二该句前一句中又提到父亲的教育方式就是持续性的教育,故选 A。 64.【答案】B 【解析】分析推断题。从全文来看,我们可以知道父亲的教育方针就是让我们持续性的学习, 故可以知道父亲是一个坚持着教育孩子的人。故选 B。 阅读 C 65.【答案】D 【解析】细节理解题。根据 benefits you can’t miss 标题下的第二个,见面百分之 30 的 tanya 语言学校的课程费用,故选择 D。 66.【答案】D 【解析】细节理解题。根据图中偏下部的内容 All bookings made before 12 September will receive free travel insurance for the entire family! 可以知道在 9 月 12 日之前预定可以得到免费保险, 10% OFF ALL BOOKINGS for departures from 5 to 11 September预定日期在 2015年 5到 11号 离开的可以享受百分之 10的减免。故选 D 67.【答案】A 【解析】细节理解题。根据主标题下面第一句With a registration fee of just $50 per child, children under the age of 12 can join Eagle Airways' FLY TO THE MOON CLUB as members. 可以知道一个 12岁以下的孩只要 50美元就可以加入俱乐部。故选 A 阅读 D 68.【答案】C 【解析】细节解析题。文章第一段第三句 Whatever our circumstances,we generally still have dreams,hopes and desire说不管在什么情况下,我们仍然有着梦想,希望和欲望。以及第二段 最后一句 If you drop that,you begin to feel a little”dead”inside because you’re dropping ‘you’,如果 你放弃了,你的内在就会变得没有生机,因为你放下了你自己。故选 C。 69.【答案】B 【解析】细节理解题。根据倒数第三段第一句 Decide on a new course and make one decision at a time.可以知道为了打破旧格局需要决定一个新目标并且立刻做出决定。故选 B 70.【答案】C 【解析】分析推断题。根据划线句子可知其本意是逃出你内心的牢笼。结合上文中在过多思考 之前快速决定和行动可以推测这个划线句子是指要从过多考虑的焦虑中解放自己。故选 C 71.【答案】A 【解析】分析推断题。从全文来看,作者主要是想要强调果断的行动而不是空想来实现自己的 梦想和欲望,故选 A.比起想要成功,行动更加容易。 阅读 E 72.【答案】C 【解析】细节题。根据文章第一句 Group exercise is one of the most effective ways to improve physical fitness and sustain a healthy lifestyle 团体运动是一项最有效增加身体健康保持健康的 生活方式,故选择 C。 73.【答案】A 【解析】分析推断题。从划线短语所在句子你可以参加一个 upbeat的集体健康课,那将会使得 你正确的锻炼自己。前半句 if you're tired of wandering around the gym wasting time and becoming bored, 是一个 if引导的条件句,意思是假如你对于在健身房游荡感到心烦并且觉得 这很浪费时间。所以这个单词应该是与浪费时间和让你心烦相反的意思,故选 A 74.【答案】D 【解析】细节理解题。根据最后一段第一句 Surrounding yourself with people who'll provide you with respect support can be very beneficial while working towards reaching health and fitness goals. 在以健康为目标锻炼的时候和那些你能够给你尊敬和支持的人们在一起是很有益处的,故选 D 75.【答案】B 【解析】分析推断题。文章的主要内容是在说团体运动的好处,团体运动包含的好处主要是因 为大家在一起运动,不仅生理上会有一些好处,而且团内其他成员会给你支持,教练也会给 你们一些信息上的支持,故选 B 短文填词 【答案】 76. happens 77. are 78. first 79. with 80. what 81. helpful/beneficial 82. better 83. advice 84.apology 85.in 【解析】 76.happens考察选词填空。Happen意思是发生,前面的 this代指前一句中的和别人发生分歧。因 为该句是一般现在时,而且主语是 this所以谓语动词应该用第三人称单数。该句意思为:当这种 情况发生了,重要的事情就是不要让一场平静的讨论变成热力的争论。 77.are考察固定结构。固定结构 Here are……表示这里有……,该句意思为:这里是一些给你的小 建议。 78.first考察形容词。因为该空前有冠词 the该空后有名词 thing,所以中间只能是形容词或者副词。 因为该句是第二段的第一句,并且后文中有表示层次的词 second,所以可以推断这里应该是 first。 79.with考察固定短语。固定短语 Share with 意思是和…分享。该句意思为:想象你是一个学生并 且和其他一个你认为她不愿意做她那份家务的学生合租一间公寓。 80.what考察特殊疑问句引导词,what引导的是一个特殊疑问句,表示什么。该句意思为:看看, 你从来不做家务,你究竟打算怎么做? 81.helpful/beneficial考察形容词。根据所给提示有帮助的,helpful或者 beneficial都有有利于的意 思。该句意思为:这样说就比较有帮助(不破坏关系)…… 82.better考察固定短语 had better do sth.表示最好做。该句意思为:我认为我们最好再看看,怎么 划分家务比较好。 83.advice考察名词。Advice 意思是建议。从第一段最后一句中我们可以知道作者下面是在给我们 提建议,而且这里有提示是第二个,所以这里应该是和建议有关的。 84.apology考查名词。Apology 意思是道歉,名词。Makea apology 表示道歉。该句意思为:只要 道歉,然后这事就过去了。 85.in考查固定短语。固定短语 In the future 表示在未来。该句意思为:如果你那样做,在未来, 一些人会对你更加尊敬。 写作 【答案】 In the picture ,we can see a boy in worn clothes,sitting at a shabby wooden table with a pile of books on it.A weak ray of light came in through a small hole in the wall and he was absorbed in his reading. This is a well-known story from ancient Chinese idiom.The boy, being poor ,couldn’t afford even a candle,so he bored a hole in the wall to “steal” light from his neighbour’s house to read at night.The moral of the story is spare no effort to acquire knowledge and never get discouraged easily no matter how difficult the situation may be Of course, things are totally different today It is not the story itself but what is reflected in the story that counts.Hard work pays off.We should take pains to improve ourselves through learning and get perpared for the future. 2015年普通高等学校招生全国统一考试(湖南卷)参考答案 PartⅠ: Section A 1-5 ACBAA 6-10 BABCC 11-15 BCACB Section B 16: Club 17: Monday 18: background 19: 5 pages 20: 750 Part Ⅱ: Section A 21-25: BABDD 26-30: CACDA 31-35: CDCAB Section B 36-40: DABAC 41-45: DCDBA 46-47: CB Section C 48. if 49. the 50. and 51. shouldn't 52. more 53. with 54. how 55. you Part Ⅲ 56-60 CBBDA 61-65 BDBAC 66-70 CACDA Part Ⅳ Section A 71. Select 72. inadequate 73. word information 74. desk dictionary 75. extra features 76. large and heavy 77. schools and libraries 78. admitting new words 79. check 80. access to Section B 81. We should use them to take optional subjects. 82. We will never know if we are interested or talented in a subject without trying it. 83. It was built incorrectly and broke in the kiln. 84. It will enrich our mind and show colleges we are diverse students. Section C As a student, I can tell you that there is nothing better than being praised by my teacher before my classmates. Today,the teacher praised me, because i discipline is very seriously. I was quite pleased, unique sense of accomplishment. I can't wait to tell my mom as I got home. Mother is also happy for me, I hope I continue to work hard. Although the teacher praised me, but I am not proud. Because, the only better in the world, no best. I will always strive to climb one after another in the peak of life. 2015年普通高等学校招生全国统一考试(湖北卷)参考答案 第一部分:听力(每小题 1.5分,满分 30分) 1. B 2. A 3. B 4. C 5. C 6. B 7. B 8. A 9. A 10. C 11. C 12. A 13. A 14. C 15. A 16. B 17. A 18. C 19. C 20. A 第二部分:词汇知识运用 第一节:多项选择(每小题 1分,满分 10分) 21. A 22. C 23. D 24. B 25. C 26. D 27. B 28. C 29. A 30. A 第二节:完形填空(每小题 1分,满分 20分) 31. D 32. B 33. A 34. D 35. B 36. D 37. B 38. A 39. C 40. B 41. C 42.D 43. C 44. A 45. C 46. A 47. B 48. A 49. C 50. C 第三部分:阅读理解(每小题 2分,满分 40分) A篇:51. C 52. B 53. D 54. B B篇:55. C 56. B 57. A 58. D C篇:59. D 60. B 61. A 62. C D篇:63. A 64. C 65. D 66. B E篇:67. D 68. C 69. D 70. A 第四部分:书面表达 第一节:完成句子(每小题 2分,满分 20分) 71. would not have picked 72. Looking at , When/While (she was) looking at 73. with whom Jack shared/who shared with Jack 74. came the teacher’s sweet voice 75. Whatever she says 76. whether/if you can undertake 77. had (already) been finished 78. It was in this lake 79. Although/Though he is known 80. to have been formed 第二节:短文写作(满分 30分) One Possible Version Questioning can lead to searching for an answer. If we are in the habit of raising doubts as to whether what we are told is true, we can find the correct answer and learn more. Two years ago, our teacher offered us an answer to a difficult math problem in class. Although the answer seemed a little strange, nobody but I doubted it. Thinking that his solution might be wrong, I carefully analyzed the problem and tried to work it out in a different way. Half an hour later, I managed to find the correct answer. When I showed my answer to him, the teacher praised me for my independent thinking. From this experience I have learnt that questioning can serve as a bridge that helps us to seek the truth. 2015年普通高等学校招生全国统一考试(四川卷)参考答案 1-5 CACBA 6-10 DDBCD 11-15 BCCAD 16-20 CBDAA 21-25 BADAC 26-30 BBDCA 31-35 CADBD 36-40 BCBAB 41-45 CBBAA 46-50 CDADD 51-55 CABGE 阅读表达 56.He felt excited. 57.Much/A lot of money[来源:学|科|网 Z|X|X|K] 58.He considered buying books. 59.He ate and drank. 60.It was under the apple tree in the backyard. 短文改错 61.加上 to 62. tell--told 63. friend--friends 64.sing--singing 65. many--much 66.so--but 67. How--What 68. are--were] 69. him--them 70. 去掉 with 书面表达 Hi Sharon, This is Li Xia. I learned from your post that you want to improve your Mandarin. I am quite interested in it . I think I’m fit for it. As a student, I have been learning Mandarin for many years. In addition ,I am Chinese, which means Mandarin is a must for me to communicate with others in my daily life. So I’m quite confident that I can help you learn Mandarin. Learning Mandarin takes time, so if you want to learn it well ,you should spend much time practicing Mandarin everyday. As the saying goes, practice makes perfect. Besides, you can also read some books in Chinese、see some Chinese films and listen to some Chinese radio programmes. Speaking and listening are very important for a language learning, by which you are sure to make great progress. In your post, you say you can teach English as a reward. It is just what I want. I’m eager to improve my English. So, if you allow me to help you learn Mandarin, you can also help me improve my English.We can contact each other by video chat every night for half an hour in English and half an hour i n Chinese, so that we can improve our oral speaking. Hope for you early reply! Yours Sincerely Li Xia 2015年普通高等学校招生全国统一考试(重庆卷)参考答案 1-5 BBCBA 6-10 CDACD 11-15 CADBD 16-20 BDCAD 21-25 BCCCD 26-30 DABAB 31-35 DCADB 36-40 CBDCC 41-45 DDABC 46-50 AACDB 51-55 BCADA 写作一 Were I three years younger than I am now, I would strongly recommend that I set more practical and specific goals. As an old saying goes, living without a clear and achievable aim is like sailing without a compass. Compared with many abstract objectives, practical ones can bring us more courage and confidence whenever we make one step forward. If only I could go back and reset my goals. 写作二 Learning that you are organizing this activity to share books worldwide, I am writing to apply for it for the simple reason that I love reading and I own a large number of books. You’ve introduced two ways to share books: wild release and controlled release. Of the two, the latter wins my favor. By the means of controlled release, I only need to deliver books to other participants, not only are books shared but it can build up a bridge connecting readers from different parts of the world as well. If you could provide more specific information, I would be well grateful. Looking forward to your reply. 2015年普通高等学校招生全国统一考试(陕西卷)参考答案 1-5 ACBCD 6-10 DGCAE 11-15 CBBAD 16-20 DABAC 21-25 DBCAA 26-30 CDADB 31-35 BBCAB 36-40 CBCDC 41-45 DADBB 46-50 ABBCD 51-55 ACDBD 56-60 AABCA 61-65 AEFCB 第一节 单词拼写 66. miss 67. proud 68. repeat 69. treat 70. income 71. Chemistry 72. safety 73. accepted 74. hospitals 75. Politely 第二节 短文改错 76.去掉 in 77.anything 改为 something 78.better改成 best 79.decide改成 decided 80.step 改为 steps 81.well改成 good 82.after改成 when/while 83.a改为 the 84.just后加 to 85.enjoy改为 enjoying 第三节 书面表达 Dear MS Chen , I’m Li Hua, a student from Class2, Grade 3. I’m writing to apply for the position as a student volunteer. I really want to obtain this precious opportunity because, by offering my service, I will be able to improve my organizational ability, communication skills as well as my confidence in speaking English in public. As an outgoing girl, I get along well with my classmates. Besides, I have such a good command of English that I am elected as assistant to my English teacher. Successfully, I have lent a helping hand to her in several English activities of my class, which have been appreciated by both teachers and classmates. I am sure that I can perfectly live up to your expectations. I will be grateful if you could give me a kind consideration. Looking forward to your reply. Yours truly, Li Hua

资料: 4.5万

进入主页

人气:

10000+的老师在这里下载备课资料